Ch.17 Labor and Birth Complications, Chapter 16: Nursing Care of the Family during Labor and Birth. OB. Exam 3, CH 15, Ch 14 Pain Management, Ch. 13 Labor and Birth Processes, Ch.11 Pregnancy at risk: Preexisting conditions, Ch 10 Assessment of High...

¡Supera tus tareas y exámenes ahora con Quizwiz!

Which is the acceptable mg/dl level, or below this level, low-density lipoprotein (LDL) cholesterol level for a child from a family with heart disease? _____ Record your answer as a whole number.

110

*A mother shares with the clinic nurse about her 4-year-old son who has acute diarrhea. She has been giving him the antidiarrheal drug loperamide (Imodium A-D). The nurse's response should be based on knowledge that this drug is: a. Not indicated. b. Indicated because it slows intestinal motility. c. Indicated because it decreases diarrhea. d. Indicated because it decreases fluid and electrolyte losses.

A

*A young child is brought to the emergency department with severe dehydration secondary to acute diarrhea and vomiting. Therapeutic management of this child will begin with: a. Intravenous fluids. b. Oral rehydration solution (ORS). c. Clear liquids, 1 to 2 ounces at a time. d. Administration of antidiarrheal medication.

A

*An important nursing consideration in the care of a child with celiac disease is to: a. Refer to a nutritionist for detailed dietary instructions and education. b. Help the child and family understand that diet restrictions are usually only temporary. c. Teach proper hand washing and Standard Precautions to prevent disease transmission. d. Suggest ways to cope more effectively with stress to minimize symptoms.

A

*Bismuth subsalicylate, clarithromycin, and metronidazole are prescribed for a child with a peptic ulcer to: a. Eradicate Helicobacter pylori. b. Treat epigastric pain. c. Coat gastric mucosa. d. Reduce gastric acid production.

A

*Nurses must be alert for increased fluid requirements when a child has: a. Fever. b. Congestive heart failure. c. Mechanical ventilation. d. Increased intracranial pressure (ICP).

A

*The best chance of survival for a child with cirrhosis is: a. Liver transplantation. b. Treatment with immune globulin. c. Treatment with corticosteroids. d. Provision of nutritional support.

A

*The earliest clinical manifestation of biliary atresia is: a. Jaundice. c. Hepatomegaly. b. Vomiting. d. Absence of stooling.

A

*The nurse is caring for a boy with probable intussusception. He had diarrhea before admission but, while waiting for administration of air pressure to reduce the intussusception, he passes a normal brown stool. The most appropriate nursing action is to: a. Notify the practitioner. b. Measure abdominal girth. c. Auscultate for bowel sounds. d. Take vital signs, including blood pressure.

A

*The nurse is caring for a neonate with a suspected tracheoesophageal fistula. Nursing care should include: a. Elevating the head to facilitate secretion drainage b. Elevating the head for feedings. c. Feeding glucose water only. d. Avoiding suctioning unless the infant is cyanotic.

A

*What food choice by the parent of a 2-year-old child with celiac disease indicates a need for further teaching? a. Oatmeal b. Rice cake c. Corn muffin d. Meat patty

A

*What is characterized by a chronic inflammatory process that may involve any part of the gastrointestinal (GI) tract from mouth to anus? a. Crohn's disease b. Meckel's diverticulum c. Ulcerative colitis d. Irritable bowel syndrome

A

1. Spastic cerebral palsy is characterized by what presentation? a. Hypertonicity and poor control of posture, balance, and coordinated motion b. Athetosis and dystonic movements c. Wide-based gait and poor performance of rapid, repetitive movements d. Tremors and lack of active movement

A

11. A young boy has just been diagnosed with pseudohypertrophic muscular dystrophy. The management plan should include which intervention? a. Recommending genetic counseling b. Explaining that the disease is easily treated c. Suggesting ways to limit the use of muscles d. Assisting the family in finding a nursing facility to provide his care

A

3. What is the most common problem for children born with a myelomeningocele? a. Neurogenic bladder b. Intellectual impairment c. Respiratory compromise d. Cranioschisis

A

4. Which problem is most often associated with myelomeningocele? a. Hydrocephalus b. Craniosynostosis c. Biliary atresia d. Esophageal atresia

A

9. Latex allergy is suspected in a child with spina bifida. What intervention should be included in the child's plan of care? a. Avoiding using any latex product b. Using only nonallergenic latex products c. Administering medication for long-term desensitization d. Teaching the family about long-term management of asthma

A

A beneficial effect of administering digoxin (Lanoxin) is that it: a. Decreases edema. c. Increases heart size. b. Decreases cardiac output. d. Increases venous pressure.

A

A child has a chronic, nonproductive cough and diffuse wheezing during the expiratory phase of respiration. This suggests: a. Asthma. c. Bronchiolitis. b. Pneumonia. d. Foreign body in the trachea.

A

A child with cystic fibrosis (CF) receives aerosolized bronchodilator medication. When should this medication be administered? a. Before chest physiotherapy (CPT) c. Before receiving 100% oxygen b. After CPT d. After receiving 100% oxygen

A

A child with pulmonary atresia exhibits cyanosis with feeding. On reviewing this child's laboratory values, the nurse is not surprised to notice which abnormality? a. Polycythemia c. Dehydration b. Infection d. Anemia

A

A major clinical manifestation of rheumatic fever is: a. Polyarthritis. b. Osler's nodes. c. Janeway spots. d. Splinter hemorrhages of distal third of nails.

A

A normal uterine activity pattern in labor is characterized by: a. contractions every 2 to 5 minutes. b. contractions lasting about 2 minutes. c. contractions about 1 minute apart. d. a contraction intensity of about 1000 mm Hg with relaxation at 50 mm Hg.

A

A nurse is charting that a hospitalized child has labored breathing. Which describes labored breathing? a. Dyspnea c. Hypopnea b. Tachypnea d. Orthopnea

A

A nurse is conducting an in-service on asthma. Which statement is the most descriptive of bronchial asthma? a. There is heightened airway reactivity. b. There is decreased resistance in the airway. c. The single cause of asthma is an allergic hypersensitivity. d. It is inherited.

A

A nurse is interpreting the results of a tuberculin skin test (TST) on an adolescent who is human immunodeficiency virus (HIV) positive. Which induration size indicates a positive result for this child 48 to 72 hours after the test? a. 5 mm c. 15 mm b. 10 mm d. 20 mm

A

A school-age child has had an upper respiratory tract infection for several days and then began having a persistent dry, hacking cough that was worse at night. The cough has become productive in the past 24 hours. This is most suggestive of: a. Bronchitis. c. Viral-induced asthma. b. Bronchiolitis. d. Acute spasmodic laryngitis.

A

Abdominal thrusts (the Heimlich maneuver) are recommended for airway obstruction in children older than: a. 1 year. c. 8 years. b. 4 years. d. 12 years.

A

An 18-month-old child is seen in the clinic with AOM. Trimethoprim-sulfamethoxazole (Bactrim) is prescribed. Which statement made by the parent indicates a correct understanding of the instructions? a. "I should administer all the prescribed medication." b. "I should continue medication until the symptoms subside." c. "I will immediately stop giving medication if I notice a change in hearing." d. "I will stop giving medication if fever is still present in 24 hours."

A

An appropriate nursing intervention when caring for a child with pneumonia is to: a. Encourage rest. b. Encourage the child to lie on the unaffected side. c. Administer analgesics. d. Place the child in the Trendelenburg position.

A

An infant's parents ask the nurse about preventing otitis media (OM). What should the nurse recommend? a. Avoid tobacco smoke. b. Use nasal decongestant. c. Avoid children with OM. d. Bottle-feed or breastfeed in supine position.

A

Decongestant nose drops are recommended for a 10-month-old infant with an upper respiratory tract infection. Instructions for nose drops should include: a. Avoiding use for more than 3 days. b. Keeping drops to use again for nasal congestion. c. Administering drops until nasal congestion subsides. d. Administering drops after feedings and at bedtime.

A

Fetal well-being during labor is assessed by: a. the response of the fetal heart rate (FHR) to uterine contractions (UCs). b. maternal pain control. c. accelerations in the FHR. d. an FHR above 110 beats/min.

A

In assisting with the two factors that have an effect on fetal status (i.e., pushing and positioning), nurses should: a. encourage the woman's cooperation in avoiding the supine position. b. advise the woman to avoid the semi-Fowler position. c. encourage the woman to hold her breath and tighten her abdominal muscles to produce a vaginal response. d. instruct the woman to open her mouth and close her glottis, letting air escape after the push.

A

In providing nourishment for a child with cystic fibrosis (CF), which factor should the nurse keep in mind? a. Diet should be high in carbohydrates and protein. b. Diet should be high in easily digested carbohydrates and fats. c. Most fruits and vegetables are not well tolerated. d. Fats and proteins must be greatly curtailed.

A

In which situation is there the greatest risk that a newborn infant will have a congenital heart defect (CHD)? a. Trisomy 21 detected on amniocentesis b. Family history of myocardial infarction c. Father has type 1 diabetes mellitus d. Older sibling born with Turner's syndrome

A

Parents of a 3-year-old child with congenital heart disease are afraid to let their child play with other children because of possible overexertion. The nurse's reply should be based on knowing that: a. The child needs opportunities to play with peers. b. The child needs to understand that peers' activities are too strenuous. c. Parents can meet all the child's needs. d. Constant parental supervision is needed to avoid overexertion.

A

Perinatal nurses are legally responsible for: a. correctly interpreting fetal heart rate (FHR) patterns, initiating appropriate nursing interventions, and documenting the outcomes. b. greeting the patient on arrival, assessing her, and starting an intravenous line. c. applying the external fetal monitor and notifying the care provider. d. making sure that the woman is comfortable.

A

The earliest recognizable clinical manifestation of cystic fibrosis (CF) is: a. Meconium ileus. b. History of poor intestinal absorption. c. Foul-smelling, frothy, greasy stools. d. Recurrent pneumonia and lung infections.

A

The nurse caring for a laboring woman is aware that maternal cardiac output can be increased by: a. change in position. b. oxytocin administration. c. regional anesthesia. d. intravenous analgesic.

A

The nurse caring for the laboring woman should understand that early decelerations are caused by: a. altered fetal cerebral blood flow. b. umbilical cord compression. c. uteroplacental insufficiency. d. spontaneous rupture of membranes.

A

The nurse closely monitors the temperature of a child with nephrosis. The purpose of this is to detect an early sign of: a. Infection. c. Encephalopathy. b. Hypertension. d. Edema.

A

The nurse is admitting a child with rheumatic fever. Which therapeutic management should the nurse expect to implement? a. Administering penicillin b. Avoiding salicylates (aspirin) c. Imposing strict bed rest for 4 to 6 weeks d. Administering corticosteroids if chorea develops

A

The nurse is caring for an infant with congestive heart disease (CHD). The nurse should plan which intervention to decrease cardiac demands? a. Organize nursing activities to allow for uninterrupted sleep. b. Allow the infant to sleep through feedings during the night. c. Wait for the infant to cry to show definite signs of hunger. d. Discourage parents from rocking the infant

A

The nurse is conducting an assessment on a school-age child with urosepsis. Which assessment finding should the nurse expect? a. Fever with a positive blood culture c. Oliguria and hypertension b. Proteinuria and edema d. Anemia and thrombocytopenia

A

The nurse is doing a neurologic assessment on a 2-month-old infant following a car accident. Moro, tonic neck, and withdrawal reflexes are present. The nurse should recognize that these reflexes suggest: A. neurologic health. B. severe brain damage. C. decorticate posturing. D. decerebrate posturing.

A

The nurse is talking to a parent of an infant with heart failure about feeding the infant. Which statement about feeding the child is correct? a. "You may need to increase the caloric density of your infant's formula." b. "You should feed your baby every 2 hours." c. "You may need to increase the amount of formula your infant eats with each feeding." d. "You should place a nasal oxygen cannula on your infant during and after each feeding."

A

The nurse knows that proper placement of the tocotransducer for electronic fetal monitoring is located: a. over the uterine fundus. b. on the fetal scalp. c. inside the uterus. d. over the mother's lower abdomen.

A

The nurse providing care for the laboring woman comprehends that accelerations with fetal movement: a. are reassuring. b. are caused by umbilical cord compression. c. warrant close observation. d. are caused by uteroplacental insufficiency.

A

The nurse providing care for the laboring woman should understand that amnioinfusion is used to treat: a. variable decelerations. b. late decelerations. c. fetal bradycardia. d. fetal tachycardia.

A

The primary clinical manifestations of acute renal failure are: a. Oliguria and hypertension. c. Proteinuria and muscle cramps. b. Hematuria and pallor. d. Bacteriuria and facial edema

A

Therapeutic management of nephrosis includes: a. Corticosteroids. c. Long-term diuretics. b. Antihypertensive agents. d. Increased fluids to promote diuresis

A

What is the nurse's first action when planning to teach the parents of an infant with a congenital heart defect (CHD)? a. Assess the parents' anxiety level and readiness to learn. b. Gather literature for the parents. c. Secure a quiet place for teaching. d. Discuss the plan with the nursing team.

A

What should the nurse include in a teaching plan for the parents of a child with vesicoureteral reflux? a. The importance of taking prophylactic antibiotics b. Suggestions for how to maintain fluid restrictions c. The use of bubble baths as an incentive to increase bath time d. The need for the child to hold urine for 6 to 8 hours

A

What should the nurse recommend to prevent urinary tract infections in young girls? a. Wearing cotton underpants b. Limiting bathing as much as possible c. Increasing fluids; decreasing salt intake d. Cleansing the perineum with water after voiding

A

When a child has chronic renal failure, the progressive deterioration produces a variety of clinical and biochemical disturbances that eventually are manifested in the clinical syndrome known as: a. Uremia. c. Proteinuria. b. Oliguria. d. Pyelonephritis.

A

When using intermittent auscultation (IA) to assess uterine activity, the nurse should be cognizant that: a. the examiner's hand should be placed over the fundus before, during, and after contractions. b. the frequency and duration of contractions is measured in seconds for consistency. c. contraction intensity is given a judgment number of 1 to 7 by the nurse and patient together. d. the resting tone between contractions is described as either placid or turbulent.

A

Which deceleration of the fetal heart rate would not require the nurse to change the maternal position? a. Early decelerations b. Late decelerations c. Variable decelerations d. It is always a good idea to change the woman's position.

A

Which diagnostic test allows visualization of the renal parenchyma and renal pelvis without exposure to external beam radiation or radioactive isotopes? a. Renal ultrasound b. Computed tomography c. Intravenous pyelography d. Voiding cystourethrography

A

Which drug is an angiotensin-converting enzyme (ACE) inhibitor? a. Captopril (Capoten) c. Spironolactone (Aldactone) b. Furosemide (Lasix) d. Chlorothiazide (Diuril)

A

Which drug would be used to treat a child who has increased intracranial pressure (ICP) resulting from cerebral edema? a. Mannitol b. Atropine sulfate c. Epinephrine hydrochloride d. Sodium bicarbonate

A

Which intervention should be included in the plan of care for an infant with the nursing diagnosis of Excess Fluid Volume related to congestive heart failure? a. Weigh the infant every day on the same scale at the same time. b. Notify the physician when weight gain exceeds more than 20 g/day. c. Put the infant in a car seat to minimize movement. d. Administer digoxin (Lanoxin) as ordered by the physician.

A

Which maternal condition is considered a contraindication for the application of internal monitoring devices? a. Unruptured membranes b. Cervix dilated to 4 cm c. External monitors in current use d. Fetus with a known heart defect

A

Which painful, tender, pea-sized nodules may appear on the pads of the fingers or toes in bacterial endocarditis? a. Osler's nodes c. Subcutaneous nodules b. Janeway lesions d. Aschoff's nodules

A

Which statement expresses accurately the genetic implications of cystic fibrosis (CF)? a. If it is present in a child, both parents are carriers of this defective gene. b. It is inherited as an autosomal dominant trait. c. It is a genetic defect found primarily in non-Caucasian population groups. d. There is a 50% chance that siblings of an affected child also will be affected.

A

Which structural defects constitute tetralogy of Fallot? a. Pulmonic stenosis, ventricular septal defect, overriding aorta, right ventricular hypertrophy b. Aortic stenosis, ventricular septal defect, overriding aorta, right ventricular hypertrophy c. Aortic stenosis, atrial septal defect, overriding aorta, left ventricular hypertrophy d. Pulmonic stenosis, ventricular septal defect, aortic hypertrophy, left ventricular hypertrophy

A

Which test is never performed on a child who is awake? a. Oculovestibular response b. Doll's head maneuver c. Funduscopic examination for papilledema d. Assessment of pyramidal tract lesions

A

While evaluating an external monitor tracing of a woman in active labor, the nurse notes that the fetal heart rate (FHR) for five sequential contractions begins to decelerate late in the contraction, with the nadir of the decelerations occurring after the peak of the contraction. The nurse's first priority is to: a. change the woman's position. b. notify the care provider. c. assist with amnioinfusion. d. insert a scalp electrode.

A

he nurse is assessing a child post-cardiac catheterization. Which complication might the nurse anticipate? a. Cardiac arrhythmia c. Congestive heart failure b. Hypostatic pneumonia d. Rapidly increasing blood pressure

A

Spontaneous termination of a pregnancy is considered to be an abortion if: a. The pregnancy is less than 20 weeks. b. The fetus weighs less than 1000 g. c. The products of conception are passed intact. d. No evidence exists of intrauterine infection.

A An abortion is the termination of pregnancy before the age of viability (20 weeks). The weight of the fetus is not considered because some older fetuses may have a low birth weight. A spontaneous abortion may be complete or incomplete. A spontaneous abortion may be caused by many problems, one being intrauterine infection.

The priority nursing intervention when admitting a pregnant woman who has experienced a bleeding episode in late pregnancy is to: a. Assess fetal heart rate (FHR) and maternal vital signs b. Perform a venipuncture for hemoglobin and hematocrit levels c. Place clean disposable pads to collect any drainage d. Monitor uterine contractions

A Assessment of the FHR and maternal vital signs will assist the nurse in determining the degree of the blood loss and its effect on the mother and fetus. The most important assessment is to check mother/fetal well-being. The blood levels can be obtained later. It is important to assess future bleeding; however, the top priority remains mother/fetal well-being. Monitoring uterine contractions is important but not the top priority.

Approximately 10% to 15% of all clinically recognized pregnancies end in miscarriage. Which is the most common cause of spontaneous abortion? a. Chromosomal abnormalities c. Endocrine imbalance b. Infections d. Immunologic factors

A At least 50% of pregnancy losses result from chromosomal abnormalities that are incompatible with life. Maternal infection may be a cause of early miscarriage. Endocrine imbalances such as hypothyroidism or diabetes are possible causes for early pregnancy loss. Women who have repeated early pregnancy losses appear to have immunologic factors that play a role in spontaneous abortion incidents.

Your patient is being induced because of her worsening preeclampsia. She is also receiving magnesium sulfate. It appears that her labor has not become active despite several hours of oxytocin administration. She asks the nurse, "Why is it taking so long?" The most appropriate response by the nurse would be: a. "The magnesium is relaxing your uterus and competing with the oxytocin. It may increase the duration of your labor." b. "I don't know why it is taking so long." c. "The length of labor varies for different women." d. "Your baby is just being stubborn."

A Because magnesium sulfate is a tocolytic agent, its use may increase the duration of labor. The amount of oxytocin needed to stimulate labor may be more than that needed for the woman who is not receiving magnesium sulfate. "I don't know why it is taking so long" is not an appropriate statement for the nurse to make. Although the length of labor does vary in different women, the most likely reason this woman's labor is protracted is the tocolytic effect of magnesium sulfate. The behavior of the fetus has no bearing on the length of labor.

Because pregnant women may need surgery during pregnancy, nurses should be aware that: a. The diagnosis of appendicitis may be difficult because the normal signs and symptoms mimic some normal changes in pregnancy. b. Rupture of the appendix is less likely in pregnant women because of the close monitoring. c. Surgery for intestinal obstructions should be delayed as long as possible because it usually affects the pregnancy. d. When pregnancy takes over, a woman is less likely to have ovarian problems that require invasive responses.

A Both appendicitis and pregnancy are linked with nausea, vomiting, and increased white blood cell count. Rupture of the appendix is two to three times more likely in pregnant women. Surgery to remove obstructions should be done right away. It usually does not affect the pregnancy. Pregnancy predisposes a woman to ovarian problems.

Which order should the nurse expect for a patient admitted with a threatened abortion? a. Bed rest b. Ritodrine IV c. NPO d. Narcotic analgesia every 3 hours, prn

A Decreasing the woman's activity level may alleviate the bleeding and allow the pregnancy to continue. Ritodrine is not the first drug of choice for tocolytic medications. There is no reason for having the woman placed NPO. At times dehydration may produce contractions, so hydration is important. Narcotic analgesia will not decrease the contractions. It may mask the severity of the contractions.

A woman with severe preeclampsia has been receiving magnesium sulfate by intravenous infusion for 8 hours. The nurse assesses the woman and documents the following findings: temperature of 37.1° C, pulse rate of 96 beats/min, respiratory rate of 24 breaths/min, blood pressure (BP) of 155/112 mm Hg, 3+ deep tendon reflexes, and no ankle clonus. The nurse calls the physician, anticipating an order for: a. Hydralazine. c. Diazepam. b. Magnesium sulfate bolus. d. Calcium gluconate.

A Hydralazine is an antihypertensive commonly used to treat hypertension in severe preeclampsia. Typically it is administered for a systolic BP greater than 160 mm Hg or a diastolic BP greater than 110 mm Hg. An additional bolus of magnesium sulfate may be ordered for increasing signs of central nervous system irritability related to severe preeclampsia (e.g., clonus) or if eclampsia develops. Diazepam sometimes is used to stop or shorten eclamptic seizures. Calcium gluconate is used as the antidote for magnesium sulfate toxicity. The client is not currently displaying any signs or symptoms of magnesium toxicity.

In planning care for women with preeclampsia, nurses should be aware that: a. Induction of labor is likely, as near term as possible. b. If at home, the woman should be confined to her bed, even with mild preeclampsia. c. A special diet low in protein and salt should be initiated. d. Vaginal birth is still an option, even in severe cases.

A Induction of labor is likely, as near term as possible; however, at less than 37 weeks of gestation, immediate delivery may not be in the best interest of the fetus. Strict bed rest is becoming controversial for mild cases; some women in the hospital are even allowed to move around. Diet and fluid recommendations are much the same as for healthy pregnant women, although some authorities have suggested a diet high in protein. Women with severe preeclampsia should expect a cesarean delivery.

The nurse caring for pregnant women must be aware that the most common medical complication of pregnancy is: a. Hypertension. c. Hemorrhagic complications. b. Hyperemesis gravidarum. d. Infections.

A Preeclampsia and eclampsia are two noted deadly forms of hypertension. A large percentage of pregnant women will have nausea and vomiting, but a relatively few have the severe form called hyperemesis gravidarum. Hemorrhagic complications are the second most common medical complication of pregnancy; hypertension is the most common.

In caring for the woman with disseminated intravascular coagulation (DIC), what order should the nurse anticipate? a. Administration of blood b. Preparation of the client for invasive hemodynamic monitoring c. Restriction of intravascular fluids d. Administration of steroids

A Primary medical management in all cases of DIC involves correction of the underlying cause, volume replacement, blood component therapy, optimization of oxygenation and perfusion status, and continued reassessment of laboratory parameters. Central monitoring would not be ordered initially in a client with DIC because this can contribute to more areas of bleeding. Management of DIC would include volume replacement, not volume restriction. Steroids are not indicated for the management of DIC.

What nursing diagnosis would be the most appropriate for a woman experiencing severe preeclampsia? a. Risk for injury to the fetus related to uteroplacental insufficiency b. Risk for eclampsia c. Risk for deficient fluid volume related to increased sodium retention secondary to administration of MgSO4 d. Risk for increased cardiac output related to use of antihypertensive drugs

A Risk for injury to the fetus related to uteroplacental insufficiency is the most appropriate nursing diagnosis for this client scenario. Other diagnoses include Risk to fetus related to preterm birth and abruptio placentae. Eclampsia is a medical, not a nursing, diagnosis. There would be a risk for excess, not deficient, fluid volume related to increased sodium retention. There would be a risk for decreased, not increased, cardiac output related to the use of antihypertensive drugs.

In caring for an immediate postpartum client, you note petechiae and oozing from her IV site. You would monitor her closely for the clotting disorder: a. Disseminated intravascular coagulation (DIC) b. Amniotic fluid embolism (AFE) c. Hemorrhage d. HELLP syndrome

A The diagnosis of DIC is made according to clinical findings and laboratory markers. Physical examination reveals unusual bleeding. Petechiae may appear around a blood pressure cuff on the woman's arm. Excessive bleeding may occur from the site of slight trauma such as venipuncture sites. These symptoms are not associated with AFE, nor is AFE a bleeding disorder. Hemorrhage occurs for a variety of reasons in the postpartum client. These symptoms are associated with DIC. Hemorrhage would be a finding associated with DIC and is not a clotting disorder in and of itself. HELLP is not a clotting disorder, but it may contribute to the clotting disorder DIC.

C

A 10-year-old boy has been hit by a car while riding his bicycle in front of the school. The school nurse immediately assesses airway, breathing, and circulation. The next nursing action should be to: a. Place on side. b. Take blood pressure. c. Stabilize neck and spine. d. Check scalp and back for bleeding.

C

A 3-year-old child is hospitalized after a near-drowning accident. The child's mother complains to the nurse, "This seems unnecessary when he is perfectly fine." The nurse's best reply is: a. "He still needs a little extra oxygen." b. "I'm sure he is fine, but the doctor wants to make sure." c. "The reason for this is that complications could still occur." d. "It is important to observe for possible central nervous system problems."

A C E

A 3-year-old child is status postshunt revision for hydrocephaly. Part of the discharge teaching plan for the parents is signs of shunt malformation. Which signs are of shunt malformation? (Select all that apply.) A. Personality change B. Bulging anterior fontanel C. Vomiting D. Dizziness E. Fever

4. The nurse is caring for an infant with myelomeningocele who is scheduled for surgical repair in the morning. Which early signs of infection should the nurse monitor on this infant? (Select all that apply.) a. Temperature instability b. Irritability c. Lethargy d. Bradycardia e. Hypertension

A B C

Which information should the nurse teach families about reducing exposure to pollens and dust (Select all that apply)? a. Replace wall-to-wall carpeting with wood and tile floors. b. Use an air conditioner. c. Put dust-proof covers on pillows and mattresses. d. Keep humidity in the house above 60%. e. Keep pets outside.

A B C

3. The nurse is conducting discharge teaching with parents of a preschool child with a myelomeningocele, repaired at birth, who is being discharged from the hospital after a urinary tract infection (UTI). Which should the nurse include in the discharge instructions related to management of the child's genitourinary function? (Select all that apply.) a. Continue to perform the clean intermittent catheterizations (CIC) at home. b. Administer the oxybutynin chloride as prescribed. c. Reduce fluid intake in the afternoon and evening hours. d. Monitor for signs of a recurrent UTI. e. Administer furosemide as prescribed.

A B D

1. A 14 year old is in the intensive care unit after a spinal cord injury 2 days ago. Which nursing care interventions are needed for this child? (Select all that apply.) a. Monitoring and maintaining systemic blood pressure. b. Administering corticosteroids. c. Minimizing environmental stimuli. d. Discussing long-term care issues with the family. e. Monitoring for respiratory complications.

A B E

2. Which assessment findings should the nurse note in a school-age child diagnosed with Duchenne's muscular dystrophy (DMD)? (Select all that apply.) a. Lordosis b. Gower's sign c. Kyphosis d. Scoliosis e. Waddling gait

A B E

A child has a total cholesterol level of 180 mg/dL. What dietary recommendations should the nurse make to the child and the child's parents (Select all that apply)? a. Replace whole milk with 2% or 1% milk b. Increase servings of red meat c. Increase servings of fish d. Avoid excessive intake of fruit juices e. Limit servings of whole grain

A C D

*Which interventions should a nurse implement when caring for a child with hepatitis (Select all that apply)? a. Provide a well-balanced, low-fat diet. b. Schedule playtime in the playroom with other children c. Teach parents not to administer any over-the-counter medications. d. Arrange for home schooling because the child will not be able to return to school. e. Instruct parents on the importance of good hand washing.

A C E

The nurse is caring for an infant with a suspected urinary tract infection. Which clinical manifestations would be observed (Select all that apply)? a. Vomiting b. Jaundice c. Failure to gain weight d. Swelling of the face e. Back pain f. Persistent diaper rash

A C F

An infant has developed staphylococcal pneumonia. Nursing care of the child with pneumonia includes which of the following? (Select all that apply). a. Cluster care to conserve energy b. Round-the-clock administration of antitussive agents c. Strict intake and output to avoid congestive heart failure d. Administration of antibiotics e. Placement in a mist tent

A D

A school-age child is admitted to the hospital with acute glomerulonephritis and oliguria. Which dietary menu items should be allowed for this child (Select all that apply)? a. Apples b. Bananas c. Cheese d. Carrot sticks e. Strawberries

A D E

An infant with hydrocephalus is hospitalized for surgical placement of a ventriculoperitoneal shunt. Which interventions should be included in the child's postoperative care (Select all that apply)? a. Observe closely for signs of infection. b. Pump the shunt reservoir to maintain patency. c. Administer sedation to decrease irritability. d. Maintain Trendelenburg position to decrease pressure on the shunt. e. Maintain an accurate record of intake and output. f. Monitor for abdominal distention.

A E F

C

A child has been seizure-free for 2 years. A father asks the nurse how much longer the child will need to take the antiseizure medications. The nurse includes which intervention in the response? a. Medications can be discontinued at this time. b. The child will need to take the drugs for 5 years after the last seizure. c. A stepwise approach will be used to reduce the dosage gradually. d. Seizure disorders are a lifelong problem. Medications cannot be discontinued.

C

A child is brought to the emergency department after experiencing a seizure at school. There is no previous history of seizures. The father tells the nurse that he cannot believe the child has epilepsy. The nurse's best response is: a. "Epilepsy is easily treated." b. "Very few children have actual epilepsy." c. "The seizure may or may not mean that your child has epilepsy." d. "Your child has had only one convulsion; it probably won't happen again."

D

A child is unconscious after a motor vehicle accident. The watery discharge from the nose tests positive for glucose. The nurse should recognize that this suggests: a. Diabetic coma. b. Brainstem injury. c. Upper respiratory tract infection. d. Leaking of cerebrospinal fluid (CSF).

Which collection of risk factors most likely would result in damaging lacerations including episiotomies ?

A first-time mother with reddish hair whose rapid labor was overseen by an obstetrician

A C D

A nurse should expect which cerebrospinal fluid (CSF) laboratory results on a child diagnosed with bacterial meningitis (Select all that apply)? a. Elevated white blood cell (WBC) count b. Decreased protein c. Decreased glucose d. Cloudy in color e. Increase in red blood cells (RBCs)

B

A school-age child has sustained a head injury and multiple fractures after being thrown from a horse. The child's level of consciousness is variable. The parents tell the nurse that they think their child is in pain because of periodic crying and restlessness. The most appropriate nursing action is to: a. Discuss with parents the child's previous experiences with pain. b. Discuss with practitioner what analgesia can be safely administered. c. Explain that analgesia is contraindicated with a head injury. d. Explain that analgesia is unnecessary when child is not fully awake and alert.

A

A toddler fell out of a second-story window. She had brief loss of consciousness and vomited four times. Since admission, she has been alert and oriented. Her mother asks why a computed tomography (CT) scan is required when she "seems fine." The nurse should explain that the toddler: a. May have a brain injury. b. May start having seizures. c. Needs this because of her age. d. Probably has a skull fracture.

C

A young child is having a seizure that has lasted 35 minutes. There is a loss of consciousness. The nurse should recognize that this is: A. absence seizure. B. generalized seizure. C. status epilepticus. D. simple partial seizure.

C

A young child's parents call the nurse after their child was bitten by a raccoon in the woods. The nurse's recommendation should be based on knowing that: a. The child should be hospitalized for close observation. b. No treatment is necessary if thorough wound cleaning is done. c. Antirabies prophylaxis must be initiated. d. Antirabies prophylaxis must be initiated if clinical manifestations appear.

The reported incidence of ectopic pregnancy in the United States has risen steadily over the past 2 decades. Causes include the increase in STDs accompanied by tubal infection and damage. The popularity of contraceptive devices such as the IUD has also increased the risk for ectopic pregnancy. The nurse who suspects that a patient has early signs of ectopic pregnancy should be observing her for symptoms such as (Select all that apply): a. Pelvic pain b. Abdominal pain c. Unanticipated heavy bleeding d. Vaginal spotting or light bleeding e. Missed period

A, B, D, E A missed period or spotting can easily be mistaken by the patient as early signs of pregnancy. More subtle signs depend on exactly where the implantation occurs. The nurse must be thorough in her assessment because pain is not a normal symptom of early pregnancy. As the fallopian tube tears open and the embryo is expelled, the patient often exhibits severe pain accompanied by intraabdominal hemorrhage. This may progress to hypovolemic shock with minimal or even no external bleeding. In about half of women, shoulder and neck pain results from irritation of the diaphragm from the hemorrhage.

A client who has undergone a dilation and curettage for early pregnancy loss is likely to be discharged the same day. The nurse must ensure that vital signs are stable, bleeding has been controlled, and the woman has adequately recovered from the administration of anesthesia. To promote an optimal recovery, discharge teaching should include (Select all that apply): a. Iron supplementation. b. Resumption of intercourse at 6 weeks following the procedure. c. Referral to a support group if necessary. d. Expectation of heavy bleeding for at least 2 weeks. e. Emphasizing the need for rest.

A, C, E The woman should be advised to consume a diet high in iron and protein. For many women iron supplementation also is necessary. Acknowledge that the client has experienced a loss, albeit early. She can be taught to expect mood swings and possibly depression. Referral to a support group, clergy, or professional counseling may be necessary. Discharge teaching should emphasize the need for rest. Nothing should be placed in the vagina for 2 weeks after the procedure. This includes tampons and vaginal intercourse. The purpose of this recommendation is to prevent infection. Should infection occur, antibiotics may be prescribed. The client should expect a scant, dark discharge for 1 to 2 weeks. Should heavy, profuse, or bright bleeding occur, she should be instructed to contact her provider.

42. Achieving and maintaining euglycemia comprise the primary goals of medical therapy for the pregnant woman with diabetes. These goals are achieved through a combination of diet, insulin, exercise, and blood glucose monitoring. The target blood glucose levels 1 hour after a meal should be: _________________

ANS: 130 to 140 mg/dL Target levels of blood glucose during pregnancy are lower than nonpregnant values. Accepted fasting levels are between 65 and 95 mg/dL, and 1-hour postmeal levels should be less than 130 to 140 mg/dL. Two-hour postmeal levels should be 120 mg/dL or less.

48. Step 6 a.Without adding air, withdraw the correct dose of NPH insulin. b.Gently rotate the insulin to mix it, and wipe the stopper. c.Inject air equal to the dose of NPH insulin into the vial, and remove the syringe. d.Inject air equal to the dose of regular insulin into the vial, and withdraw the medication. e.Check the insulin bottles for the expiration date. f.Wash hands.

ANS: A

30. Before the physician performs an external version, the nurse should expect an order for a: a.Tocolytic drug. b.Contraction stress test (CST) c.Local anesthetic. d.Foley catheter.

ANS: A A tocolytic drug will relax the uterus before and during version, thus making manipulation easier. CST is used to determine the fetal response to stress. A local anesthetic is not used with external version. The bladder should be emptied; however, catheterization is not necessary.

9. A woman is undergoing a nipple-stimulated contraction stress test (CST). She is having contractions that occur every 3 minutes. The fetal heart rate (FHR) has a baseline of approximately 120 beats/min without any decelerations. The interpretation of this test is said to be: a. Negative. c. Satisfactory. b. Positive. d. Unsatisfactory.

ANS: A Adequate uterine activity necessary for a CST consists of the presence of three contractions in a 10-minute time frame. If no decelerations are observed in the FHR pattern with the contractions, the findings are considered to be negative. A positive CST indicates the presence of repetitive later FHR decelerations. Satisfactory and unsatisfactory are not applicable terms.

13. Nurses should be aware that the biophysical profile (BPP): a. Is an accurate indicator of impending fetal death. b. Is a compilation of health risk factors of the mother during the later stages of pregnancy. c. Consists of a Doppler blood flow analysis and an amniotic fluid index. d. Involves an invasive form of ultrasound examination.

ANS: A An abnormal BPP score is an indication that labor should be induced. The BPP evaluates the health of the fetus, requires many different measures, and is a noninvasive procedure.

8. A client asks her nurse, "My doctor told me that he is concerned with the grade of my placenta because I am overdue. What does that mean?" The best response by the nurse is: a. "Your placenta changes as your pregnancy progresses, and it is given a score that indicates the amount of calcium deposits it has. The more calcium deposits, the higher the grade, or number, that is assigned to the placenta. It also means that less blood and oxygen can be delivered to your baby." b. "Your placenta isn't working properly, and your baby is in danger." c. "This means that we will need to perform an amniocentesis to detect if you have any placental damage." d. "Don't worry about it. Everything is fine."

ANS: A An accurate and appropriate response is, "Your placenta changes as your pregnancy progresses, and it is given a score that indicates the amount of calcium deposits it has. The more calcium deposits, the higher the grade, or number, that is assigned to the placenta. It also means that less blood and oxygen can be delivered to your baby." Although "Your placenta isn't working properly, and your baby is in danger" may be valid, it does not reflect therapeutic communication techniques and is likely to alarm the client. An ultrasound, not an amniocentesis, is the method of assessment used to determine placental maturation. The response "Don't worry about it. Everything is fine" is not appropriate and discredits the client's concerns.

2. A 39-year-old primigravida thinks that she is about 8 weeks pregnant, although she has had irregular menstrual periods all her life. She has a history of smoking approximately one pack of cigarettes a day, but she tells you that she is trying to cut down. Her laboratory data are within normal limits. What diagnostic technique could be used with this pregnant woman at this time? a. Ultrasound examination b. Maternal serum alpha-fetoprotein (MSAFP) screening c. Amniocentesis d. Nonstress test (NST)

ANS: A An ultrasound examination could be done to confirm the pregnancy and determine the gestational age of the fetus. It is too early in the pregnancy to perform MSAFP screening, amniocentesis, or NST. MSAFP screening is performed at 16 to 18 weeks of gestation, followed by amniocentesis if MSAFP levels are abnormal or if fetal/maternal anomalies are detected. NST is performed to assess fetal well-being in the third trimester.ANS: A An ultrasound examination could be done to confirm the pregnancy and determine the gestational age of the fetus. It is too early in the pregnancy to perform MSAFP screening, amniocentesis, or NST. MSAFP screening is performed at 16 to 18 weeks of gestation, followed by amniocentesis if MSAFP levels are abnormal or if fetal/maternal anomalies are detected. NST is performed to assess fetal well-being in the third trimester.

4. A woman in preterm labor at 30 weeks of gestation receives two 12-mg doses of betamethasone intramuscularly. The purpose of this pharmacologic treatment is to: a.Stimulate fetal surfactant production. b.Reduce maternal and fetal tachycardia associated with ritodrine administration. c.Suppress uterine contractions. d.Maintain adequate maternal respiratory effort and ventilation during magnesium sulfate therapy.

ANS: A Antenatal glucocorticoids given as intramuscular injections to the mother accelerate fetal lung maturity. Inderal would be given to reduce the effects of ritodrine administration. Betamethasone has no effect on uterine contractions. Calcium gluconate would be given to reverse the respiratory depressive effects of magnesium sulfate therapy.

23. With regard to the process of augmentation of labor, the nurse should be aware that it: a.Is part of the active management of labor that is instituted when the labor process is unsatisfactory. b.Relies on more invasive methods when oxytocin and amniotomy have failed. c.Is a modern management term to cover up the negative connotations of forceps-assisted birth. d.Uses vacuum cups.

ANS: A Augmentation is part of the active management of labor that stimulates uterine contractions after labor has started but is not progressing satisfactorily. Augmentation uses amniotomy and oxytocin infusion, as well as some gentler, noninvasive methods. Forceps-assisted births and vacuum-assisted births are appropriately used at the end of labor and are not part of augmentation.

27. Marfan syndrome is an autosomal dominant genetic disorder that displays as weakness of the connective tissue, joint deformities, ocular dislocation, and weakness to the aortic wall and root. While providing care to a client with Marfan syndrome during labor, which intervention should the nurse complete first? a.Antibiotic prophylaxis b.b-blockers c.Surgery d.Regional anesthesia

ANS: A Because of the potential for cardiac involvement during the third trimester and after birth, treatment with prophylactic antibiotics is highly recommended. b-Blockers and restricted activity are recommended as treatment modalities earlier in the pregnancy. Regional anesthesia is well tolerated by clients with Marfan syndrome; however, it is not essential to care. Adequate labor support may be all that is necessary if an epidural is not part of the woman's birth plan. Surgery for cardiovascular changes such as mitral valve prolapse, aortic regurgitation, root dilation, or dissection may be necessary. Mortality rates may be as high as 50% in women who have severe cardiac disease.

16. Compared with contraction stress test (CST), nonstress test (NST) for antepartum fetal assessment: a. Has no known contraindications. b. Has fewer false-positive results. c. Is more sensitive in detecting fetal compromise. d. Is slightly more expensive.

ANS: A CST has several contraindications. NST has a high rate of false-positive results, is less sensitive than the CST, and is relatively inexpensive.

25. As related to the care of the patient with anemia, the nurse should be aware that: a.It is the most common medical disorder of pregnancy. b.It can trigger reflex brachycardia. c.The most common form of anemia is caused by folate deficiency. d.Thalassemia is a European version of sickle cell anemia.

ANS: A Combined with any other complication, anemia can result in congestive heart failure. Reflex bradycardia is a slowing of the heart in response to the blood flow increases immediately after birth. The most common form of anemia is iron deficiency anemia. Both thalassemia and sickle cell hemoglobinopathy are hereditary but not directly related or confined to geographic areas.

27. Which patient status is an acceptable indication for serial oxytocin induction of labor? a.Past 42 weeks' gestation b.Multiple fetuses c.Polyhydramnios d.History of long labors

ANS: A Continuing a pregnancy past the normal gestational period is likely to be detrimental to fetal health. Multiple fetuses overdistend the uterus and make induction of labor high risk. Polyhydramnios overdistends the uterus, again making induction of labor high risk. History of rapid labors is a reason for induction of labor because of the possibility that the baby would otherwise be born in uncontrolled circumstances.

34. When the pregnant diabetic woman experiences hypoglycemia while hospitalized, the nurse should intervene by having the patient: a.Eat six saltine crackers. b.Drink 8 oz of orange juice with 2 tsp of sugar added. c.Drink 4 oz of orange juice followed by 8 oz of milk. d.Eat hard candy or commercial glucose wafers.

ANS: A Crackers provide carbohydrates in the form of polysaccharides. Orange juice and sugar will increase the blood sugar but not provide a slow-burning carbohydrate to sustain the blood sugar. Milk is a disaccharide and orange juice is a monosaccharide. They will provide an increase in blood sugar but will not sustain the level. Hard candy or commercial glucose wafers provide only monosaccharides.

3. The nurse sees a woman for the first time when she is 30 weeks pregnant. The woman has smoked throughout the pregnancy, and fundal height measurements now are suggestive of growth restriction in the fetus. In addition to ultrasound to measure fetal size, what other tool would be useful in confirming the diagnosis? a. Doppler blood flow analysis c. Amniocentesis b. Contraction stress test (CST) d. Daily fetal movement counts

ANS: A Doppler blood flow analysis allows the examiner to study the blood flow noninvasively in the fetus and the placenta. It is a helpful tool in the management of high risk pregnancies because of intrauterine growth restriction (IUGR), diabetes mellitus, multiple fetuses, or preterm labor. Because of the potential risk of inducing labor and causing fetal distress, CST is not performed on a woman whose fetus is preterm. Indications for amniocentesis include diagnosis of genetic disorders or congenital anomalies, assessment of pulmonary maturity, and diagnosis of fetal hemolytic disease, not IUGR. Fetal kick count monitoring is performed to monitor the fetus in pregnancies complicated by conditions that may affect fetal oxygenation. Although this may be a useful tool at some point later in this woman's pregnancy, it is not used to diagnose IUGR.

1. In assessing the knowledge of a pregestational woman with type 1 diabetes concerning changing insulin needs during pregnancy, the nurse recognizes that further teaching is warranted when the client states: a."I will need to increase my insulin dosage during the first 3 months of pregnancy." b."Insulin dosage will likely need to be increased during the second and third trimesters." c."Episodes of hypoglycemia are more likely to occur during the first 3 months." d."Insulin needs should return to normal within 7 to 10 days after birth if I am bottle-feeding."

ANS: A Insulin needs are reduced in the first trimester because of increased insulin production by the pancreas and increased peripheral sensitivity to insulin. "Insulin dosage will likely need to be increased during the second and third trimesters," "Episodes of hypoglycemia are more likely to occur during the first 3 months," and "Insulin needs should return to normal within 7 to 10 days after birth if I am bottle-feeding" are accurate statements and signify that the woman has understood the teachings regarding control of her diabetes during pregnancy.

22. Which analysis of maternal serum may predict chromosomal abnormalities in the fetus? a. Multiple-marker screening b. Lecithin/sphingomyelin (L/S) ratio c. Biophysical profile d. Type and crossmatch of maternal and fetal serum

ANS: A Maternal serum can be analyzed for abnormal levels of alpha-fetoprotein, human chorionic gonadotropin, and estriol. The multiple-marker screening may predict chromosomal defects in the fetus. The L/S ratio is used to determine fetal lung maturity. A biophysical profile is used for evaluating fetal status during the antepartum period. Five variables are used, but none is concerned with chromosomal problems. The blood type and crossmatch would not predict chromosomal defects in the fetus.

28. With one exception, the safest pregnancy is one in which the woman is drug and alcohol free. For women addicted to opioids, ________________________ treatment is the current standard of care during pregnancy. a.Methadone maintenance b.Detoxification c.Smoking cessation d.4 Ps Plus

ANS: A Methadone maintenance treatment (MMT) is currently considered the standard of care for pregnant women who are dependent on heroin or other narcotics. Buprenorphine is another medication approved for opioid addiction treatment that is increasingly being used during pregnancy. Opioid replacement therapy has been shown to decrease opioid and other drug use, reduce criminal activity, improve individual functioning, and decrease rates of infections such as hepatitis B and C, HIV, and other sexually transmitted infections. Detoxification is the treatment used for alcohol addiction. Pregnant women requiring withdrawal from alcohol should be admitted for inpatient management. Women are more likely to stop smoking during pregnancy than at any other time in their lives. A smoking cessation program can assist in achieving this goal. The 4 Ps Plus is a screening tool designed specifically to identify pregnant women who need in-depth assessment related to substance abuse.

5. Screening at 24 weeks of gestation reveals that a pregnant woman has gestational diabetes mellitus (GDM). In planning her care, the nurse and the woman mutually agree that an expected outcome is to prevent injury to the fetus as a result of GDM. The nurse identifies that the fetus is at greatest risk for: a.Macrosomia. b.Congenital anomalies of the central nervous system. c.Preterm birth. d.Low birth weight.

ANS: A Poor glycemic control later in pregnancy increases the rate of fetal macrosomia. Poor glycemic control during the preconception time frame and into the early weeks of the pregnancy is associated with congenital anomalies. Preterm labor or birth is more likely to occur with severe diabetes and is the greatest risk in women with pregestational diabetes. Increased weight, or macrosomia, is the greatest risk factor for this woman.

10. With regard to the association of maternal diabetes and other risk situations affecting mother and fetus, nurses should be aware that: a.Diabetic ketoacidosis (DKA) can lead to fetal death at any time during pregnancy. b.Hydramnios occurs approximately twice as often in diabetic pregnancies. c.Infections occur about as often and are considered about as serious in diabetic and nondiabetic pregnancies. d.Even mild to moderate hypoglycemic episodes can have significant effects on fetal well-being.

ANS: A Prompt treatment of DKA is necessary to save the fetus and the mother. Hydramnios occurs 10 times more often in diabetic pregnancies. Infections are more common and more serious in pregnant women with diabetes. Mild to moderate hypoglycemic episodes do not appear to have significant effects on fetal well-being.

17. Prophylaxis of subacute bacterial endocarditis is given before and after birth when a pregnant woman has: a.Valvular disease. b.Congestive heart disease c.Arrhythmias. d.Postmyocardial infarction.

ANS: A Prophylaxis for intrapartum endocarditis and pulmonary infection may be provided for women who have mitral valve stenosis. Prophylaxis for intrapartum endocarditis is not indicated for congestive heart disease, arrhythmias, or after myocardial infarction.

9. In evaluating the effectiveness of oxytocin induction, the nurse would expect: a.Contractions lasting 40 to 90 seconds, 2 to 3 minutes apart. b.The intensity of contractions to be at least 110 to 130 mm Hg. c.Labor to progress at least 2 cm/hr dilation. d.At least 30 mU/min of oxytocin will be needed to achieve cervical dilation.

ANS: A The goal of induction of labor would be to produce contractions that occur every 2 to 3 minutes and last 60 to 90 seconds. The intensity of the contractions should be 40 to 90 mm Hg by intrauterine pressure catheter. Cervical dilation of 1 cm/hr in the active phase of labor would be the goal in an oxytocin induction. The dose is increased by 1 to 2 mU/min at intervals of 30 to 60 minutes until the desired contraction pattern is achieved. Doses are increased up to a maximum of 20 to 40 mU/min.

34. Immediately after the forceps-assisted birth of an infant, the nurse should: a.Assess the infant for signs of trauma. b.Give the infant prophylactic antibiotics. c.Apply a cold pack to the infant's scalp. d.Measure the circumference of the infant's head.

ANS: A The infant should be assessed for bruising or abrasions at the site of application, facial palsy, and subdural hematoma. Prophylactic antibiotics are not necessary with a forceps delivery. A cold pack would put the infant at risk for cold stress and is contraindicated. Measuring the circumference of the head is part of the initial nursing assessment.

26. The exact cause of preterm labor is unknown and believed to be multifactorial. Infection is thought to be a major factor in many preterm labors. Select the type of infection that has not been linked to preterm births. a.Viral b.Periodontal c.Cervical d.Urinary tract

ANS: A The infections that increase the risk of preterm labor and birth are all bacterial. They include cervical, urinary tract, periodontal, and other bacterial infections. Therefore, it is important for the client to participate in early, continual, and comprehensive prenatal care. Evidence has shown a link between periodontal infections and preterm labor. Researchers recommend regular dental care before and during pregnancy, oral assessment as a routine part of prenatal care, and scrupulous oral hygiene to prevent infection. Cervical infections of a bacterial nature have been linked to preterm labor and birth. The presence of urinary tract infections increases the risk of preterm labor and birth.

30. Which major neonatal complication is carefully monitored after the birth of the infant of a diabetic mother? a.Hypoglycemia b.Hypercalcemia c.Hypobilirubinemia d.Hypoinsulinemia

ANS: A The neonate is at highest risk for hypoglycemia because fetal insulin production is accelerated during pregnancy to metabolize excessive glucose from the mother. At birth, the maternal glucose supply stops and the neonatal insulin exceeds the available glucose, thus leading to hypoglycemia. Hypocalcemia is associated with preterm birth, birth trauma, and asphyxia, all common problems of the infant of a diabetic mother. Excess erythrocytes are broken down after birth and release large amounts of bilirubin into the neonate's circulation, with resulting hyperbilirubinemia. Because fetal insulin production is accelerated during pregnancy, the neonate presents with hyperinsulinemia.

21. A pregnant woman's biophysical profile score is 8. She asks the nurse to explain the results. The nurse's best response is: a. "The test results are within normal limits." b. "Immediate delivery by cesarean birth is being considered." c. "Further testing will be performed to determine the meaning of this score." d. "An obstetric specialist will evaluate the results of this profile and, within the next week, will inform you of your options regarding delivery."

ANS: A The normal biophysical score ranges from 8 to 10 points if the amniotic fluid volume is adequate. A normal score allows conservative treatment of high-risk patients. Delivery can be delayed if fetal well-being is indicated. Scores less than 4 should be investigated, and delivery could be initiated sooner than planned. This score is within normal range, and no further testing is required at this time. The results of the biophysical profile are usually available immediately after the procedure is performed.

12. A pregnant woman's amniotic membranes rupture. Prolapsed umbilical cord is suspected. What intervention would be the top priority? a.Placing the woman in the knee-chest position b.Covering the cord in sterile gauze soaked in saline c.Preparing the woman for a cesarean birth d.Starting oxygen by face mask

ANS: A The woman is assisted into a position (e.g., modified Sims position, Trendelenburg position, or the knee-chest position) in which gravity keeps the pressure of the presenting part off the cord. Although covering the cord in sterile gauze soaked saline, preparing the woman for a cesarean, and starting oxygen by face mark are appropriate nursing interventions in the event of a prolapsed cord, the intervention of top priority would be positioning the mother to relieve cord compression.

Nursing care measures are commonly offered to women in labor. Which nursing measure reflects application of the gate-control theory? a. Massaging the woman's back b. Changing the woman's position c. Giving the prescribed medication d. Encouraging the woman to rest between contractions

ANS: A According to the gate-control theory, pain sensations travel along sensory nerve pathways to the brain, but only a limited number of sensations, or messages, can travel through these nerve pathways at one time. Distraction techniques such as massage or stroking, music, focal points, and imagery reduce or completely block the capacity of nerve pathways to transmit pain. These distractions are thought to work by closing down a hypothetic gate in the spinal cord and thus preventing pain signals from reaching the brain. The perception of pain is thereby diminished. Changing the woman's position, giving prescribed medication, and encouraging rest do not reduce or block the capacity of nerve pathways to transmit pain using the gate-control theory.

The nurse providing newborn stabilization must be aware that the primary side effect of maternal narcotic analgesia in the newborn is: a. Respiratory depression. b. Bradycardia. c. Acrocyanosis. d. Tachypnea.

ANS: A An infant delivered within 1 to 4 hours of maternal analgesic administration is at risk for respiratory depression from the sedative effects of the narcotic. Bradycardia is not the anticipated side effect of maternal analgesics. Acrocyanosis is an expected finding in a newborn and is not related to maternal analgesics. The infant who is having a side effect to maternal analgesics normally would have a decrease in respirations, not an increase.

Which occurrence is associated with cervical dilation and effacement? a. Bloody show b. False labor c. Lightening d. Bladder distention

ANS: A As the cervix begins to soften, dilate, and efface, expulsion of the mucous plug that sealed the cervix during pregnancy occurs. This causes rupture of small cervical capillaries. Cervical dilation and effacement do not occur with false labor. Lightening is the descent of the fetus toward the pelvic inlet before labor. Bladder distention occurs when the bladder is not emptied frequently. It may slow down the descent of the fetus during labor.

With regard to nerve block analgesia and anesthesia, nurses should be aware that: a. Most local agents are related chemically to cocaine and end in the suffix -caine. b. Local perineal infiltration anesthesia is effective when epinephrine is added, but it can be injected only once. c. A pudendal nerve block is designed to relieve the pain from uterine contractions. d. A pudendal nerve block, if done correctly, does not significantly lessen the bearing-down reflex.

ANS: A Common agents include lidocaine and chloroprocaine. Injections can be repeated to prolong the anesthesia. A pudendal nerve block relieves pain in the vagina, vulva, and perineum but not the pain from uterine contractions, and it lessens or shuts down the bearing-down reflex.

A woman is experiencing back labor and complains of intense pain in her lower back. An effective relief measure would be to use: a. Counterpressure against the sacrum. b. Pant-blow (breaths and puffs) breathing techniques. c. Effleurage. d. Conscious relaxation or guided imagery.

ANS: A Counterpressure is steady pressure applied by a support person to the sacral area with the fist or heel of the hand. This technique helps the woman cope with the sensations of internal pressure and pain in the lower back. The pain management techniques of pant-blow, effleurage, and conscious relaxation or guided imagery are usually helpful for contractions per the gate-control theory.

The nurse would expect which maternal cardiovascular finding during labor? a. Increased cardiac output b. Decreased pulse rate c. Decreased white blood cell (WBC) count d. Decreased blood pressure

ANS: A During each contraction, 400 mL of blood is emptied from the uterus into the maternal vascular system. This increases cardiac output by about 51% above baseline pregnancy values at term. The heart rate increases slightly during labor. The WBC count can increase during labor. During the first stage of labor, uterine contractions cause systolic readings to increase by about 10 mm Hg. During the second stage, contractions may cause systolic pressures to increase by 30 mm Hg and diastolic readings to increase by 25 mm Hg.

With regard to breathing techniques during labor, maternity nurses should understand that: a. Breathing techniques in the first stage of labor are designed to increase the size of the abdominal cavity to reduce friction. b. By the time labor has begun, it is too late for instruction in breathing and relaxation. c. Controlled breathing techniques are most difficult near the end of the second stage of labor. d. The patterned-paced breathing technique can help prevent hyperventilation.

ANS: A First-stage techniques promote relaxation of abdominal muscles, thereby increasing the size of the abdominal cavity. Instruction in simple breathing and relaxation techniques early in labor is possible and effective. Controlled breathing techniques are most difficult in the transition phase at the end of the first stage of labor when the cervix is dilated 8 to 10 cm. Patterned-paced breathing sometimes can lead to hyperventilation.

Which description of the four stages of labor is correct for both definition and duration? a. First stage: onset of regular uterine contractions to full dilation; less than 1 hour to 20 hours b. Second stage: full effacement to 4 to 5 cm; visible presenting part; 1 to 2 hours c. Third state: active pushing to birth; 20 minutes (multiparous women), 50 minutes (first-timer) d. Fourth stage: delivery of the placenta to recovery; 30 minutes to 1 hour

ANS: A Full dilation may occur in less than 1 hour, but in first-time pregnancies it can take up to 20 hours. The second stage extends from full dilation to birth and takes an average of 20 to 50 minutes, although 2 hours is still considered normal. The third stage extends from birth to expulsion of the placenta and usually takes a few minutes. The fourth stage begins after expulsion of the placenta and lasts until homeostasis is reestablished (about 2 hours).

Which presentation is described accurately in terms of both presenting part and frequency of occurrence? a. Cephalic: occiput; at least 95% b. Breech: sacrum; 10% to 15% c. Shoulder: scapula; 10% to 15% d. Cephalic: cranial; 80% to 85%

ANS: A In cephalic presentations (head first), the presenting part is the occiput; this occurs in 96% of births. In a breech birth, the sacrum emerges first; this occurs in about 3% of births. In shoulder presentations, the scapula emerges first; this occurs in only 1% of births.

A woman in active labor receives an analgesic opioid agonist. Which medication relieves severe, persistent, or recurrent pain; creates a sense of well-being; overcomes inhibitory factors; and may even relax the cervix but should be used cautiously in women with cardiac disease? a. Meperidine (Demerol) b. Promethazine (Phenergan) c. Butorphanol tartrate (Stadol) d. Nalbuphine (Nubain)

ANS: A Meperidine is the most commonly used opioid agonist analgesic for women in labor throughout the world. It overcomes inhibitory factors in labor and may even relax the cervix. Because tachycardia is a possible adverse reaction, meperidine is used cautiously in women with cardiac disease. Phenergan is an ataractic (tranquilizer) that may be used to augment the desirable effects of the opioid analgesics but has few of the undesirable effects of those drugs. Stadol and Nubain are opioid agonist-antagonist analgesics.

Nurses should be aware of the differences experience can make in labor pain such as: a. Sensory pain for nulliparous women often is greater than for multiparous women during early labor. b. Affective pain for nulliparous women usually is less than for multiparous women throughout the first stage of labor. c. Women with a history of substance abuse experience more pain during labor. d. Multiparous women have more fatigue from labor and therefore experience more pain.

ANS: A Sensory pain is greater for nulliparous women because their reproductive tract structures are less supple. Affective pain is greater for nulliparous women during the first stage but decreases for both nulliparous and multiparous during the second stage. Women with a history of substance abuse experience the same amount of pain as those without such a history. Nulliparous women have longer labors and therefore experience more fatigue.

In assessing a woman for pain and discomfort management during labor, a nurse most likely would: a. Have the woman use a visual analog scale (VAS) to determine her level of pain. b. Note drowsiness as a sign that the medications were working. c. Interpret a woman's fist clenching as an indication that she is angry at her male partner and the physician. d. Evaluate the woman's skin turgor to see whether she needs a gentle oil massage.

ANS: A The VAS is a means of adding the woman's assessment of her pain to the nurse's observations. Drowsiness is a side effect of medications, not usually (sedatives aside) a sign of effectiveness. The fist clenching likely is a sign of apprehension that may need attention. Skin turgor, along with the moistness of the membranes and the concentration of the urine, is a sign that helps the nurse evaluate hydration.

To care for a laboring woman adequately, the nurse understands that the __________ stage of labor varies the most in length? a. First b. Second c. Third d. Fourth

ANS: A The first stage of labor is considered to last from the onset of regular uterine contractions to full dilation of the cervix. The first stage is much longer than the second and third stages combined. In a first-time pregnancy the first stage of labor can take up to 20 hours. The second stage of labor lasts from the time the cervix is fully dilated to the birth of the fetus. The average length is 20 minutes for a multiparous woman and 50 minutes for a nulliparous woman. The third stage of labor lasts from the birth of the fetus until the placenta is delivered. This stage may be as short as 3 minutes or as long as 1 hour. The fourth stage of labor, recovery, lasts about 2 hours after delivery of the placenta.

In relation to primary and secondary powers, the maternity nurse comprehends that: a. Primary powers are responsible for effacement and dilation of the cervix. b. Effacement generally is well ahead of dilation in women giving birth for the first time; they are closer together in subsequent pregnancies. c. Scarring of the cervix caused by a previous infection or surgery may make the delivery a bit more painful, but it should not slow or inhibit dilation. d. Pushing in the second stage of labor is more effective if the woman can breathe deeply and control some of her involuntary needs to push, as the nurse directs.

ANS: A The primary powers are responsible for dilation and effacement; secondary powers are concerned with expulsion of the fetus. Effacement generally is well ahead of dilation in first-timers; they are closer together in subsequent pregnancies. Scarring of the cervix may slow dilation. Pushing is more effective and less fatiguing when the woman begins to push only after she has the urge to do so.

To help clients manage discomfort and pain during labor, nurses should be aware that: a. The predominant pain of the first stage of labor is the visceral pain located in the lower portion of the abdomen. b. Referred pain is the extreme discomfort between contractions. c. The somatic pain of the second stage of labor is more generalized and related to fatigue. d. Pain during the third stage is a somewhat milder version of the second stage.

ANS: A This pain comes from cervical changes, distention of the lower uterine segment, and uterine ischemia. Referred pain occurs when the pain that originates in the uterus radiates to the abdominal wall, lumbosacral area of the back, iliac crests, and gluteal area. Second-stage labor pain is intense, sharp, burning, and localized. Third-stage labor pain is similar to that of the first stage.

39. Congenital anomalies can occur with the use of antiepileptic drugs (AEDs), including (Select all that apply): a.Cleft lip. b.Congenital heart disease. c.Neural tube defects. d.Gastroschisis. e.Diaphragmatic hernia.

ANS: A, B, C Congenital anomalies that can occur with AEDs include cleft lip or palate, congenital heart disease, urogenital defects, and neural tube defects. Gastroschisis and diaphragmatic hernia are not associated with the use of AEDs.

26. Intrauterine growth restriction (IUGR) is associated with numerous pregnancy-related risk factors (Select all that apply). a. Poor nutrition b. Maternal collagen disease c. Gestational hypertension d. Premature rupture of membranes e. Smoking

ANS: A, B, C, E Poor nutrition, maternal collagen disease, gestational hypertension, and smoking all are risk factors associated with IUGR. Premature rupture of membranes is associated with preterm labor, not IUGR.

27. Transvaginal ultrasonography is often performed during the first trimester. While preparing your 6-week gestation patient for this procedure, she expresses concerns over the necessity for this test. The nurse should explain that this diagnostic test may be indicated for a number of situations (Select all that apply). a. Multifetal gestation b. Obesity c. Fetal abnormalities d. Amniotic fluid volume e. Ectopic pregnancy

ANS: A, B, C, E Transvaginal ultrasound is useful in obese women whose thick abdominal layers cannot be penetrated with traditional abdominal ultrasound. This procedure is also used for identifying multifetal gestation, ectopic pregnancy, estimating gestational age, confirming fetal viability, and identifying fetal abnormalities. Amniotic fluid volume is assessed during the second and third trimester. Conventional ultrasound would be used.

While developing an intrapartum care plan for the patient in early labor, it is important that the nurse recognize that psychosocial factors may influence a woman's experience of pain. These include (Select all that apply): a. Culture. b. Anxiety and fear. c. Previous experiences with pain. d. Intervention of caregivers. e. Support systems.

ANS: A, B, C, E Culture: a woman's sociocultural roots influence how she perceives, interprets, and responds to pain during childbirth. Some cultures encourage loud and vigorous expressions of pain, whereas others value self-control. The nurse should avoid praising some behaviors (stoicism) while belittling others (noisy expression). Anxiety and fear: extreme anxiety and fear magnify sensitivity to pain and impair a woman's ability to tolerate it. Anxiety and fear increase muscle tension in the pelvic area, which counters the expulsive forces of uterine contractions and pushing efforts. Previous experiences with pain: fear and withdrawal are a natural response to pain during labor. Learning about these normal sensations ahead of time helps a woman suppress her natural reactions of fear regarding the impending birth. If a woman previously had a long and difficult labor, she is likely to be anxious. She may also have learned ways to cope and may use these skills to adapt to the present labor experience. Support systems: an anxious partner is less able to provide help and support to a woman during labor. A woman's family and friends can be an important source of support if they convey realistic and positive information about labor and delivery. Although the intervention of caregivers may be necessary for the well-being of the woman and her fetus, some interventions add discomfort to the natural pain of labor (i.e., fetal monitor straps, intravenous lines).

40. Diabetes refers to a group of metabolic diseases characterized by hyperglycemia resulting from defects in insulin action, insulin secretion, or both. Over time, diabetes causes significant changes in the microvascular and macrovascular circulations. These complications include: a.Atherosclerosis. b.Retinopathy. c.IUFD. d.Nephropathy. e.Neuropathy. Autonomcs neuropathy.

ANS: A, B, D, E These structural changes are most likely to affect a variety of systems, including the heart, eyes, kidneys, and nerves. Intrauterine fetal death (stillbirth) remains a major complication of diabetes in pregnancy; however, this is a fetal complication.

Which factors influence cervical dilation (Select all that apply) ? a. Strong uterine contractions b. The force of the presenting fetal part against the cervix c. The size of the female pelvis d. The pressure applied by the amniotic sac e. Scarring of the cervix

ANS: A, B, D, E Dilation of the cervix occurs by the drawing upward of the musculofibrous components of the cervix, which is caused by strong uterine contractions. Pressure exerted by the amniotic fluid while the membranes are intact or by the force applied by the presenting part also can promote cervical dilation. Scarring of the cervix as a result of a previous infection or surgery may slow cervical dilation. Pelvic size does not affect cervical dilation.

Signs that precede labor include (Select all that apply): a. Lightening. b. Exhaustion. c. Bloody show. d. Rupture of membranes. e. Decreased fetal movement.

ANS: A, C, D Signs that precede labor may include lightening, urinary frequency, backache, weight loss, surge of energy, bloody show, and rupture of membranes. Many women experience a burst of energy before labor. A decrease in fetal movement is an ominous sign that does not always correlate with labor.

38. Induction of labor is considered an acceptable obstetric procedure if it is in the best interest to deliver the fetus. The charge nurse on the labor and delivery unit is often asked to schedule patients for this procedure and therefore must be cognizant of the specific conditions appropriate for labor induction. These include (Select all that apply): a.Rupture of membranes at or near term. b.Convenience of the woman or her physician. c.Chorioamnionitis (inflammation of the amniotic sac). d.Post-term pregnancy. e.Fetal death.

ANS: A, C, D, E These are all acceptable indications for induction. Other conditions include intrauterine growth retardation (IUGR), maternal-fetal blood incompatibility, hypertension, and placental abruption. Elective inductions for the convenience of the woman or her provider are not recommended; however, they have become commonplace. Factors such as rapid labors and living a long distance from a health care facility may be valid reasons in such a circumstance. Elective delivery should not occur before 39 weeks' completed gestation.

The class of drugs known as opioid analgesics (butorphanol, nalbuphine) is not suitable for administration to women with known opioid dependence. The antagonistic activity could precipitate withdrawal symptoms (abstinence syndrome) in both mothers and newborns. Signs of opioid/narcotic withdrawal in the mother would include (Select all that apply): a. Yawning, runny nose. b. Increase in appetite. c. Chills and hot flashes. d. Constipation. e. Irritability, restlessness.

ANS: A, C, E The woman experiencing maternal opioid withdrawal syndrome will exhibit yawning, runny nose, sneezing, anorexia, chills or hot flashes, vomiting, diarrhea, abdominal pain, irritability, restlessness, muscle spasms, weakness, and drowsiness. It is important for the nurse to assess both mother and baby and to plan care accordingly.

45. Step 3 a.Without adding air, withdraw the correct dose of NPH insulin. b.Gently rotate the insulin to mix it, and wipe the stopper. c.Inject air equal to the dose of NPH insulin into the vial, and remove the syringe. d.Inject air equal to the dose of regular insulin into the vial, and withdraw the medication. e.Check the insulin bottles for the expiration date. f.Wash hands.

ANS: B

10. In planning for an expected cesarean birth for a woman who has given birth by cesarean previously and who has a fetus in the transverse presentation, which information would the nurse include? a."Because this is a repeat procedure, you are at the lowest risk for complications." b."Even though this is your second cesarean birth, you may wish to review the preoperative and postoperative procedures." c."Because this is your second cesarean birth, you will recover faster." d."You will not need preoperative teaching because this is your second cesarean birth."

ANS: B "Even though this is your second cesarean birth, you may wish to review the preoperative and postoperative procedures" is the most appropriate statement. It is not accurate to state that the woman is at the lowest risk for complications. Both maternal and fetal risks are associated with every cesarean section. "Because this is your second cesarean birth, you will recover faster" is not an accurate statement. Physiologic and psychologic recovery from a cesarean section is multifactorial and individual to each client each time. Preoperative teaching should always be performed, regardless of whether the client has already had this procedure.

31. A maternal indication for the use of vacuum extraction is: a.A wide pelvic outlet. b.Maternal exhaustion c.A history of rapid deliveries. d.Failure to progress past 0 station.

ANS: B A mother who is exhausted may be unable to assist with the expulsion of the fetus. The patient with a wide pelvic outlet will likely not require vacuum extraction. With a rapid delivery, vacuum extraction is not necessary. A station of 0 is too high for a vacuum extraction.

38. A woman has a history of drug use and is screened for hepatitis B during the first trimester. What is an appropriate action? a.Provide a low-protein diet. b.Offer the vaccine. c.Discuss the recommendation to bottle-feed her baby. d.Practice respiratory isolation.

ANS: B A person who has a history of high risk behaviors should be offered the hepatitis B vaccine. Care is supportive and includes bed rest and a high-protein, low-fat diet. The first trimester is too early to discuss feeding methods with a woman in the high risk category. Hepatitis B is transmitted through blood.

4. Concerning the use and abuse of legal drugs or substances, nurses should be aware that: a.Although cigarette smoking causes a number of health problems, it has little direct effect on maternity-related health. b.Caucasian women are more likely to experience alcohol-related problems. c.Coffee is a stimulant that can interrupt body functions and has been related to birth defects. d.Prescription psychotherapeutic drugs taken by the mother do not affect the fetus; otherwise, they would not have been prescribed.

ANS: B African-American and poor women are more likely to use illicit substances, particularly cocaine, whereas Caucasian and educated women are more likely to use alcohol.

22. While caring for the patient who requires an induction of labor, the nurse should be cognizant that: a.Ripening the cervix usually results in a decreased success rate for induction. b.Labor sometimes can be induced with balloon catheters or laminaria tents. c.Oxytocin is less expensive than prostaglandins and more effective but creates greater health risks. d.Amniotomy can be used to make the cervix more favorable for labor.

ANS: B Balloon catheters or laminaria tents are mechanical means of ripening the cervix. Ripening the cervix, making it softer and thinner, increases the success rate of induced labor. Prostaglandin E1 is less expensive and more effective than oxytocin but carries a greater risk. Amniotomy is the artificial rupture of membranes, which is used to induce labor only when the cervix is already ripe.

18. While providing care in an obstetric setting, the nurse should understand that postpartum care of the woman with cardiac disease: a.Is the same as that for any pregnant woman. b.Includes rest, stool softeners, and monitoring of the effect of activity. c.Includes ambulating frequently, alternating with active range of motion. d.Includes limiting visits with the infant to once per day.

ANS: B Bed rest may be ordered, with or without bathroom privileges. Bowel movements without stress or strain for the woman are promoted with stool softeners, diet, and fluid. Care of the woman with cardiac disease in the postpartum period is tailored to the woman's functional capacity. The woman will be on bed rest to conserve energy and reduce the strain on the heart. Although the woman may need help caring for the infant, breastfeeding and infant visits are not contraindicated.

16. Nurses should know some basic definitions concerning preterm birth, preterm labor, and low birth weight. For instance: a.The terms preterm birth and low birth weight can be used interchangeably. b.Preterm labor is defined as cervical changes and uterine contractions occurring between 20 and 37 weeks of pregnancy. c.Low birth weight is anything below 3.7 pounds. d.In the United States early in this century, preterm birth accounted for 18% to 20% of all births.

ANS: B Before 20 weeks, it is not viable (miscarriage); after 37 weeks, it can be considered term. Although these terms are used interchangeably, they have different meanings: preterm birth describes the length of gestation (37 weeks) regardless of weight; low birth weight describes weight only (2500 g or less) at the time of birth, whenever it occurs. Low birth weight is anything less than 2500 g, or about 5.5 pounds. In 2003 the preterm birth rate in the United States was 12.3%, but it is increasing in frequency.

15. An 18-year-old client who has reached 16 weeks of gestation was recently diagnosed with pregestational diabetes. She attends her centering appointment accompanied by one of her girlfriends. This young woman appears more concerned about how her pregnancy will affect her social life than about her recent diagnosis of diabetes. Several nursing diagnoses are applicable to assist in planning adequate care. The most appropriate diagnosis at this time is: a.Risk for injury to the fetus related to birth trauma. b.Noncompliance related to lack of understanding of diabetes and pregnancy and requirements of the treatment plan. c.Deficient knowledge related to insulin administration. d.Risk for injury to the mother related to hypoglycemia or hyperglycemia.

ANS: B Before a treatment plan is developed or goals for the outcome of care are outlined, this client must come to an understanding of diabetes and the potential effects on her pregnancy. She appears to have greater concern for changes to her social life than adoption of a new self-care regimen. Risk for injury to the fetus related to either placental insufficiency or birth trauma may come much later in the pregnancy. At this time the client is having difficulty acknowledging the adjustments that she needs to make to her lifestyle to care for herself during pregnancy. The client may not yet be on insulin. Insulin requirements increase with gestation. The importance of glycemic control must be part of health teaching for this client. However, she has not yet acknowledged that changes to her lifestyle need to be made, and she may not participate in the plan of care until understanding takes place.

7. Maternal phenylketonuria (PKU) is an important health concern during pregnancy because: a.It is a recognized cause of preterm labor. b.The fetus may develop neurologic problems. c.A pregnant woman is more likely to die without dietary control. d.Women with PKU are usually retarded and should not reproduce.

ANS: B Children born to women with untreated PKU are more likely to be born with mental retardation, microcephaly, congenital heart disease, and low birth weight. Maternal PKU has no effect on labor. Women without dietary control of PKU are more likely to miscarry or bear a child with congenital anomalies. Screening for undiagnosed maternal PKU at the first prenatal visit may be warranted, especially in individuals with a family history of the disorder, with low intelligence of uncertain etiology, or who have given birth to microcephalic infants.

24. Nurses caring for antepartum women with cardiac conditions should be aware that: a.Stress on the heart is greatest in the first trimester and the last 2 weeks before labor. b.Women with class II cardiac disease should avoid heavy exertion and any activity that causes even minor symptoms. c.Women with class III cardiac disease should have 8 to 10 hours of sleep every day and limit housework, shopping, and exercise. d.Women with class I cardiac disease need bed rest through most of the pregnancy and face the possibility of hospitalization near term.

ANS: B Class II cardiac disease is symptomatic with ordinary activity. Women in this category need to avoid heavy exertion and limit regular activities as symptoms dictate. Stress is greatest between weeks 28 and 32, when homodynamic changes reach their maximum. Class III cardiac disease is symptomatic with less than ordinary activity. These women need bed rest most of the day and face the possibility of hospitalization near term. Class I cardiac disease is asymptomatic at normal levels of activity. These women can carry on limited normal activities with discretion, although they still need a good amount of sleep.

11. Diabetes in pregnancy puts the fetus at risk in several ways. Nurses should be aware that: a.With good control of maternal glucose levels, sudden and unexplained stillbirth is no longer a major concern. b.The most important cause of perinatal loss in diabetic pregnancy is congenital malformations. c.Infants of mothers with diabetes have the same risks for respiratory distress syndrome because of the careful monitoring. d.At birth the neonate of a diabetic mother is no longer in any risk.

ANS: B Congenital malformations account for 30% to 50% of perinatal deaths. Even with good control, sudden and unexplained stillbirth remains a major concern. Infants of diabetic mothers are at increased risk for respiratory distress syndrome. The transition to extrauterine life often is marked by hypoglycemia and other metabolic abnormalities.

12. In the first trimester, ultrasonography can be used to gain information on: a. Amniotic fluid volume. b. Location of Gestational sacs c. Placental location and maturity. d. Cervical length.

ANS: B During the first trimester, ultrasound examination is performed to obtain information regarding the number, size, and location of gestatials sacs; the presence or absence of fetal cardiac and body movements; the presences or absence of uterine abnormalities (e.g., bicornuate uterus or fibroids) or adnexal masses (e.g., ovarian cysts or an ectopic pregnancy); and pregnancy dating.

7. Maternal serum alpha-fetoprotein (MSAFP) screening indicates an elevated level. MSAFP screening is repeated and again is reported as higher than normal. What would be the next step in the assessment sequence to determine the well-being of the fetus? a. Percutaneous umbilical blood sampling (PUBS) b. Ultrasound for fetal anomalies c. Biophysical profile (BPP) for fetal well-being d. Amniocentesis for genetic anomalies

ANS: B If MSAFP findings are abnormal, follow-up procedures include genetic counseling for families with a history of neural tube defect, repeated MSAFP screening, ultrasound examination, and possibly amniocentesis. Indications for use of PUBS include prenatal diagnosis of inherited blood disorders, karyotyping of malformed fetuses, detection of fetal infection, determination of the acid-base status of fetuses with intrauterine growth restriction, and assessment and treatment of isoimmunization and thrombocytopenia in the fetus. BPP is a method of assessing fetal well-being in the third trimester. Before amniocentesis is considered, the client first would have an ultrasound for direct visualization of the fetus.

20. The use of methamphetamine (meth) has been described as a significant drug problem in the United States. In order to provide adequate nursing care to this client population the nurse must be cognizant that methamphetamine: a.Is similar to opiates. b.Is a stimulant with vasoconstrictive characteristics. c.Should not be discontinued during pregnancy. d.Is associated with a low rate of relapse.

ANS: B Methamphetamines are stimulants with vasoconstrictive characteristics similar to cocaine and are used similarly. As is the case with cocaine users, methamphetamine users are urged to immediately stop all use during pregnancy. Unfortunately, because methamphetamine users are extremely psychologically addicted, the rate of relapse is very high.

14. A new mother with which of these thyroid disorders would be strongly discouraged from breastfeeding? a.Hyperthyroidism b.Phenylketonuria (PKU) c.Hypothyroidism d.Thyroid storm

ANS: B PKU is a cause of mental retardation in infants; mothers with PKU pass on phenylalanine. A woman with hyperthyroidism or hypothyroidism would have no particular reason not to breastfeed. A thyroid storm is a complication of hyperthyroidism

20. Risk factors tend to be interrelated and cumulative in their effect. While planning the care for a laboring client with diabetes mellitus, the nurse is aware that she is at a greater risk for: a. Oligohydramnios. c. Postterm pregnancy. b. Polyhydramnios. d. Chromosomal abnormalities.

ANS: B Polyhydramnios (amniotic fluid >2000 mL) is 10 times more likely to occur in diabetic compared with nondiabetic pregnancies. Polyhydramnios puts the mother at risk for premature rupture of membranes, premature labor, and postpartum hemorrhage. Prolonged rupture of membranes, intrauterine growth restriction, intrauterine fetal death, and renal agenesis (Potter syndrome) all put the client at risk for developing oligohydramnios. Anencephaly, placental insufficiency, and perinatal hypoxia all contribute to the risk for postterm pregnancy. Maternal age older than 35 and balanced translocation (maternal and paternal) are risk factors for chromosome abnormalities.

19. With regard to dysfunctional labor, nurses should be aware that: a.Women who are underweight are more at risk. b.Women experiencing precipitous labor are about the only "dysfunctionals" not to be exhausted. c.Hypertonic uterine dysfunction is more common than hypotonic dysfunction. d.Abnormal labor patterns are most common in older women.

ANS: B Precipitous labor lasts less than 3 hours. Short women more than 30 pounds overweight are more at risk for dysfunctional labor. Hypotonic uterine dysfunction, in which the contractions become weaker, is more common. Abnormal labor patterns are more common in women less than 20 years of age.

2. Preconception counseling is critical to the outcome of diabetic pregnancies because poor glycemic control before and during early pregnancy is associated with: a.Frequent episodes of maternal hypoglycemia. b.Congenital anomalies in the fetus. c.Polyhydramnios. d.Hyperemesis gravidarum.

ANS: B Preconception counseling is particularly important because strict metabolic control before conception and in the early weeks of gestation is instrumental in decreasing the risks of congenital anomalies. Frequent episodes of maternal hypoglycemia may occur during the first trimester (not before conception) as a result of hormone changes and the effects on insulin production and usage. Hydramnios occurs about 10 times more often in diabetic pregnancies than in nondiabetic pregnancies. Typically it is seen in the third trimester of pregnancy. Hyperemesis gravidarum may exacerbate hypoglycemic events because the decreased food intake by the mother and glucose transfer to the fetus contribute to hypoglycemia.

4. A 41-week pregnant multigravida presents in the labor and delivery unit after a nonstress test indicated that her fetus could be experiencing some difficulties in utero. Which diagnostic tool would yield more detailed information about the fetus? a. Ultrasound for fetal anomalies b. Biophysical profile (BPP) c. Maternal serum alpha-fetoprotein (MSAFP) screening d. Percutaneous umbilical blood sampling (PUBS)

ANS: B Real-time ultrasound permits detailed assessment of the physical and physiologic characteristics of the developing fetus and cataloging of normal and abnormal biophysical responses to stimuli. BPP is a noninvasive, dynamic assessment of a fetus that is based on acute and chronic markers of fetal disease. An ultrasound for fetal anomalies would most likely have been performed earlier in the pregnancy. It is too late in the pregnancy to perform MSAFP screening. Also, MSAFP screening does not provide information related to fetal well-being. Indications for PUBS include prenatal diagnosis or inherited blood disorders, karyotyping of malformed fetuses, detection of fetal infection, determination of the acid-base status of a fetus with IUGR, and assessment and treatment of isoimmunization and thrombocytopenia in the fetus.

2. The nurse providing care for a woman with preterm labor who is receiving terbutaline would include which intervention to identify side effects of the drug? a.Assessing deep tendon reflexes (DTRs) b.Assessing for chest discomfort and palpitations c.Assessing for bradycardia d.Assessing for hypoglycemia

ANS: B Terbutaline is a b2-adrenergic agonist that affects the cardiopulmonary and metabolic systems of the mother. Signs of cardiopulmonary decompensation would include chest pain and palpitations. Assessing DTRs would not address these concerns. b2-Adrenergic agonist drugs cause tachycardia, not bradycardia. The metabolic effect leads to hyperglycemia, not hypoglycemia.

14. A pregnant woman at 29 weeks of gestation has been diagnosed with preterm labor. Her labor is being controlled with tocolytic medications. She asks when she would be able to go home. Which response by the nurse is most accurate? a."After the baby is born." b."When we can stabilize your preterm labor and arrange home health visits." c."Whenever the doctor says that it is okay." d."It depends on what kind of insurance coverage you have."

ANS: B The client's preterm labor is being controlled with tocolytics. Once she is stable, home care may be a viable option for this type of client. Care of a woman with preterm labor is multifactorial; the goal is to prevent delivery. In many cases this may be achieved at home. Care of the preterm client is multidisciplinary and multifactorial. Managed care may dictate earlier hospital discharges or a shift from hospital to home care. Insurance coverage may be one factor in the care of clients, but ultimately client safety remains the most important factor.

18. A woman has been diagnosed with a high risk pregnancy. She and her husband come into the office in a very anxious state. She seems to be coping by withdrawing from the discussion, showing declining interest. The nurse can best help the couple by: a. Telling her that the physician will isolate the problem with more tests. b. Encouraging her and urging her to continue with childbirth classes. c. Becoming assertive and laying out the decisions the couple needs to make. d. Downplaying her risks by citing success rate studies.

ANS: B The nurse can best help the woman and her husband regain a sense of control in their lives by providing support and encouragement (including active involvement in preparations and classes). The nurse can try to present opportunities for the couple to make as many choices as possible in prenatal care.

15. The nurse is caring for a client whose labor is being augmented with oxytocin. He or she recognizes that the oxytocin should be discontinued immediately if there is evidence of: a.Uterine contractions occurring every 8 to 10 minutes. b.A fetal heart rate (FHR) of 180 with absence of variability. c.The client's needing to void. d.Rupture of the client's amniotic membranes.

ANS: B This FHR is nonreassuring. The oxytocin should be discontinued immediately, and the physician should be notified. The oxytocin should be discontinued if uterine hyperstimulation occurs. Uterine contractions that are occurring every 8 to 10 minutes do not qualify as hyperstimulation. The client's needing to void is not an indication to discontinue the oxytocin induction immediately or to call the physician. Unless a change occurs in the FHR pattern that is nonreassuring or the client experiences uterine hyperstimulation, the oxytocin does not need to be discontinued. The physician should be notified that the client's membranes have ruptured.

6. A 26-year-old primigravida has come to the clinic for her regular prenatal visit at 12 weeks. She appears thin and somewhat nervous. She reports that she eats a well-balanced diet, although her weight is 5 pounds less than it was at her last visit. The results of laboratory studies confirm that she has a hyperthyroid condition. Based on the available data, the nurse formulates a plan of care. What nursing diagnosis is most appropriate for the woman at this time? a.Deficient fluid volume b.Imbalanced nutrition: less than body requirements c.Imbalanced nutrition: more than body requirements d.Disturbed sleep pattern

ANS: B This client's clinical cues include weight loss, which would support the nursing diagnosis of Imbalanced nutrition: less than body requirements. No clinical signs or symptoms support the nursing diagnosis of Deficient fluid volume. This client reports weight loss, not weight gain. Imbalanced nutrition: more than body requirements is not an appropriate nursing diagnosis. Although the client reports nervousness, based on the client's other clinical symptoms the most appropriate nursing diagnosis would be Imbalanced nutrition: less than body requirements.

8.. In terms of the incidence and classification of diabetes, maternity nurses should know that: a.Type 1 diabetes is most common. b.Type 2 diabetes often goes undiagnosed. c.Gestational diabetes mellitus (GDM) means that the woman will be receiving insulin treatment until 6 weeks after birth. d.Type 1 diabetes may become type 2 during pregnancy.

ANS: B Type 2 diabetes often goes undiagnosed because hyperglycemia develops gradually and often is not severe. Type 2 diabetes, sometimes called adult onset diabetes, is the most common. GDM refers to any degree of glucose intolerance first recognized during pregnancy. Insulin may or may not be needed. People do not go back and forth between types 1 and 2 diabetes.

24. Which nursing intervention is necessary before a second-trimester transabdominal ultrasound? a. Place the woman NPO for 12 hours. b. Instruct the woman to drink 1 to 2 quarts of water. c. Administer an enema. d. Perform an abdominal preparation.

ANS: B When the uterus is still in the pelvis, visualization may be difficult. It is necessary to perform the test when the woman has a full bladder, which provides a "window" through which the uterus and its contents can be viewed. The woman needs a full bladder to elevate the uterus; therefore being NPO is not appropriate. Neither an enema nor an abdominal preparation is necessary for this procedure.

The nerve block used in labor that provides anesthesia to the lower vagina and perineum is called: a. An epidural. b. A pudendal. c. A local. d. A spinal block.

ANS: B A pudendal block anesthetizes the lower vagina and perineum to provide anesthesia for an episiotomy and use of low forceps if needed. An epidural provides anesthesia for the uterus, perineum, and legs. A local provides anesthesia for the perineum at the site of the episiotomy. A spinal block provides anesthesia for the uterus, perineum, and down the legs.

With regard to a pregnant woman's anxiety and pain experience, nurses should be aware that: a. Even mild anxiety must be treated. b. Severe anxiety increases tension, which increases pain, which in turn increases fear and anxiety, and so on. c. Anxiety may increase the perception of pain, but it does not affect the mechanism of labor. d. Women who have had a painful labor will have learned from the experience and have less anxiety the second time because of increased familiarity.

ANS: B Anxiety and pain reinforce each other in a negative cycle. Mild anxiety is normal for a woman in labor and likely needs no special treatment other than the standard reassurances. Anxiety increases muscle tension and ultimately can build sufficiently to slow the progress of labor. Unfortunately, an anxious, painful first labor is likely to carry over, through expectations and memories, into an anxious and painful experience in the second pregnancy.

After change-of-shift report the nurse assumes care of a multiparous client in labor. The woman is complaining of pain that radiates to her abdominal wall, lower back, and buttocks and down her thighs. Before implementing a plan of care, the nurse should understand that this type of pain is: a. Visceral. b. Referred. c. Somatic. d. Afterpain.

ANS: B As labor progresses the woman often experiences referred pain. This occurs when pain that originates in the uterus radiates to the abdominal wall, the lumbosacral area of the back, the gluteal area, and thighs. The woman usually has pain only during a contraction and is free from pain between contractions. Visceral pain is that which predominates in the first stage of labor. This pain originates from cervical changes, distention of the lower uterine segment, and uterine ischemia. Visceral pain is located over the lower portion of the abdomen. Somatic pain is described as intense, sharp, burning, and well localized. This results from stretching of the perineal tissues and the pelvic floor. This occurs during the second stage of labor. Pain experienced during the third stage of labor or afterward during the early postpartum period is uterine. This pain is very similar to that experienced in the first stage of labor.

It is important for the nurse to develop a realistic birth plan with the pregnant woman in her care. The nurse can explain that a major advantage of nonpharmacologic pain management is: a. Greater and more complete pain relief is possible. b. No side effects or risks to the fetus are involved. c. The woman remains fully alert at all times. d. A more rapid labor is likely.

ANS: B Because nonpharmacologic pain management does not include analgesics, adjunct drugs, or anesthesia, it is harmless to the mother and the fetus. There is less pain relief with nonpharmacologic pain management during childbirth. The woman's alertness is not altered by medication; however, the increase in pain will decrease alertness. Pain management may or may not alter the length of labor. At times when pain is decreased, the mother relaxes and labor progresses at a quicker pace.

Which statement is the best rationale for assessing maternal vital signs between contractions? a. During a contraction, assessing fetal heart rates is the priority. b. Maternal circulating blood volume increases temporarily during contractions. c. Maternal blood flow to the heart is reduced during contractions. d. Vital signs taken during contractions are not accurate.

ANS: B During uterine contractions, blood flow to the placenta temporarily stops, causing a relative increase in the mother's blood volume, which in turn temporarily increases blood pressure and slows pulse. It is important to monitor fetal response to contractions; however, this question is concerned with the maternal vital signs. Maternal blood flow is increased during a contraction. Vital signs are altered by contractions but are considered accurate for that period of time.

With regard to systemic analgesics administered during labor, nurses should be aware that: a. Systemic analgesics cross the maternal blood-brain barrier as easily as they do the fetal blood-brain barrier. b. Effects on the fetus and newborn can include decreased alertness and delayed sucking. c. Intramuscular administration (IM) is preferred over intravenous (IV) administration. d. IV patient-controlled analgesia (PCA) results in increased use of an analgesic.

ANS: B Effects depend on the specific drug given, the dosage, and the timing. Systemic analgesics cross the fetal blood-brain barrier more readily than the maternal blood-brain barrier. IV administration is preferred over IM administration because the drug acts faster and more predictably. PCA results in decreased use of an analgesic.

The slight overlapping of cranial bones or shaping of the fetal head during labor is called: a. Lightening. b. Molding. c. Ferguson reflex. d. Valsalva maneuver.

ANS: B Fetal head formation is called molding. Molding also permits adaptation to various diameters of the maternal pelvis. Lightening is the mother's sensation of decreased abdominal distention, which usually occurs the week before labor. The Ferguson reflex is the contraction urge of the uterus after stimulation of the cervix. The Valsalva maneuver describes conscious pushing during the second stage of labor.

With regard to spinal and epidural (block) anesthesia, nurses should know that: a. This type of anesthesia is commonly used for cesarean births but is not suitable for vaginal births. b. A high incidence of after-birth headache is seen with spinal blocks. c. Epidural blocks allow the woman to move freely. d. Spinal and epidural blocks are never used together.

ANS: B Headaches may be prevented or mitigated to some degree by a number of methods. Spinal blocks may be used for vaginal births, but the woman must be assisted through labor. Epidural blocks limit the woman's ability to move freely. Combined use of spinal and epidural blocks is becoming increasingly popular.

The primary difference between the labor of a nullipara and that of a multipara is the: a. Amount of cervical dilation. c. Level of pain experienced. b. Total duration of labor. d. Sequence of labor mechanisms.

ANS: B In a first-time pregnancy, descent is usually slow but steady; in subsequent pregnancies, descent is more rapid, resulting in a shorter duration of labor. Cervical dilation is the same for all labors. Level of pain is individual to the woman, not to the number of labors she has experienced. The sequence of labor mechanisms is the same with all labors.

The role of the nurse with regard to informed consent is to: a. Inform the client about the procedure and have her sign the consent form. b. Act as a client advocate and help clarify the procedure and the options. c. Call the physician to see the client. d. Witness the signing of the consent form.

ANS: B Nurses play a part in the informed consent process by clarifying and describing procedures or by acting as the woman's advocate and asking the primary health care provider for further explanations. The physician is responsible for informing the woman of her options, explaining the procedure, and advising the client about potential risk factors. The physician must be present to explain the procedure to the client. However, the nurse's responsibilities go further than simply asking the physician to see the client. The nurse may witness the signing of the consent form. However, depending on the state's guidelines, the woman's husband or another hospital health care employee may sign as witness.

A first-time mother is concerned about the type of medications she will receive during labor. She is in a fair amount of pain and is nauseous. In addition, she appears to be very anxious. You explain that opioid analgesics often are used with sedatives because: a. "The two together work the best for you and your baby." b. "Sedatives help the opioid work better, and they also will assist you to relax and relieve your nausea." c. "They work better together so you can sleep until you have the baby." d. "This is what the doctor has ordered for you."

ANS: B Sedatives can be used to reduce the nausea and vomiting that often accompany opioid use. In addition, some ataractics reduce anxiety and apprehension and potentiate the opioid analgesic affects. A potentiator may cause the two drugs to work together more effectively, but it does not ensure maternal or fetal complications will not occur. Sedation may be a related effect of some ataractics, but it is not the goal. Furthermore, a woman is unlikely to be able to sleep through transitional labor and birth. "This is what the doctor has ordered for you" may be true, but it is not an acceptable comment for the nurse to make.

The nurse should be aware that an effective plan to achieve adequate pain relief without maternal risk is most effective if: a. The mother gives birth without any analgesic or anesthetic. b. The mother and family's priorities and preferences are incorporated into the plan. c. The primary health care provider decides the best pain relief for the mother and family. d. The nurse informs the family of all alternative methods of pain relief available in the hospital setting.

ANS: B The assessment of the woman, her fetus, and her labor is a joint effort of the nurse and the primary health care providers, who consult with the woman about their findings and recommendations. The needs of each woman are different, and many factors must be considered before a decision is made whether pharmacologic methods, nonpharmacologic methods, or a combination of the two will be used to manage labor pain.

To teach patients about the process of labor adequately, the nurse knows that which event is the best indicator of true labor? a. Bloody show b. Cervical dilation and effacement c. Fetal descent into the pelvic inlet d. Uterine contractions every 7 minutes

ANS: B The conclusive distinction between true and false labor is that contractions of true labor cause progressive change in the cervix. Bloody show can occur before true labor. Fetal descent can occur before true labor. False labor may have contractions that occur this frequently; however, this is usually inconsistent.

The nurse has received report regarding her patient in labor. The woman's last vaginal examination was recorded as 3 cm, 30%, and ?2-2. The nurse's interpretation of this assessment is that: a. The cervix is effaced 3 cm, it is dilated 30%, and the presenting part is 2 cm above the ischial spines. b. The cervix is 3 cm dilated, it is effaced 30%, and the presenting part is 2 cm above the ischial spines. c. The cervix is effaced 3 cm, it is dilated 30%, and the presenting part is 2 cm below the ischial spines. d. The cervix is dilated 3 cm, it is effaced 30%, and the presenting part is 2 cm below the ischial spines.

ANS: B The correct description of the vaginal examination for this woman in labor is the cervix is 3 cm dilated, it is effaced 30%, and the presenting part is 2 cm above the ischial spines. The sterile vaginal examination is recorded as centimeters of cervical dilation, percentage of cervical dilation, and the relationship of the presenting part to the ischial spines (either above or below).

As relates to fetal positioning during labor, nurses should be aware that: a. Position is a measure of the degree of descent of the presenting part of the fetus through the birth canal. b. Birth is imminent when the presenting part is at +4 to +5 cm below the spine. c. The largest transverse diameter of the presenting part is the suboccipitobregmatic diameter. d. Engagement is the term used to describe the beginning of labor.

ANS: B The station of the presenting part should be noted at the beginning of labor so that the rate of descent can be determined. Position is the relation of the presenting part of the fetus to the four quadrants of the mother's pelvis;station is the measure of degree of descent. The largest diameter usually is the biparietal diameter. The suboccipitobregmatic diameter is the smallest, although one of the most critical. Engagement often occurs in the weeks just before labor in nulliparas and before or during labor in multiparas.

To assist the woman after delivery of the infant, the nurse knows that the blood patch is used after spinal anesthesia to relieve: a. Hypotension. b. Headache. c. Neonatal respiratory depression. d. Loss of movement.

ANS: B The subarachnoid block may cause a postspinal headache resulting from loss of cerebrospinal fluid from the puncture in the dura. When blood is injected into the epidural space in the area of the dural puncture, it forms a seal over the hole to stop leaking of cerebrospinal fluid. Hypotension is prevented by increasing fluid volume before the procedure. Neonatal respiratory depression is not an expected outcome with spinal anesthesia. Loss of movement is an expected outcome of spinal anesthesia.

If an opioid antagonist is administered to a laboring woman, she should be told that: a. Her pain will decrease. b. Her pain will return. c. She will feel less anxious. d. She will no longer feel the urge to push.

ANS: B The woman should be told that the pain that was relieved by the opioid analgesic will return with administration of the opioid antagonist. Opioid antagonists, such as Narcan, promptly reverse the central nervous system (CNS) depressant effects of opioids. In addition, the antagonist counters the effect of the stress-induced levels of endorphins. An opioid antagonist is especially valuable if labor is more rapid than expected and birth is anticipated when the opioid is at its peak effect.

Maternal hypotension is a potential side effect of regional anesthesia and analgesia. What nursing interventions could you use to raise the client's blood pressure (Select all that apply)? a. Place the woman in a supine position. b. Place the woman in a lateral position. c. Increase intravenous (IV) fluids. d. Administer oxygen. e. Perform a vaginal examination.

ANS: B, C, D Nursing interventions for maternal hypotension arising from analgesia or anesthesia include turning the woman to a lateral position, increasing IV fluids, administering oxygen via face mask, elevating the woman's legs, notifying the physician, administering an IV vasopressor, and monitoring the maternal and fetal status at least every 5 minutes until these are stable. Placing the client in a supine position would cause venous compression, thereby limiting blood flow to and oxygenation of the placenta and fetus. A sterile vaginal examination has no bearing on maternal blood pressure.

41. Autoimmune disorders often occur during pregnancy because a large percentage of women with an autoimmune disorder are of childbearing age. Identify all disorders that fall into the category of collagen vascular disease. a.Multiple sclerosis b.Systemic lupus erythematosus c.Antiphospholipid syndrome d.Rheumatoid arthritis e.Myasthenia gravis

ANS: B, C, D, E Multiple sclerosis is not an autoimmune disorder. This patchy demyelinization of the spinal cord may be a viral disorder. Autoimmune disorders (collagen vascular disease) make up a large group of conditions that disrupt the function of the immune system of the body. They include those listed, as well as systemic sclerosis.

37. Complications and risks associated with cesarean births include (Select all that apply): a.Placental abruption. b.Wound dehiscence. c.Hemorrhage. d.Urinary tract infections. e.Fetal injuries.

ANS: B, C, D, E Placental abruption and placenta previa are both indications for cesarean birth and are not complications thereof. Wound dehiscence, hemorrhage, urinary tract infection, and fetal injuries are all possible complications and risks associated with delivery by cesarean section.

36. The nurse recognizes that uterine hyperstimulation with oxytocin requires emergency interventions. What clinical cues would alert the nurse that the woman is experiencing uterine hyperstimulation (Select all that apply)? a.Uterine contractions lasting <90 seconds and occurring >2 minutes in frequency b.Uterine contractions lasting >90 seconds and occurring <2 minutes in frequency c.Uterine tone <20 mm Hg d.Uterine tone >20 mm Hg e.Increased uterine activity accompanied by a nonreassuring fetal heart rate (FHR) and pattern

ANS: B, D, E Uterine contractions that occur less than 2 minutes apart and last more than 90 seconds, a uterine tone of over 20 mm Hg, and a nonreassuring FHR and pattern are all indications of uterine hyperstimulation with oxytocin administration. Uterine contractions that occur more than 2 minutes apart and last less than 90 seconds are the expected goal of oxytocin induction. A uterine tone of less than 20 mm Hg is normal.

46. Step 4 a.Without adding air, withdraw the correct dose of NPH insulin. b.Gently rotate the insulin to mix it, and wipe the stopper. c.Inject air equal to the dose of NPH insulin into the vial, and remove the syringe. d.Inject air equal to the dose of regular insulin into the vial, and withdraw the medication. e.Check the insulin bottles for the expiration date. f.Wash hands.

ANS: C

12. The nurse providing care for a woman with gestational diabetes understands that a laboratory test for glycosylated hemoglobin Alc: a.Is now done for all pregnant women, not just those with or likely to have diabetes. b.Is a snapshot of glucose control at the moment. c.Would be considered evidence of good diabetes control with a result of 5% to 6%. d.Is done on the patient's urine, not her blood.

ANS: C A score of 5% to 6% indicates good control. This is an extra test for diabetic women, not one done for all pregnant women. This test defines glycemic control over the previous 4 to 6 weeks. Glycosylated hemoglobin level tests are done on the blood.

5. At 35 weeks of pregnancy a woman experiences preterm labor. Tocolytics are administered and she is placed on bed rest, but she continues to experience regular uterine contractions, and her cervix is beginning to dilate and efface. What would be an important test for fetal well-being at this time? a. Percutaneous umbilical blood sampling (PUBS) b. Ultrasound for fetal size c. Amniocentesis for fetal lung maturity d. Nonstress test (NST)

ANS: C Amniocentesis would be performed to assess fetal lung maturity in the event of a preterm birth. Indications for PUBS include prenatal diagnosis or inherited blood disorders, karyotyping of malformed fetuses, detection of fetal infection, determination of the acid-base status of a fetus with intrauterine growth restriction, and assessment and treatment of isoimmunization and thrombocytopenia in the fetus. Typically, fetal size is determined by ultrasound during the second trimester and is not indicated in this scenario. NST measures the fetal response to fetal movement in a noncontracting mother.

33. The priority nursing care associated with an oxytocin (Pitocin) infusion is: a.Measuring urinary output. b.Increasing infusion rate every 30 minutes. c.Monitoring uterine response. d.Evaluating cervical dilation.

ANS: C Because of the risk of hyperstimulation, which could result in decreased placental perfusion and uterine rupture, the nurse's priority intervention is monitoring uterine response. Monitoring urinary output is also important; however, it is not the top priority during the administration of Pitocin. The infusion rate may be increased after proper assessment that it is an appropriate interval to do so. Monitoring labor progression is the standard of care for all labor patients.

18. As relates to the use of tocolytic therapy to suppress uterine activity, nurses should be aware that: a.The drugs can be given efficaciously up to the designated beginning of term at 37 weeks. b.There are no important maternal (as opposed to fetal) contraindications. c.Its most important function is to afford the opportunity to administer antenatal glucocorticoids. d.If the client develops pulmonary edema while receiving tocolytics, intravenous (IV) fluids should be given.

ANS: C Buying time for antenatal glucocorticoids to accelerate fetal lung development may be the best reason to use tocolytics. Once the pregnancy has reached 34 weeks, the risks of tocolytic therapy outweigh the benefits. There are important maternal contraindications to tocolytic therapy. Tocolytic-induced edema can be caused by IV fluids.

23. While working with the pregnant woman in her first trimester, the nurse is aware that chorionic villus sampling (CVS) can be performed during pregnancy at: a. 4 weeks c. 10 weeks b. 8 weeks d. 14 weeks

ANS: C CVS can be performed in the first or second trimester, ideally between 10 and 13 weeks of gestation. During this procedure, a small piece of tissue is removed from the fetal portion of the placenta. If performed after 9 completed weeks of gestation, the risk of limb reduction is no greater than in the general population.

14. With regard to amniocentesis, nurses should be aware that: a. Because of new imaging techniques, amniocentesis is now possible in the first trimester. b. Despite the use of ultrasound, complications still occur in the mother or infant in 5% to 10% of cases. c. The shake test, or bubble stability test, is a quick means of determining fetal maturity. d. The presence of meconium in the amniotic fluid is always cause for concern.

ANS: C Diluted fluid is mixed with ethanol and shaken. After 15 minutes, the bubbles tell the story. Amniocentesis is possible after the fourteenth week of pregnancy when the uterus becomes an abdominal organ. Complications occur in less than 1% of cases; many have been minimized or eliminated through the use of ultrasound. Meconium in the amniotic fluid before the beginning of labor is not usually a problem.

33. To manage her diabetes appropriately and ensure a good fetal outcome, the pregnant woman with diabetes will need to alter her diet by: a.Eating six small equal meals per day. b.Reducing carbohydrates in her diet. c.Eating her meals and snacks on a fixed schedule. d.Increasing her consumption of protein.

ANS: C Having a fixed meal schedule will provide the woman and the fetus with a steadier blood sugar level, provide better balance with insulin administration, and help prevent complications. It is more important to have a fixed meal schedule than equal division of food intake. Approximately 45% of the food eaten should be in the form of carbohydrates.

35. Nursing intervention for the pregnant diabetic patient is based on the knowledge that the need for insulin: a.Increases throughout pregnancy and the postpartum period. b.Decreases throughout pregnancy and the postpartum period. c.Varies depending on the stage of gestation. d.Should not change because the fetus produces its own insulin.

ANS: C Insulin needs decrease during the first trimester, when nausea, vomiting, and anorexia are a factor. They increase during the second and third trimesters, when the hormones of pregnancy create insulin resistance in maternal cells. Insulin needs increase during the second and third trimesters, when the hormones of pregnancy create insulin resistance in maternal cells. The insulin needs change throughout the different stages of pregnancy.

13. Prepidil (prostaglandin gel) has been ordered for a pregnant woman at 43 weeks of gestation. The nurse recognizes that this medication will be administered to: a.Enhance uteroplacental perfusion in an aging placenta b.Increase amniotic fluid volume. c.Ripen the cervix in preparation for labor induction. d.Stimulate the amniotic membranes to rupture.

ANS: C It is accurate to state that Prepidil will be administered to ripen the cervix in preparation for labor induction. It is not administered to enhance uteroplacental perfusion in an aging placenta, increase amniotic fluid volume, or stimulate the amniotic membranes to rupture.

17. The nurse providing care for the antepartum woman should understand that contraction stress test (CST): a. Sometimes uses vibroacoustic stimulation. b. Is an invasive test; however, contractions are stimulated. c. Is considered negative if no late decelerations are observed with the contractions. d. Is more effective than nonstress test (NST) if the membranes have already been ruptured.

ANS: C No late decelerations is good news. Vibroacoustic stimulation is sometimes used with NST. CST is invasive if stimulation is by intravenous oxytocin but not if by nipple stimulation and is contraindicated if the membranes have ruptured.

10. When nurses help their expectant mothers assess the daily fetal movement counts, they should be aware that: a. Alcohol or cigarette smoke can irritate the fetus into greater activity. b. "Kick counts" should be taken every half hour and averaged every 6 hours, with every other 6-hour stretch off. c. The fetal alarm signal should go off when fetal movements stop entirely for 12 hours. d. Obese mothers familiar with their bodies can assess fetal movement as well as average-size women.

ANS: C No movement in a 12-hour period is cause for investigation and possibly intervention. Alcohol and cigarette smoke temporarily reduce fetal movement. The mother should count fetal activity ("kick counts") two or three times daily for 60 minutes each time. Obese women have a harder time assessing fetal movement.

35. Surgical, medical, or mechanical methods may be used for labor induction. Which technique is considered a mechanical method of induction? a.Amniotomy b.Intravenous Pitocin c.Transcervical catheter d.Vaginal insertion of prostaglandins

ANS: C Placement of a balloon-tipped Foley catheter into the cervix is a mechanical method of induction. Other methods to expand and gradually dilate the cervix include hydroscopic dilators such as laminaria tents (made from desiccated seaweed), or Lamicel (contains magnesium sulfate). Amniotomy is a surgical method of augmentation and induction. Intravenous Pitocin and insertion of prostaglandins are medical methods of induction.

22. Which heart condition is not a contraindication for pregnancy? a.Peripartum cardiomyopathy b.Eisenmenger syndrome c.Heart transplant d.All of these contraindicate pregnancy.

ANS: C Pregnancy is contraindicated for peripartum cardiomyopathy and Eisenmenger syndrome. Women who have had heart transplants are successfully having babies. However, conception should be postponed for at least 1 year after transplantation.

9. Metabolic changes throughout pregnancy that affect glucose and insulin in the mother and the fetus are complicated but important to understand. Nurses should understand that: a.Insulin crosses the placenta to the fetus only in the first trimester, after which the fetus secretes its own. b.Women with insulin-dependent diabetes are prone to hyperglycemia during the first trimester because they are consuming more sugar. c.During the second and third trimesters, pregnancy exerts a diabetogenic effect that ensures an abundant supply of glucose for the fetus. d.Maternal insulin requirements steadily decline during pregnancy.

ANS: C Pregnant women develop increased insulin resistance during the second and third trimesters. Insulin never crosses the placenta; the fetus starts making its own insulin around the tenth week. As a result of normal metabolic changes during pregnancy, insulin-dependent women are prone to hypoglycemia (low levels). Maternal insulin requirements may double or quadruple by the end of pregnancy.

31. Which factor is known to increase the risk of gestational diabetes mellitus? a.Underweight before pregnancy b.Maternal age younger than 25 years c.Previous birth of large infant d.Previous diagnosis of type 2 diabetes mellitus

ANS: C Previous birth of a large infant suggests gestational diabetes mellitus. Obesity (BMI of 30 or greater) creates a higher risk for gestational diabetes. A woman younger than 25 years generally is not at risk for gestational diabetes mellitus. The person with type 2 diabetes mellitus already has diabetes and will continue to have it after pregnancy. Insulin may be required during pregnancy because oral hypoglycemia drugs are contraindicated during pregnancy.

1. In planning for home care of a woman with preterm labor, which concern must the nurse address? a.Nursing assessments will be different from those done in the hospital setting. b.Restricted activity and medications will be necessary to prevent recurrence of preterm labor. c.Prolonged bed rest may cause negative physiologic effects. d.Home health care providers will be necessary.

ANS: C Prolonged bed rest may cause adverse effects such as weight loss, loss of appetite, muscle wasting, weakness, bone demineralization, decreased cardiac output, risk for thrombophlebitis, alteration in bowel functions, sleep disturbance, and prolonged postpartum recovery. Nursing assessments will differ somewhat from those performed in the acute care setting, but this is not the concern that needs to be addressed. Restricted activity and medication may prevent preterm labor, but not in all women. In addition, the plan of care is individualized to meet the needs of each woman. Many women will receive home health nurse visits, but care is individualized for each woman.

19. A woman with asthma is experiencing a postpartum hemorrhage. Which drug would not be used to treat her bleeding because it may exacerbate her asthma? a.Pitocin b.Nonsteroidal antiinflammatory drugs (NSAIDs) c.Hemabate d.Fentanyl

ANS: C Prostaglandin derivatives should not be used to treat women with asthma, because they may exacerbate symptoms. Pitocin would be the drug of choice to treat this woman's bleeding because it would not exacerbate her asthma. NSAIDs are not used to treat bleeding. Fentanyl is used to treat pain, not bleeding.

29. _____ use/abuse during pregnancy causes vasoconstriction and decreased placental perfusion, resulting in maternal and neonatal complications. a.Alcohol b.Caffeine c.Tobacco d.Chocolate

ANS: C Smoking in pregnancy is known to cause a decrease in placental perfusion and has serious health risks, including bleeding complications, low birth weight, prematurity, miscarriage, stillbirth, and sudden infant death syndrome. Prenatal alcohol exposure is the single greatest preventable cause of mental retardation. Alcohol use during pregnancy can cause high blood pressure, miscarriage, premature birth, stillbirth, and anemia. Caffeine and chocolate may safely be consumed in small quantities during pregnancy.

25. To provide safe care for the woman, the nurse understands that which condition is a contraindication for an amniotomy? a.Dilation less than 3 cm b.Cephalic presentation c.-2 station d.Right occiput posterior position

ANS: C The dilation of the cervix must be great enough to determine labor. The presenting part of the fetus should be engaged and well applied to the cervix before the procedure in order to prevent cord prolapse. Amniotomy is deferred if the presenting part is higher in the pelvis. ROP indicates a cephalic presentation, which is appropriate for an amniotomy.

26. The most common neurologic disorder accompanying pregnancy is: a.Eclampsia. b.Bell's palsy c.Epilepsy. d.Multiple sclerosis.

ANS: C The effects of pregnancy on epilepsy are unpredictable. Eclampsia sometimes may be confused with epilepsy, which is the most common neurologic disorder accompanying pregnancy. Bell's palsy is a form of facial paralysis. Multiple sclerosis is a patchy demyelinization of the spinal cord that does not affect the normal course of pregnancy or birth.

20. The least common cause of long, difficult, or abnormal labor (dystocia) is: a.Midplane contracture of the pelvis. b.Compromised bearing-down efforts as a result of pain medication. c.Disproportion of the pelvis. d.Low-lying placenta.

ANS: C The least common cause of dystocia is disproportion of the pelvis.

24. The nurse providing care to a woman in labor should understand that cesarean birth: a.Is declining in frequency in the twenty-first century in the United States. b.Is more likely to be performed for poor women in public hospitals who do not receive the nurse counseling as do wealthier clients. c.Is performed primarily for the benefit of the fetus. d.Can be either elected or refused by women as their absolute legal righ

ANS: C The most common indications for cesarean birth are danger to the fetus related to labor and birth complications. Cesarean births are increasing in the United States in this century. Wealthier women who have health insurance and who give birth in a private hospital are more likely to experience cesarean birth. A woman's right to elect cesarean surgery is in dispute, as is her right to refuse it if in doing so she endangers the fetus. Legal issues are not absolutely clear.

19. In the past, factors to determine whether a woman was likely to develop a high risk pregnancy were evaluated primarily from a medical point of view. A broader, more comprehensive approach to high-risk pregnancy has been adopted today. There are now four categories based on threats to the health of the woman and the outcome of pregnancy. Which of the following is not one of these categories? a. Biophysical c. Geographic b. Psychosocial d. Environmental

ANS: C This category is correctly referred to as sociodemographic risk. These factors stem from the mother and her family. Ethnicity may be one of the risks to pregnancy; however, it is not the only factor in this category. Low income, lack of prenatal care, age, parity, and marital status also are included. Biophysical is one of the broad categories used for determining risk. These include genetic considerations, nutritional status, and medical and obstetric disorders. Psychosocial risks include smoking, caffeine, drugs, alcohol, and psychologic status. All of these adverse lifestyles can have a negative effect on the health of the mother or fetus. Environmental risks are risks that can affect both fertility and fetal development. These include infections, chemicals, radiation, pesticides, illicit drugs, and industrial pollutants.

8. A woman is having her first child. She has been in labor for 15 hours. Two hours ago her vaginal examination revealed the cervix to be dilated to 5 cm and 100% effaced, and the presenting part was at station 0. Five minutes ago her vaginal examination indicated that there had been no change. What abnormal labor pattern is associated with this description? a.Prolonged latent phase b.Protracted active phase c.Arrest of active phase d.Protracted descent

ANS: C With an arrest of the active phase, the progress of labor has stopped. This client has not had any anticipated cervical change, thus indicating an arrest of labor. In the nulliparous woman a prolonged latent phase typically would last more than 20 hours. A protracted active phase, the first or second stage of labor, would be prolonged (slow dilation). With protracted descent, the fetus would fail to descend at an anticipated rate during the deceleration phase and second stage of labor.

6. A primigravida at 40 weeks of gestation is having uterine contractions every 1.5 to 2 minutes and says that they are very painful. Her cervix is dilated 2 cm and has not changed in 3 hours. The woman is crying and wants an epidural. What is the likely status of this woman's labor? a.She is exhibiting hypotonic uterine dysfunction. b.She is experiencing a normal latent stage. c.She is exhibiting hypertonic uterine dysfunction. d.She is experiencing pelvic dystocia.

ANS: C Women who experience hypertonic uterine dysfunction, or primary dysfunctional labor, often are anxious first-time mothers who are having painful and frequent contractions that are ineffective at causing cervical dilation or effacement to progress. With hypotonic uterine dysfunction, the woman initially makes normal progress into the active stage of labor; then the contractions become weak and inefficient or stop altogether. The contraction pattern seen in this woman signifies hypertonic uterine activity. Typically uterine activity in this phase occurs at 4- to 5-minute intervals lasting 30 to 45 seconds. Pelvic dystocia can occur whenever contractures of the pelvic diameters reduce the capacity of the bony pelvis, including the inlet, midpelvis, outlet, or any combination of these planes.

37. In caring for a pregnant woman with sickle cell anemia, the nurse is aware that signs and symptoms of sickle cell crisis include: a.Anemia. b.Endometritis c.Fever and pain. d.Urinary tract infection.

ANS: C Women with sickle cell anemia have recurrent attacks (crisis) of fever and pain, most often in the abdomen, joints, and extremities. These attacks are attributed to vascular occlusion when RBCs assume the characteristic sickled shape. Crises are usually triggered by dehydration, hypoxia, or acidosis. Women with sickle cell anemia are not iron deficient. Therefore, routine iron supplementation, even that found in prenatal vitamins, should be avoided in order to prevent iron overload. Women with sickle cell trait usually are at greater risk for postpartum endometritis (uterine wall infection); however, this is not likely to occur in pregnancy and is not a sign of crisis. These women are at an increased risk for UTIs; however, this is not an indication of sickle cell crisis.

A laboring woman received an opioid agonist (meperidine) intravenously 90 minutes before she gave birth. Which medication should be available to reduce the postnatal effects of Demerol on the neonate? a. Fentanyl (Sublimaze) b. Promethazine (Phenergan) c. Naloxone (Narcan) d. Nalbuphine (Nubain)

ANS: C An opioid antagonist can be given to the newborn as one part of the treatment for neonatal narcosis, which is a state of central nervous system (CNS) depression in the newborn produced by an opioid. Opioid antagonists such as naloxone (Narcan) can promptly reverse the CNS depressant effects, especially respiratory depression. Fentanyl, promethazine, and nalbuphine do not act as opioid antagonists to reduce the postnatal effects of Demerol on the neonate. Although meperidine (Demerol) is a low-cost medication and readily available, the use of Demerol in labor has been controversial because of its effects on the neonate.

When assessing a woman in labor, the nurse is aware that the relationship of the fetal body parts to one another is called fetal: a. Lie. b. Presentation. c. Attitude. d. Position.

ANS: C Attitude is the relation of the fetal body parts to one another. Lie is the relation of the long axis (spine) of the fetus to the long axis (spine) of the mother. Presentation refers to the part of the fetus that enters the pelvic inlet first and leads through the birth canal during labor at term. Position is the relation of the presenting part to the four quadrants of the mother's pelvis.

Which nursing assessment indicates that a woman who is in second-stage labor is almost ready to give birth? a. The fetal head is felt at 0 station during vaginal examination. b. Bloody mucus discharge increases. c. The vulva bulges and encircles the fetal head. d. The membranes rupture during a contraction.

ANS: C During the active pushing (descent) phase, the woman has strong urges to bear down as the presenting part of the fetus descends and presses on the stretch receptors of the pelvic floor. The vulva stretches and begins to bulge encircling the fetal head. Birth of the head occurs when the station is +4. A 0 station indicates engagement. Bloody show occurs throughout the labor process and is not an indication of an imminent birth. Rupture of membranes can occur at any time during the labor process and does not indicate an imminent birth.

While providing care to a patient in active labor, the nurse should instruct the woman that: a. The supine position commonly used in the United States increases blood flow. b. The "all fours" position, on her hands and knees, is hard on her back. c. Frequent changes in position will help relieve her fatigue and increase her comfort. d. In a sitting or squatting position, her abdominal muscles will have to work harder.

ANS: C Frequent position changes relieve fatigue, increase comfort, and improve circulation. Blood flow can be compromised in the supine position; any upright position benefits cardiac output. The "all fours" position is used to relieve backache in certain situations. In a sitting or squatting position, the abdominal muscles work in greater harmony with uterine contractions.

Nurses with an understanding of cultural differences regarding likely reactions to pain may be better able to help clients. Nurses should know that _____ women may be stoic until late in labor, when they may become vocal and request pain relief. a. Chinese b. Arab or Middle Eastern c. Hispanic d. African-American

ANS: C Hispanic women may be stoic early and more vocal and ready for medications later. Chinese women may not show reactions to pain. Medical interventions must be offered more than once. Arab or Middle Eastern women may be vocal in response to labor pain from the start. They may prefer pain medications. African-American women may express pain openly; use of medications for pain is more likely to vary with the individual.

The laboring woman who imagines her body opening to let the baby out is using a mental technique called: a. Dissociation. b. Effleurage. c. Imagery. d. Distraction.

ANS: C Imagery is a technique of visualizing images that will assist the woman in coping with labor. Dissociation helps the woman learn to relax all muscles except those that are working. Effleurage is self-massage. Distraction can be used in the early latent phase by having the woman engage in another activity.

When assessing the fetus using Leopold maneuvers, the nurse feels a round, firm, movable fetal part in the fundal portion of the uterus and a long, smooth surface in the mother's right side close to midline. What is the likely position of the fetus? a. ROA b. LSP c. RSA d. LOA

ANS: C The fetus is positioned anteriorly in the right side of the maternal pelvis with the sacrum as the presenting part. RSA is the correct three-letter abbreviation to indicate this fetal position. The first letter indicates the presenting part in either the right or left side of the maternal pelvis. The second letter indicates the anatomic presenting part of the fetus. The third letter stands for the location of the presenting part in relation to the anterior, posterior, or transverse portion of the maternal pelvis. Palpation of a round, firm fetal part in the fundal portion of the uterus would be the fetal head, indicating that the fetus is in a breech position with the sacrum as the presenting part in the maternal pelvis. Palpation of the fetal spine along the mother's right side denotes the location of the presenting part in the mother's pelvis. The ability to palpate the fetal spine indicates that the fetus is anteriorly positioned in the maternal pelvis.

A woman in labor has just received an epidural block. The most important nursing intervention is to: a. Limit parenteral fluids. b. Monitor the fetus for possible tachycardia. c. Monitor the maternal blood pressure for possible hypotension. d. Monitor the maternal pulse for possible bradycardia.

ANS: C The most important nursing intervention for a woman who has received an epidural block is to monitor the maternal blood pressure frequently for signs of hypotension. Intravenous fluids are increased for a woman receiving an epidural, to prevent hypotension. The nurse observes for signs of fetal bradycardia. The nurse monitors for signs of maternal tachycardia secondary to hypotension.

With regard to factors that affect how the fetus moves through the birth canal, nurses should be aware that: a. The fetal attitude describes the angle at which the fetus exits the uterus. b. Of the two primary fetal lies, the horizontal lie is that in which the long axis of the fetus is parallel to the long axis of the mother. c. The normal attitude of the fetus is called general flexion. d. The transverse lie is preferred for vaginal birth.

ANS: C The normal attitude of the fetus is general flexion. The fetal attitude is the relation of fetal body parts to one another. The horizontal lie is perpendicular to the mother; in the longitudinal (or vertical) lie the long axes of the fetus and the mother are parallel. Vaginal birth cannot occur if the fetus stays in a transverse lie.

A woman has requested an epidural for her pain. She is 5 cm dilated and 100% effaced. The baby is in a vertex position and is engaged. The nurse increases the woman's intravenous fluid for a preprocedural bolus. She reviews her laboratory values and notes that the woman's hemoglobin is 12 g/dL, hematocrit is 38%, platelets are 67,000, and white blood cells (WBCs) are 12,000/mm3. Which factor would contraindicate an epidural for the woman? a. She is too far dilated. b. She is anemic. c. She has thrombocytopenia. d. She is septic.

ANS: C The platelet count indicates a coagulopathy, specifically, thrombocytopenia (low platelets), which is a contraindication to epidural analgesia/anesthesia. Typically epidural analgesia/anesthesia is used in the laboring woman when a regular labor pattern has been achieved, as evidenced by progressive cervical change. The laboratory values show that the woman's hemoglobin and hematocrit are in the normal range and show a slight increase in the WBC count that is not uncommon in laboring women.

With regard to the turns and other adjustments of the fetus during the birth process, known as the mechanism of labor, nurses should be aware that: a. The seven critical movements must progress in a more or less orderly sequence. b. Asynclitism sometimes is achieved by means of the Leopold maneuver. c. The effects of the forces determining descent are modified by the shape of the woman's pelvis and the size of the fetal head. d. At birth the baby is said to achieve "restitution" (i.e., a return to the C-shape of the womb).

ANS: C The size of the maternal pelvis and the ability of the fetal head to mold also affect the process. The seven identifiable movements of the mechanism of labor occur in combinations simultaneously, not in precise sequences. Asynclitism is the deflection of the baby's head; the Leopold maneuver is a means of judging descent by palpating the mother's abdomen. Restitution is the rotation of the baby's head after the infant is born.

A woman in labor is breathing into a mouthpiece just before the start of her regular contractions. As she inhales, a valve opens, and gas is released. She continues to inhale the gas slowly and deeply until the contraction starts to subside. When the inhalation stops, the valve closes. This procedure is: a. Not used much anymore. b. Likely to be used in the second stage of labor but not in the first stage. c. An application of nitrous oxide. d. A prelude to cesarean birth.

ANS: C This is an application of nitrous oxide, which could be used in either the first or second stage of labor (or both) as part of the preparation for a vaginal birth. Nitrous oxide is self-administered and found to be very helpful.

The maternity nurse understands that as the uterus contracts during labor, maternal-fetal exchange of oxygen and waste products: a. Continues except when placental functions are reduced. b. Increases as blood pressure decreases. c. Diminishes as the spiral arteries are compressed. d. Is not significantly affected.

ANS: C Uterine contractions during labor tend to decrease circulation through the spiral electrodes and subsequent perfusion through the intervillous space. The maternal blood supply to the placenta gradually stops with contractions. The exchange of oxygen and waste products decreases. The exchange of oxygen and waste products is affected by contractions.

47. Step 5 a.Without adding air, withdraw the correct dose of NPH insulin. b.Gently rotate the insulin to mix it, and wipe the stopper. c.Inject air equal to the dose of NPH insulin into the vial, and remove the syringe. d.Inject air equal to the dose of regular insulin into the vial, and withdraw the medication. e.Check the insulin bottles for the expiration date. f.Wash hands.

ANS: D

13. A woman with gestational diabetes has had little or no experience reading and interpreting glucose levels. She shows the nurse her readings for the past few days. Which one should the nurse tell her indicates a need for adjustment (insulin or sugar)? a.75 mg/dL before lunch. This is low; better eat now. b.115 mg/dL 1 hour after lunch. This is a little high; maybe eat a little less next time. c.115 mg/dL 2 hours after lunch; This is too high; it is time for insulin. d.60 mg/dL just after waking up from a nap. This is too low; maybe eat a snack before going to sleep.

ANS: D 60 mg/dL after waking from a nap is too low. During hours of sleep glucose levels should not be less than 70 mg/dL. Snacks before sleeping can be helpful. The premeal acceptable range is 65 to 95 mg/dL. The readings 1 hour after a meal should be less than 140 mg/dL. Two hours after eating, the readings should be less than 120 mg/dL.

5. A woman at 26 weeks of gestation is being assessed to determine whether she is experiencing preterm labor. What finding indicates that preterm labor is occurring? a.Estriol is not found in maternal saliva. b.Irregular, mild uterine contractions are occurring every 12 to 15 minutes. c.Fetal fibronectin is present in vaginal secretions. d.The cervix is effacing and dilated to 2 cm.

ANS: D Cervical changes such as shortened endocervical length, effacement, and dilation are predictors of imminent preterm labor. Changes in the cervix accompanied by regular contractions indicate labor at any gestation. Estriol is a form of estrogen produced by the fetus that is present in plasma at 9 weeks of gestation. Levels of salivary estriol have been shown to increase before preterm birth. Irregular, mild contractions that do not cause cervical change are not considered a threat. The presence of fetal fibronectin in vaginal secretions between 24 and 36 weeks of gestation could predict preterm labor, but it has only a 20% to 40% positive predictive value. Of more importance are other physiologic clues of preterm labor such as cervical changes.

21. Since the gene for cystic fibrosis was identified in 1989, data can be collected for the purposes of genetic counseling for couples regarding carrier status. According to statistics, how often does cystic fibrosis occur in Caucasian live births? a.1 in 100 b.1 in 1200 c.1 in 2500 d.1 in 3000

ANS: D Cystic fibrosis occurs in about 1 in 3000 Caucasian live births.

17. With regard to the care management of preterm labor, nurses should be aware that: a.Because all women must be considered at risk for preterm labor and prediction is so hit-and-miss, teaching pregnant women the symptoms probably causes more harm through false alarms. b.Braxton Hicks contractions often signal the onset of preterm labor. c.Because preterm labor is likely to be the start of an extended labor, a woman with symptoms can wait several hours before contacting the primary caregiver. d.The diagnosis of preterm labor is based on gestational age, uterine activity, and progressive cervical change.

ANS: D Gestational age of 20 to 37 weeks, uterine contractions, and a cervix that is 80% effaced or dilated 2 cm indicates preterm labor. It is essential that nurses teach women how to detect the early symptoms of preterm labor. Braxton Hicks contractions resemble preterm labor contractions, but they are not true labor. Waiting too long to see a health care provider could result in not administering essential medications. Preterm labor is not necessarily long-term labor.

1. A woman arrives at the clinic seeking confirmation that she is pregnant. The following information is obtained: She is 24 years old with a body mass index (BMI) of 17.5. She admits to having used cocaine "several times" during the past year and drinks alcohol occasionally. Her blood pressure (BP) is 108/70 mm Hg, her pulse rate is 72 beats/min, and her respiratory rate is 16 breaths/min. The family history is positive for diabetes mellitus and cancer. Her sister recently gave birth to an infant with a neural tube defect (NTD). Which characteristics place the woman in a high risk category? a. Blood pressure, age, BMI b. Drug/alcohol use, age, family history c. Family history, blood pressure, BMI d. Family history, BMI, drug/alcohol abuse

ANS: D Her family history of NTD, low BMI, and substance abuse all are high risk factors of pregnancy. The woman's BP is normal, and her age does not put her at risk. Her BMI is low and may indicate poor nutritional status, which would be a high risk. The woman's drug/alcohol use and family history put her in a high risk category, but her age does not. The woman's family history puts her in a high risk category. Her BMI is low and may indicate poor nutritional status, which would be high risk. Her BP is normal.

21. Nurses should be aware that the induction of labor: a.Can be achieved by external and internal version techniques. b.Is also known as a trial of labor (TOL). c.Is almost always done for medical reasons. d.Is rated for viability by a Bishop score.

ANS: D Induction of labor is likely to be more successful with a Bishop score of 9 or higher for first-time mothers and 5 or higher for veterans. Version is turning of the fetus to a better position by a physician for an easier or safer birth. A trial of labor is the observance of a woman and her fetus for several hours of active labor to assess the safety of vaginal birth. Two thirds of cases of induced labor are elective and are not done for medical reasons

28. The standard of care for obstetrics dictates that an internal version may be used to manipulate the: a.Fetus from a breech to a cephalic presentation before labor begins. b.Fetus from a transverse lie to a longitudinal lie before cesarean birth. c.Second twin from an oblique lie to a transverse lie before labor begins. d.Second twin from a transverse lie to a breech presentation during vaginal birth.

ANS: D Internal version is used only during vaginal birth to manipulate the second twin into a presentation that allows it to be born vaginally. For internal version to occur, the cervix needs to be completely dilated.

15. Nurses should be aware of the strengths and limitations of various biochemical assessments during pregnancy, including that: a. Chorionic villus sampling (CVS) is becoming more popular because it provides early diagnosis. b. Maternal serum alpha-fetoprotein (MSAFP) screening is recommended only for women at risk for neural tube defects. c. Percutaneous umbilical blood sampling (PUBS) is one of the triple-marker tests for Down syndrome. d. MSAFP is a screening tool only; it identifies candidates for more definitive procedures.

ANS: D MSAFP is a screening tool, not a diagnostic tool. CVS provides a rapid result, but it is declining in popularity because of advances in noninvasive screening techniques. MSAFP screening is recommended for all pregnant women. MSAFP screening, not PUBS, is part of the triple-marker tests for Down syndrome.

36. What form of heart disease in women of childbearing years usually has a benign effect on pregnancy? a.Cardiomyopathy b. Rheumatic heart disease c.Congenital heart disease d.Mitral valve prolapse

ANS: D Mitral valve prolapse is a benign condition that is usually asymptomatic. Cardiomyopathy produces congestive heart failure during pregnancy. Rheumatic heart disease can lead to heart failure during pregnancy. Some congenital heart diseases produce pulmonary hypertension or endocarditis during pregnancy.

32. Glucose metabolism is profoundly affected during pregnancy because: a.Pancreatic function in the islets of Langerhans is affected by pregnancy. b.The pregnant woman uses glucose at a more rapid rate than the nonpregnant woman. c.The pregnant woman increases her dietary intake significantly. d.Placental hormones are antagonistic to insulin, thus resulting in insulin resistance.

ANS: D Placental hormones, estrogen, progesterone, and human placental lactogen (HPL) create insulin resistance. Insulin also is broken down more quickly by the enzyme placental insulinase. Pancreatic functioning is not affected by pregnancy. The glucose requirements differ because of the growing fetus. The pregnant woman should increase her intake by 200 calories a day.

7. Which assessment is least likely to be associated with a breech presentation? a.Meconium-stained amniotic fluid b.Fetal heart tones heard at or above the maternal umbilicus c.Preterm labor and birth d.Post-term gestation

ANS: D Post-term gestation is not likely to be seen with a breech presentation. The presence of meconium in a breech presentation may result from pressure on the fetal wall as it traverses the birth canal. Fetal heart tones heard at the level of the umbilical level of the mother are a typical finding in a breech presentation because the fetal back would be located in the upper abdominal area. Breech presentations often occur in preterm births.

11. For a woman at 42 weeks of gestation, which finding would require further assessment by the nurse? a.Fetal heart rate of 116 beats/min b.Cervix dilated 2 cm and 50% effaced c.Score of 8 on the biophysical profile d.One fetal movement noted in 1 hour of assessment by the mother

ANS: D Self-care in a post-term pregnancy should include performing daily fetal kick counts three times per day. The mother should feel four fetal movements per hour. If fewer than four movements have been felt by the mother, she should count for 1 more hour. Fewer than four movements in that hour warrants evaluation. Normal findings in a 42-week gestation include fetal heart rate of 116 beats/min, cervix dilated 20 cm and 50% effaced, and a score of 8 on the biophysical profile.

16. When caring for a pregnant woman with cardiac problems, the nurse must be alert for signs and symptoms of cardiac decompensation, which include: a.A regular heart rate and hypertension. b.An increased urinary output, tachycardia, and dry cough. c.Shortness of breath, bradycardia, and hypertension. d.Dyspnea; crackles; and an irregular, weak pulse.

ANS: D Signs of cardiac decompensation include dyspnea; crackles; an irregular, weak, rapid pulse; rapid respirations; a moist, frequent cough; generalized edema; increasing fatigue; and cyanosis of the lips and nail beds. A regular heart rate and hypertension are not generally associated with cardiac decompensation. Tachycardia would indicate cardiac decompensation, but increased urinary output and a dry cough would not. Shortness of breath would indicate cardiac decompensation, but bradycardia and hypertension would not.

23. During a physical assessment of an at-risk client, the nurse notes generalized edema, crackles at the base of the lungs, and some pulse irregularity. These are most likely signs of: a.Euglycemia. b.Rheumatic fever. c.Pneumonia. d.Cardiac decompensation.

ANS: D Symptoms of cardiac decompensation may appear abruptly or gradually. Euglycemia is a condition of normal glucose levels. These symptoms indicate cardiac decompensation. Rheumatic fever can cause heart problems, but it does not manifest with these symptoms, which indicate cardiac decompensation. Pneumonia is an inflammation of the lungs and would not likely generate these symptoms, which indicate cardiac decompensation.

25. The nurse recognizes that a nonstress test (NST) in which two or more fetal heart rate (FHR) accelerations of 15 beats/min or more occur with fetal movement in a 20-minute period is: a. Nonreactive c. Negative b. Positive d. Reactive

ANS: D The NST is reactive (normal) when two or more FHR accelerations of at least 15 beats/min (each with a duration of at least 15 seconds) occur in a 20-minute period. A nonreactive result means that the heart rate did not accelerate during fetal movement. A positive result is not used with NST. Contraction stress test (CST) uses positive as a result term. A negative result is not used with NST. CST uses negative as a result term.

32. The priority nursing intervention after an amniotomy should be to: a.Assess the color of the amniotic fluid. b.Change the patient's gown. c.Estimate the amount of amniotic fluid. d.Assess the fetal heart rate.

ANS: D The fetal heart rate must be assessed immediately after the rupture of the membranes to determine whether cord prolapse or compression has occurred. Secondary to FHR assessment, amniotic fluid amount, color, odor, and consistency is assessed. Dry clothing is important for patient comfort; however, it is not the top priority.

29. The nurse practicing in a labor setting knows that the woman most at risk for uterine rupture is: a.A gravida 3 who has had two low-segment transverse cesarean births. b.A gravida 2 who had a low-segment vertical incision for delivery of a 10-pound infant. c.A gravida 5 who had two vaginal births and two cesarean births. d.A gravida 4 who has had all cesarean births.

ANS: D The risk of uterine rupture increases for the patient who has had multiple prior births with no vaginal births. As the number of prior uterine incisions increases, so does the risk for uterine rupture. Low-segment transverse cesarean scars do not predispose the patient to uterine rupture.

3. In evaluating the effectiveness of magnesium sulfate for the treatment of preterm labor, what finding would alert the nurse to possible side effects? a.Urine output of 160 mL in 4 hours b.Deep tendon reflexes 2+ and no clonus c.Respiratory rate of 16 breaths/min d.Serum magnesium level of 10 mg/dL

ANS: D The therapeutic range for magnesium sulfate management is 5 to 8 mg/dL. A serum magnesium level of 10 mg/dL could lead to signs and symptoms of magnesium toxicity, including oliguria and respiratory distress. Urine output of 160 mL in 4 hours, deep tendon reflexes 2+ with no clonus, and respiratory rate of 16 breaths/min are normal findings.

11. In comparing the abdominal and transvaginal methods of ultrasound examination, nurses should explain to their clients that: a. Both require the woman to have a full bladder. b. The abdominal examination is more useful in the first trimester. c. Initially the transvaginal examination can be painful. d. The transvaginal examination allows pelvic anatomy to be evaluated in greater detail.

ANS: D The transvaginal examination allows pelvic anatomy to be evaluated in greater detail and allows intrauterine pregnancies to be diagnosed earlier. The abdominal examination requires a full bladder; the transvaginal examination requires an empty bladder. The transvaginal examination is more useful in the first trimester; the abdominal examination works better after the first trimester. Neither method should be painful, although with the transvaginal examination the woman feels pressure as the probe is moved.

6. A 40-year-old woman is 10 weeks pregnant. Which diagnostic tool would be appropriate to suggest to her at this time? a. Biophysical profile (BPP) b. Amniocentesis c. Maternal serum alpha-fetoprotein (MSAFP) screening d. Transvaginal ultrasound

ANS: D Ultrasound would be performed at this gestational age for biophysical assessment of the infant. BPP would be a method of biophysical assessment of fetal well-being in the third trimester. Amniocentesis is performed after the fourteenth week of pregnancy. MSAFP screening is performed from week 15 to week 22 of gestation (weeks 16 to 18 are ideal).

3. In planning for the care of a 30-year-old woman with pregestational diabetes, the nurse recognizes that the most important factor affecting pregnancy outcome is the: a.Mother's age. b.Number of years since diabetes was diagnosed. c.Amount of insulin required prenatally. d.Degree of glycemic control during pregnancy.

ANS: D Women with excellent glucose control and no blood vessel disease should have good pregnancy outcomes.

Which basic type of pelvis includes the correct description and percentage of occurrence in women? a. Gynecoid: classic female; heart shaped; 75% b. Android: resembling the male; wider oval; 15% c. Anthropoid: resembling the ape; narrower; 10% d. Platypelloid: flattened, wide, shallow; 3%

ANS: D A platypelloid pelvis is flattened, wide, and shallow; about 3% of women have this shape. The gynecoid shape is the classical female shape, slightly ovoid and rounded; about 50% of women have this shape. An android, or malelike, pelvis is heart shaped; about 23% of women have this shape. An anthropoid, or apelike, pelvis is oval and wider; about 24% of women have this shape.

The obstetric nurse is preparing the patient for an emergency cesarean birth, with no time to administer spinal anesthesia. The nurse is aware and prepared for the greatest risk of administering general anesthesia to the patient. This risk is: a. Respiratory depression. b. Uterine relaxation. c. Inadequate muscle relaxation. d. Aspiration of stomach contents.

ANS: D Aspiration of acidic gastric contents with possible airway obstruction is a potentially fatal complication of general anesthesia. Respirations can be altered during general anesthesia, and the anesthesiologist will take precautions to maintain proper oxygenation. Uterine relaxation can occur with some anesthesia; however, this can be monitored and prevented. Inadequate muscle relaxation can be improved with medication.

An 18-year-old pregnant woman, gravida 1, is admitted to the labor and birth unit with moderate contractions every 5 minutes that last 40 seconds. The woman states, "My contractions are so strong that I don't know what to do with myself." The nurse should: a. Assess for fetal well-being. b. Encourage the woman to lie on her side. c. Disturb the woman as little as possible. d. Recognize that pain is personalized for each individual.

ANS: D Each woman's pain during childbirth is unique and is influenced by a variety of physiologic, psychosocial, and environmental factors. A critical issue for the nurse is how support can make a difference in the pain of the woman during labor and birth. Assessing for fetal well-being includes no information that would indicate fetal distress or a logical reason to be overly concerned about the well-being of the fetus. The left lateral position is used to alleviate fetal distress, not maternal stress. The nurse has an obligation to provide physical, emotional, and psychosocial care and support to the laboring woman. This client clearly needs support.

In order to care for obstetric patients adequately, the nurse understands that labor contractions facilitate cervical dilation by: a. Contracting the lower uterine segment. b. Enlarging the internal size of the uterus. c. Promoting blood flow to the cervix. d. Pulling the cervix over the fetus and amniotic sac.

ANS: D Effective uterine contractions pull the cervix upward at the same time that the fetus and amniotic sac are pushed downward. The contractions are stronger at the fundus. The internal size becomes smaller with the contractions; this helps to push the fetus down. Blood flow decreases to the uterus during a contraction.

In the current practice of childbirth preparation, emphasis is placed on: a. The Dick-Read (natural) childbirth method. b. The Lamaze (psychoprophylactic) method. c. The Bradley (husband-coached) method. d. Having expectant parents attend childbirth preparation in any or no specific method.

ANS: D Encouraging expectant parents to attend childbirth preparation class is most important because preparation increases a woman's confidence and thus her ability to cope with labor and birth. Although still popular, the "method" format of classes is being replaced with other offerings such as Hypnobirthing and Birthing from Within.

Which statement correctly describes the effects of various pain factors? a. Higher prostaglandin levels arising from dysmenorrhea can blunt the pain of childbirth. b. Upright positions in labor increase the pain factor because they cause greater fatigue. c. Women who move around trying different positions are experiencing more pain. d. Levels of pain-mitigating b-endorphins are higher during a spontaneous, natural childbirth.

ANS: D Higher endorphin levels help women tolerate pain and reduce anxiety and irritability. Higher prostaglandin levels correspond to more severe labor pains. Upright positions in labor usually result in improved comfort and less pain. Moving freely to find more comfortable positions is important for reducing pain and muscle tension.

Which method of pain management is safest for a gravida 3 para 2 admitted at 8 cm cervical dilation? a. Epidural anesthesia b. Narcotics c. Spinal block d. Breathing and relaxation techniques

ANS: D Nonpharmacologic methods of pain management may be the best option for a woman in advanced labor. It is unlikely that enough time remains to administer epidural or spinal anesthesia. A narcotic given at this time may reach its peak about the time of birth and result in respiratory depression in the newborn.

In order to evaluate the condition of the patient accurately during labor, the nurse should be aware that: a. The woman's blood pressure will increase during contractions and fall back to prelabor normal between contractions. b. Use of the Valsalva maneuver is encouraged during the second stage of labor to relieve fetal hypoxia. c. Having the woman point her toes will reduce leg cramps. d. The endogenous endorphins released during labor will raise the woman's pain threshold and produce sedation.

ANS: D The endogenous endorphins released during labor will raise the woman's pain threshold and produce sedation. In addition, physiologic anesthesia of the perineal tissues, caused by the pressure of the presenting part, decreases the mother's perception of pain. Blood pressure increases during contractions but remains somewhat elevated between them. Use of the Valsalva maneuver is discouraged during second stage labor because of a number of unhealthy outcomes, including fetal hypoxia. Pointing the toes can cause leg cramps, as can the process of labor itself.

The factors that affect the process of labor and birth, known commonly as the five Ps, include all except: a. Passenger. b. Passageway. c. Powers. d. Pressure.

ANS: D The five Ps are passenger (fetus and placenta), passageway (birth canal), powers (contractions), position of the mother, and psychologic response.

A new mother asks the nurse when the "soft spot" on her son's head will go away. The nurse's answer is based on the knowledge that the anterior fontanel closes after birth by _____ months. a. 2 b. 8 c. 12 d. 18

ANS: D The larger of the two fontanels, the anterior fontanel, closes by 18 months after birth.

A primigravida at 39 weeks of gestation is observed for 2 hours in the intrapartum unit. The fetal heart rate has been normal. Contractions are 5 to 9 minutes apart, 20 to 30 seconds in duration, and of mild intensity. Cervical dilation is 1 to 2 cm and uneffaced (unchanged from admission). Membranes are intact. The nurse should expect the woman to be: a. Admitted and prepared for a cesarean birth. b. Admitted for extended observation. c. Discharged home with a sedative. d. Discharged home to await the onset of true labor.

ANS: D This situation describes a woman with normal assessments who is probably in false labor and will probably not deliver rapidly once true labor begins. These are all indications of false labor without fetal distress. There is no indication that further assessment or cesarean birth is indicated. The patient will likely be discharged; however, there is no indication that a sedative is needed.

A woman in the active phase of the first stage of labor is using a shallow pattern of breathing, which is about twice the normal adult breathing rate. She starts to complain about feeling lightheaded and dizzy and states that her fingers are tingling. The nurse should: a. Notify the woman's physician. b. Tell the woman to slow the pace of her breathing. c. Administer oxygen via a mask or nasal cannula. d. Help her breathe into a paper bag

ANS: D This woman is experiencing the side effects of hyperventilation, which include the symptoms of lightheadedness, dizziness, tingling of the fingers, or circumoral numbness. Having the woman breathe into a paper bag held tightly around her mouth and nose may eliminate respiratory alkalosis. This enables her to rebreathe carbon dioxide and replace the bicarbonate ion.

Maternity nurses often have to answer questions about the many, sometimes unusual ways people have tried to make the birthing experience more comfortable. For instance, nurses should be aware that: a. Music supplied by the support person has to be discouraged because it could disturb others or upset the hospital routine. b. Women in labor can benefit from sitting in a bathtub, but they must limit immersion to no longer than 15 minutes at a time. c. Effleurage is permissible, but counterpressure is almost always counterproductive. d. Electrodes attached to either side of the spine to provide high-intensity electrical impulses facilitate the release of endorphins.

ANS: D Transcutaneous electrical nerve stimulation does help. Music may be very helpful for reducing tension and certainly can be accommodated by the hospital. Women can stay in a bath as long as they want, although repeated baths with breaks may be more effective than a long soak. Counterpressure can help the woman cope with lower back pain.

44. Step 2 a.Without adding air, withdraw the correct dose of NPH insulin. b.Gently rotate the insulin to mix it, and wipe the stopper. c.Inject air equal to the dose of NPH insulin into the vial, and remove the syringe. d.Inject air equal to the dose of regular insulin into the vial, and withdraw the medication. e.Check the insulin bottles for the expiration date. f.Wash hands.

ANS: E

MATCHING 43. Step 1 a.Without adding air, withdraw the correct dose of NPH insulin. b.Gently rotate the insulin to mix it, and wipe the stopper. c.Inject air equal to the dose of NPH insulin into the vial, and remove the syringe. d.Inject air equal to the dose of regular insulin into the vial, and withdraw the medication. e.Check the insulin bottles for the expiration date. f.Wash hands.

ANS: F

A patient whose cervix is dilated to 5 cm is considered to be in which phase of labor?

Active phase

Nurses can help their clients by keeping them informed about the distinctive stages of labor. What description of the phases of the first stage of labor is accurate?

Active: Moderate, regular contractions; 4- to 7-cm dilation; duration of 3 to 6 hours

Instruct the patient and partner in the use of specific relaxation techniques.

Acute pain related to contractions

Step 4

After obtaining permission, gently insert the index and middle fingers into the vagina.

A

An adolescent boy is brought to the emergency department after a motorcycle accident. His respirations are deep, periodic, and gasping. There are extreme fluctuations in blood pressure. Pupils are dilated and fixed. What type of head injury should the nurse suspect? a. Brainstem b. Subdural hemorrhage c. Skull fracture d. Epidural hemorrhage

Concerning the third stage of labor, nurses should be aware that:

An expectant or active approach to managing this stage of labor reduces the risk of complications.

A

An important nursing intervention when caring for a child who is experiencing a seizure is to: a. Describe and record the seizure activity observed. b. Restrain the child when seizure occurs to prevent bodily harm. c. Place a tongue blade between the teeth if they become clenched. d. Suction the child during a seizure to prevent aspiration.

Orient the patient and family to the labor and birth unit.

Anxiety related to labor and the birthing process

In documenting labor experiences, nurses should know that a uterine contraction is described according to all these characteristics except:

Appearance shape and height

A laboring woman is lying in the supine position. The most appropriate nursing action at this time is to:

Ask her to turn to one side

When a nulliparous woman telephones the hospital to report that she is in labor, the nurse initially should:

Ask the woman to describe why she believes she is in labor.

A multiparous woman has been in labor for 8 hours. Her membranes have just ruptured. The nurse's initial response would be to:

Assess the fetal heart rate and pattern.

*A 4-month-old infant has gastroesophageal reflux disease (GERD) but is thriving without other complications. What should the nurse suggest to minimize reflux? a. Place in Trendelenburg position after eating. b. Thicken formula with rice cereal. c. Give continuous nasogastric tube feedings. d. Give larger, less frequent feedings.

B

*Acute diarrhea is often caused by: a. Hirschsprung's disease. c. Hypothyroidism. b. Antibiotic therapy. d. Meconium ileus.

B

*An infant is brought to the emergency department with poor skin turgor, weight loss, lethargy, and tachycardia. This is suggestive of: a. Overhydration. c. Sodium excess. b. Dehydration. d. Calcium excess.

B

*Which vaccine is now recommended for the immunization of all newborns? a. Hepatitis A vaccine b. Hepatitis B vaccine c. Hepatitis C vaccine d. Hepatitis A, B, and C vaccines

B

10. Which finding should cause the nurse to suspect a diagnosis of spastic cerebral palsy? a. Tremulous movements at rest and with activity b. Positive Babinski reflex c. Writhing, uncontrolled, involuntary movements d. Clumsy, uncoordinated movements

B

15. Which interventions should the nurse include in the plan of care for the infant awaiting surgical closure of a myelomeningocele sac? a. Open to air b. Covered with a sterile, moist, nonadherent dressing c. Reinforcement of the original dressing if drainage noted d. A diaper secured over the dressing

B

16. The nurse is admitting a school-age child with suspected Guillain-Barré syndrome (GBS). Which nursing intervention is a priority in the care for this child? a. Monitoring intake and output b. Assessing respiratory efforts c. Placing on a telemetry monitor d. Obtaining laboratory studies

B

6. How much folic acid is recommended for women of childbearing age? a. 0.1 mg b. 0.4 mg c. 1.5 mg d. 2 mg

B

7. The nurse is caring for a neonate born with a myelomeningocele. Surgery to repair the defect is scheduled the next day. The most appropriate way to position and feed this neonate is to place him: a. prone and tube feed. b. prone, turn head to side, and nipple feed. c. supine in infant carrier and nipple feed. d. supine, with defect supported with rolled blankets, and nipple feed.

B

A child is admitted with acute glomerulonephritis. The nurse would expect the urinalysis during this acute phase to show: a. Bacteriuria and hematuria. b. Hematuria and proteinuria. c. Bacteriuria and increased specific gravity. Test Bank - Maternal Child Nursing Care by Perry (6th Edition, 2017) 761 d. Proteinuria and decreased specific gravity.

B

A child with cystic fibrosis is receiving recombinant human deoxyribonuclease (rhDNase). This drug: a. May cause mucus to thicken. b. May cause voice alterations. c. Is given subcutaneously. d. Is not indicated for children younger than 12 years.

B

A new patient and her partner arrive in the labor, delivery, recovery, and after birth unit for the birth of their first child. You apply the electronic fetal monitor (EFM) to the woman. Her partner asks you to explain what is printing on the graph, referring to the EFM strip. He wants to know what the baby's heart rate should be. Your best response is: a. "Don't worry about that machine; that's my job." b. "The top line graphs the baby's heart rate. Generally the heart rate is between 110 and 160. The heart rate will fluctuate in response to what is happening during labor." c. "The top line graphs the baby's heart rate, and the bottom line lets me know how strong the contractions are." d. "Your doctor will explain all of that later."

B

A nurse is teaching nursing students the physiology of congenital heart defects. Which defect results in decreased pulmonary blood flow? a. Atrial septal defect c. Ventricular septal defect b. Tetralogy of Fallot d. Patent ductus arteriosus

B

A school-age child with chronic renal failure is admitted to the hospital with a serum potassium level of 5.2 mEq/L. Which prescribed medication should the nurse plan to administer? a. Spironolactone (Aldactone) b. Sodium polystyrene sulfonate (Kayexalate) c. Lactulose (Cephulac) d. Calcium carbonate (Calcitab)

B

An 8-month-old infant has a hypercyanotic spell while blood is being drawn. The nurse's first action should be to: a. Assess for neurologic defects. b. Place the child in the knee-chest position. c. Begin cardiopulmonary resuscitation. d. Prepare the family for imminent death.

B

An adolescent being seen by the nurse practitioner for a sports physical is identified as having hypertension. On further testing, it is discovered the child has a cardiac abnormality. The initial treatment of secondary hypertension initially involves: a. Weight control and diet. b. Treating the underlying disease. c. Administration of digoxin. d. Administration of β-adrenergic receptor blockers.

B

An objective of care for the child with nephrosis is to: a. Reduce blood pressure. b. Reduce excretion of urinary protein. c. Increase excretion of urinary protein. d. Increase ability of tissues to retain fluid

B

As part of the treatment for congestive heart failure, the child takes the diuretic furosemide. As part of teaching home care, the nurse encourages the family to give the child foods such as bananas, oranges, and leafy vegetables. These foods are recommended because they are high in: a. Chlorides. c. Sodium. b. Potassium. d. Vitamins.

B

Asthma in infants is usually triggered by: a. Medications. c. Exposure to cold air. b. A viral infection. d. Allergy to dust or dust mites.

B

Calcium carbonate is given with meals to a child with chronic renal disease. The purpose of this is to: a. Prevent vomiting. b. Bind phosphorus. c. Stimulate appetite. d. Increase absorption of fat-soluble vitamins

B

Cardiopulmonary resuscitation is begun on a toddler. Which pulse is usually palpated because it is the most central and accessible? a. Radial c. Femoral b. Carotid d. Brachial

B

During labor a fetus with an average heart rate of 135 beats/min over a 10-minute period would be considered to have: a. bradycardia. b. a normal baseline heart rate. c. tachycardia. d. hypoxia.

B

One of the clinical manifestations of chronic renal failure is uremic frost. What best describes this term? a. Deposits of urea crystals in urine b. Deposits of urea crystals on skin c. Overexcretion of blood urea nitrogen d. Inability of body to tolerate cold temperatures

B

One of the most frequent causes of hypovolemic shock in children is: a. Myocardial infarction. c. Anaphylaxis. b. Blood loss. d. Congenital heart disease.

B

The diet of a child with nephrosis usually includes: a. High protein. c. Low fat. b. Salt restriction. d. High carbohydrate

B

The leading cause of death after heart transplantation is: a. Infection. c. Cardiomyopathy. b. Rejection. d. Congestive heart failure.

B

The narrowing of preputial opening of foreskin is called: a. Chordee c. Epispadias b. Phimosis d. Hypospadias

B

The narrowing of the preputial opening of the foreskin is called: a. Chordee. c. Epispadias. b. Phimosis. d. Hypospadias

B

The nurse assessing a premature newborn infant auscultates a continuous machinery-like murmur. This finding is associated with which congenital heart defect? a. Pulmonary stenosis c. Ventricular septal defect b. Patent ductus arteriosus d. Coarctation of the aorta

B

The nurse encourages the mother of a toddler with acute laryngotracheobronchitis to stay at the bedside as much as possible. The nurse's rationale for this action is primarily that: a. Mothers of hospitalized toddlers often experience guilt. b. The mother's presence will reduce anxiety and ease the child's respiratory efforts. c. Separation from the mother is a major developmental threat at this age. d. The mother can provide constant observations of the child's respiratory efforts.

B

The nurse is assessing a child with acute epiglottitis. Examining the child's throat by using a tongue depressor might precipitate which symptom or condition? a. Inspiratory stridor c. Sore throat b. Complete obstruction d. Respiratory tract infection

B

The nurse is caring for a child with acute respiratory distress syndrome (ARDS) associated with sepsis. Nursing actions should include: a. Force fluids. c. Institute seizure precautions. b. Monitor pulse oximetry. d. Encourage a high-protein diet.

B

The nurse is caring for a child with carbon monoxide (CO) poisoning associated with smoke inhalation. What is essential in this child's care? a. Monitor pulse oximetry. b. Monitor arterial blood gases. c. Administer oxygen if respiratory distress develops. d. Administer oxygen if child's lips become bright, cherry red.

B

The nurse is caring for a child with persistent hypoxia secondary to a cardiac defect. The nurse recognizes that a risk of cerebrovascular accidents (strokes) exists. An important objective to decrease this risk is to: a. Minimize seizures. c. Promote cardiac output. b. Prevent dehydration. d. Reduce energy expenditure.

B

The nurse is caring for an infant whose cleft lip was repaired. Important aspects of this infant's postoperative care include: a. Arm restraints, postural drainage, and mouth irrigations. b. Cleansing of suture line, supine and side-lying positions, and arm restraints. c. Mouth irrigations, prone position, and cleansing of suture line. d. Supine and side-lying positions, postural drainage, and arm restraints.

B

The nurse is conducting teaching for an adolescent being discharged to home after a renal transplantation. The adolescent needs further teaching if which statement is made? a. I will report any fever to my primary health care provider. b. I am glad I only have to take the immunosuppressant medication for two weeks. Test Bank - Maternal Child Nursing Care by Perry (6th Edition, 2017) 769 c. I will observe my incision for any redness or swelling. d. I wont miss doing kidney dialysis every week

B

The nurse is evaluating a child who is taking digoxin for her cardiac condition. The nurse is cognizant that a common sign of digoxin toxicity is: a. Seizures. c. Bradypnea. b. Vomiting. d. Tachycardia.

B

The nurse is preparing an adolescent for discharge after a cardiac catheterization. Which statement by the adolescent would indicate a need for further teaching? a. "I should avoid tub baths but may shower." b. "I have to stay on strict bed rest for 3 days." c. "I should remove the pressure dressing the day after the procedure." d. "I may attend school but should avoid exercise for several days."

B

The nurse providing care for the laboring woman realizes that variable fetal heart rate (FHR) decelerations are caused by: a. altered fetal cerebral blood flow. b. umbilical cord compression. c. uteroplacental insufficiency. d. fetal hypoxemia.

B

The parent of a toddler calls the nurse, asking about croup. What is a distinguishing manifestation of spasmodic croup? a. Wheezing is heard audibly. c. It is bacterial in nature. b. It has a harsh, barky cough. d. The child has a high fever.

B

The primary nursing intervention necessary to prevent bacterial endocarditis is to: a. Institute measures to prevent dental procedures. b. Counsel parents of high risk children about prophylactic antibiotics. c. Observe children for complications such as embolism and heart failure. d. Encourage restricted mobility in susceptible children.

B

What correctly matches the type of deceleration with its likely cause? a. Early deceleration—umbilical cord compression b. Late deceleration—uteroplacental inefficiency c. Variable deceleration—head compression d. Prolonged deceleration—cause unknown

B

What is the appropriate priority nursing action for the infant with a CHD who has an increased respiratory rate, is sweating, and is not feeding well? a. Recheck the infant's blood pressure. c. Withhold oral feeding. b. Alert the physician. d. Increase the oxygen rate.

B

What three measures should the nurse implement to provide intrauterine resuscitation? Select the response that best indicates the priority of actions that should be taken. a. Call the provider, reposition the mother, and perform a vaginal examination. b. Reposition the mother, increase intravenous (IV) fluid, and provide oxygen via face mask. c. Administer oxygen to the mother, increase IV fluid, and notify the care provider. d. Perform a vaginal examination, reposition the mother, and provide oxygen via face mask.

B

When preparing a school-age child and the family for heart surgery, the nurse should consider: a. Not showing unfamiliar equipment. b. Letting child hear the sounds of an electrocardiograph monitor. c. Avoiding mentioning postoperative discomfort and interventions. d. Explaining that an endotracheal tube will not be needed if the surgery goes well.

B

Which clinical changes occur as a result of septic shock? a. Hypothermia c. Vasoconstriction b. Increased cardiac output d. Angioneurotic edema

B

Which consideration is the most important in managing tuberculosis (TB) in children? a. Skin testing annually c. Adequate nutrition b. Pharmacotherapy d. Adequate hydration

B

Which factor predisposes a child to urinary tract infections? a. Increased fluid intake b. Short urethra in young girls c. Prostatic secretions in males d. Frequent emptying of the bladder

B

Which statement best describes why children have fewer respiratory tract infections as they grow older? a. The amount of lymphoid tissue decreases. b. Repeated exposure to organisms causes increased immunity. c. Viral organisms are less prevalent in the population. d. Secondary infections rarely occur after viral illnesses.

B

Which statement is descriptive of renal transplantation in children? a. It is an acceptable means of treatment after age 10 years. b. It is preferred means of renal replacement therapy in children. c. Children can receive kidneys only from other children. d. The decision for transplantation is difficult since a relatively normal lifestyle is not possible

B

Which type of croup is always considered a medical emergency? a. Laryngitis c. Spasmodic croup b. Epiglottitis d. Laryngotracheobronchitis (LTB)

B

β-Adrenergic agonists and methylxanthines are often prescribed for a child with an asthma attack. What is their action? a. Liquefy secretions c. Reduce inflammation of the lungs b. Dilate the bronchioles d. Reduce infection

B

The patient that you are caring for has severe preeclampsia and is receiving a magnesium sulfate infusion. You become concerned after assessment when the woman exhibits: a. A sleepy, sedated affect. c. Deep tendon reflexes of 2. b. A respiratory rate of 10 breaths/min. d. Absent ankle clonus.

B A respiratory rate of 10 breaths/min indicates that the client is experiencing respiratory depression from magnesium toxicity. Because magnesium sulfate is a central nervous system depressant, the client will most likely become sedated when the infusion is initiated. Deep tendon reflexes of 2 and absent ankle clonus are normal findings.

What laboratory marker is indicative of disseminated intravascular coagulation (DIC)? a. Bleeding time of 10 minutes c. Thrombocytopenia b. Presence of fibrin split products d. Hyperfibrinogenemia

B Degradation of fibrin leads to the accumulation of fibrin split products in the blood. Bleeding time in DIC is normal. Low platelets may occur with but are not indicative of DIC because they may result from other coagulopathies. Hypofibrinogenemia would occur with DIC.

A patient with pregnancy-induced hypertension is admitted complaining of pounding headache, visual changes, and epigastric pain. Nursing care is based on the knowledge that these signs are an indication of: a. Anxiety due to hospitalization. b. Worsening disease and impending convulsion. c. Effects of magnesium sulfate. d. Gastrointestinal upset.

B Headache and visual disturbances are caused by increased cerebral edema. Epigastric pain indicates distention of the hepatic capsules and often warns that a convulsion is imminent. These are danger signs showing increased cerebral edema and impending convulsion and should be treated immediately. The patient has not been started on magnesium sulfate treatment yet. Also, these are not anticipated effects of the medication.

Which maternal condition always necessitates delivery by cesarean section? a. Partial abruptio placentae c. Ectopic pregnancy b. Total placenta previa d. Eclampsia

B In total placenta previa, the placenta completely covers the cervical os. The fetus would die if a vaginal delivery occurred. If the mother has stable vital signs and the fetus is alive, a vaginal delivery can be attempted in cases of partial abruptio placentae. If the fetus has died, a vaginal delivery is preferred. The most common ectopic pregnancy is a tubal pregnancy, which is usually detected and treated in the first trimester. Labor can be safely induced if the eclampsia is under control.

The nurse caring for a woman hospitalized for hyperemesis gravidarum should expect that initial treatment to involve: a. Corticosteroids to reduce inflammation. b. IV therapy to correct fluid and electrolyte imbalances. c. An antiemetic, such as pyridoxine, to control nausea and vomiting. d. Enteral nutrition to correct nutritional deficits.

B Initially, the woman who is unable to keep down clear liquids by mouth requires IV therapy for correction of fluid and electrolyte imbalances. Corticosteroids have been used successfully to treat refractory hyperemesis gravidarum; however, they are not the expected initial treatment for this disorder. Pyridoxine is vitamin B6, not an antiemetic. Promethazine, a common antiemetic, may be prescribed. In severe cases of hyperemesis gravidarum, enteral nutrition via a feeding tube may be necessary to correct maternal nutritional deprivation. This is not an initial treatment for this patient.

An abortion in which the fetus dies but is retained within the uterus is called a(n): a. Inevitable abortion c. Incomplete abortion b. Missed abortion d. Threatened abortion

B Missed abortion refers to retention of a dead fetus in the uterus. An inevitable abortion means that the cervix is dilating with the contractions. An incomplete abortion means that not all of the products of conception were expelled. With a threatened abortion the woman has cramping and bleeding but not cervical dilation.

The most prevalent clinical manifestation of abruptio placentae (as opposed to placenta previa) is: a. Bleeding. c. Uterine activity. b. Intense abdominal pain. d. Cramping.

B Pain is absent with placenta previa and may be agonizing with abruptio placentae. Bleeding may be present in varying degrees for both placental conditions. Uterine activity and cramping may be present with both placental conditions.

A 26-year-old pregnant woman, gravida 2, para 1-0-0-1 is 28 weeks pregnant when she experiences bright red, painless vaginal bleeding. On her arrival at the hospital, what would be an expected diagnostic procedure? a. Amniocentesis for fetal lung maturity b. Ultrasound for placental location c. Contraction stress test (CST) d. Internal fetal monitoring

B The presence of painless bleeding should always alert the health care team to the possibility of placenta previa. This can be confirmed through ultrasonography. Amniocentesis would not be performed on a woman who is experiencing bleeding. In the event of an imminent delivery, the fetus would be presumed to have immature lungs at this gestational age, and the mother would be given corticosteroids to aid in fetal lung maturity. A CST would not be performed at a preterm gestational age. Furthermore, bleeding would be a contraindication to this test. Internal fetal monitoring would be contraindicated in the presence of bleeding.

The perinatal nurse is giving discharge instructions to a woman after suction curettage secondary to a hydatidiform mole. The woman asks why she must take oral contraceptives for the next 12 months. The best response from the nurse would be: a. "If you get pregnant within 1 year, the chance of a successful pregnancy is very small. Therefore, if you desire a future pregnancy, it would be better for you to use the most reliable method of contraception available." b. "The major risk to you after a molar pregnancy is a type of cancer that can be diagnosed only by measuring the same hormone that your body produces during pregnancy. If you were to get pregnant, it would make the diagnosis of this cancer more difficult." c. "If you can avoid a pregnancy for the next year, the chance of developing a second molar pregnancy is rare. Therefore, to improve your chance of a successful pregnancy, it is better not to get pregnant at this time." d. "Oral contraceptives are the only form of birth control that will prevent a recurrence of a molar pregnancy."

B This is an accurate statement. -Human chorionic gonadotropin (hCG) levels will be drawn for 1 year to ensure that the mole is completely gone. There is an increased chance of developing choriocarcinoma after the development of a hydatidiform mole. The goal is to achieve a "zero" hCG level. If the woman were to become pregnant, it could obscure the presence of the potentially carcinogenic cells. Women should be instructed to use birth control for 1 year after treatment for a hydatidiform mole. The rationale for avoiding pregnancy for 1 year is to ensure that carcinogenic cells are not present. Any contraceptive method except an intrauterine device is acceptable.

A laboring woman with no known risk factors suddenly experiences spontaneous rupture of membranes (ROM). The fluid consists of bright red blood. Her contractions are consistent with her current stage of labor. There is no change in uterine resting tone. The fetal heart rate begins to decline rapidly after the ROM. The nurse should suspect the possibility of: a. Placenta previa. b. Vasa previa. c. Severe abruptio placentae. d. Disseminated intravascular coagulation (DIC).

B Vasa previa is the result of a velamentous insertion of the umbilical cord. The umbilical vessels are not surrounded by Wharton jelly and have no supportive tissue. They are at risk for laceration at any time, but laceration occurs most frequently during ROM. The sudden appearance of bright red blood at the time of ROM and a sudden change in the fetal heart rate without other known risk factors should immediately alert the nurse to the possibility of vasa previa. The presence of placenta previa most likely would be ascertained before labor and would be considered a risk factor for this pregnancy. In addition, if the woman had a placenta previa, it is unlikely that she would be allowed to pursue labor and a vaginal birth. With the presence of severe abruptio placentae, the uterine tonicity would typically be tetanus (i.e., a boardlike uterus). DIC is a pathologic form of diffuse clotting that consumes large amounts of clotting factors and causes widespread external bleeding, internal bleeding, or both. DIC is always a secondary diagnosis, often associated with obstetric risk factors such as HELLP syndrome. This woman did not have any prior risk factors.

Women with hyperemesis gravidarum: a. Are a majority, because 80% of all pregnant women suffer from it at some time. b. Have vomiting severe and persistent enough to cause weight loss, dehydration, and electrolyte imbalance. c. Need intravenous (IV) fluid and nutrition for most of their pregnancy. d. Often inspire similar, milder symptoms in their male partners and mothers.

B Women with hyperemesis gravidarum have severe vomiting; however, treatment for several days sets things right in most cases. Although 80% of pregnant women experience nausea and vomiting, fewer than 1% (0.5%) proceed to this severe level. IV administration may be used at first to restore fluid levels, but it is seldom needed for very long. Women suffering from this condition want sympathy because some authorities believe that difficult relationships with mothers and/or partners may be the cause.

An infant with an unrepaired tetralogy of Fallot defect is becoming extremely cyanotic during a routine blood draw. Which interventions should the nurse implement? Place in order from the highest-priority intervention to the lowest-priority intervention. a. Administer 100% oxygen by blow-by. b. Place infant in knee-chest position. c. Remain calm. d. Give morphine subcutaneously or by an existing intravenous line. First priority Second priority Third priority Fourth priority

B A D C

*The nurse, caring for an infant whose cleft lip was repaired, should include which interventions into the infant's postoperative plan of care? (SATA) a. Postural drainage b. Petroleum jelly to the suture line c. Elbow restraints d. Supine and side-lying positions e. Mouth irrigations

B C

*Which statements regarding hepatitis B are correct (Select all that apply)? a. Hepatitis B cannot exist in a carrier state. b. Hepatitis B can be prevented by hepatitis B virus vaccine. c. Hepatitis B can be transferred to an infant of a breastfeeding mother. d. The onset of hepatitis B is insidious. e. Immunity to hepatitis B occurs after one attack.

B C D E

*A mother who intended to breastfeed has given birth to an infant with a cleft palate. Nursing interventions should include (Select all that apply): a. Giving medication to suppress lactation. b. Encouraging and helping mother to breastfeed. c. Teaching mother to feed breast milk by gavage. d. Recommending use of a breast pump to maintain lactation until infant can suck.

B D

*The nurse is preparing to care for an infant returning from pyloromyotomy surgery. Which prescribed orders should the nurse anticipate implementing (Select all that apply)? a. Nothing by mouth for 24 hours b. Administration of analgesics for pain c. Ice bag to the incisional area d. Intravenous (IV) fluids continued until tolerating fluids by mouth e. Clear liquids as the first feeding

B D E

A school-age child has been admitted to the hospital with an exacerbation of nephrotic syndrome. Which clinical manifestations should the nurse expect to assess (Select all that apply)? a. Weight loss b. Facial edema c. Cloudy, smoky browncolored urine d. Fatigue e. Frothy-appearing urine

B D E

A tiered system of categorizing FHR has been recommended by regulatory agencies. Nurses, midwives, and physicians who care for women in labor must have a working knowledge of fetal monitoring standards and understand the significance of each category. These categories include: (Select all that apply.) a. reassuring. b. Category I. c. Category II. d. nonreassuring. e. Category III. BCE

BCE

*A histamine receptor antagonist such as cimetidine (Tagamet) or ranitidine (Zantac) is ordered for an infant with gastroesophageal reflux. The purpose of this is to: a. Prevent reflux. b. Prevent hematemesis. c. Reduce gastric acid production. d. Increase gastric acid production.

C

*Constipation has recently become a problem for a school-age girl. She is healthy except for seasonal allergies, which are now being successfully treated with antihistamines. The nurse should suspect that the constipation is most likely caused by: a. Diet. c. Antihistamines. b. Allergies. d. Emotional factors.

C

*The nurse is caring for an infant with suspected pyloric stenosis. Which clinical manifestation would indicate pyloric stenosis? a. Abdominal rigidity and pain on palpation b. Rounded abdomen and hypoactive bowel sounds c. Visible peristalsis and weight loss d. Distention of lower abdomen and constipation

C

*The viral pathogen that frequently causes acute diarrhea in young children is: a. Giardia organisms. b. Shigella organisms. c. Rotavirus. d. Salmonella organisms.

C

*Therapeutic management of the child with acute diarrhea and dehydration usually begins with: a. Clear liquids. b. Adsorbents such as kaolin and pectin. c. Oral rehydration solution (ORS). d. Antidiarrheal medications such as paregoric.

C

*What is the major focus of the therapeutic management for a child with lactose intolerance? a. Compliance with the medication regimen b. Providing emotional support to family members c. Teaching dietary modifications d. Administration of daily normal saline enemas

C

*What is used to treat moderate-to-severe inflammatory bowel disease? a. Antacids b. Antibiotics c. Corticosteroids d. Antidiarrheal medications

C

*When caring for a child with probable appendicitis, the nurse should be alert to recognize that a sign of perforation is: a. Bradycardia. b. Anorexia c. Sudden relief from pain. d. Decreased abdominal distention.

C

*Which description of a stool is characteristic of intussusception? a. Ribbon-like stools b. Hard stools positive for guaiac c. "Currant jelly" stools d. Loose, foul-smelling stools

C

*Which statement is most descriptive of Meckel's diverticulum? a. It is more common in females than in males. b. It is acquired during childhood. c. Intestinal bleeding may be mild or profuse. d. Medical interventions are usually sufficient to treat the problem.

C

14. Which signs and symptoms are associated with Werdnig-Hoffmann disease? a. Spinal muscular atrophy b. Neural atrophy of muscles c. Progressive weakness and wasting of skeletal muscle d. Pseudohypertrophy of certain muscle groups

C

5. A current recommendation to prevent neural tube defects is the administration of what supplement? a. Vitamin A throughout pregnancy b. Multivitamin preparations as soon as pregnancy is suspected c. Folic acid for all women of childbearing age d. Folic acid during the first and second trimesters of pregnancy

C

A child is brought to the emergency department experiencing an anaphylactic reaction to a bee sting. While an airway is being established, what medication should the nurse prepare for immediate administration? a. Diphenhydramine (Benadryl) c. Epinephrine b. Dopamine d. Calcium chloride

C

A child is diagnosed with influenza, probably type A disease. Management includes: a. Clear liquid diet for hydration. b. Aspirin to control fever. c. Amantadine hydrochloride to reduce symptoms. d. Antibiotics to prevent bacterial infection.

C

A clinical manifestation of the systemic venous congestion that can occur with congestive heart failure is: a. Tachypnea. c. Peripheral edema. b. Tachycardia. d. Pale, cool extremities.

C

A major complication in a child with chronic renal failure is: a. Hypokalemia. b. Metabolic alkalosis. c. Water and sodium retention. d. Excessive excretion of blood urea nitrogen

C

A nurse is teaching an adolescent about primary hypertension. The nurse knows that which of the following is correct? a. Primary hypertension should be treated with diuretics as soon as it is detected. b. Congenital heart defects are the most common cause of primary hypertension. c. Primary hypertension may be treated with weight reduction. d. Primary hypertension is not affected by exercise.

C

A parent whose two school-age children have asthma asks the nurse in what sports, if any, they can participate. The nurse should recommend: a. Soccer. c. Swimming. b. Running. d. Basketball.

C

A preschool child is scheduled for an echocardiogram. Parents ask the nurse whether they can hold the child during the procedure. The nurse should answer with which response? a. "You will be able to hold your child during the procedure." b. "Your child can be active during the procedure, but can't sit in your lap." c. "Your child must lie quietly; sometimes a mild sedative is administered before the procedure." d. "The procedure is invasive so your child will be restrained during the echocardiogram."

C

According to standard professional thinking, nurses should auscultate the fetal heart rate (FHR): a. every 15 minutes in the active phase of the first stage of labor in the absence of risk factors. b. every 20 minutes in the second stage, regardless of whether risk factors are present. c. before and after ambulation and rupture of membranes. d. more often in a woman's first pregnancy.

C

An appropriate nursing intervention when caring for an unconscious child should be to: a. Change the child's position infrequently to minimize the chance of increased intracranial pressure (ICP). b. Avoid using narcotics or sedatives to provide comfort and pain relief. c. Monitor fluid intake and output carefully to avoid fluid overload and cerebral edema. d. Give tepid sponge baths to reduce fever because antipyretics are contraindicated.

C

An important nursing consideration when chest tubes will be removed from a child is to: a. Explain that it is not painful. b. Explain that only a Band-Aid will be needed. c. Administer analgesics before the procedure. d. Expect bright red drainage for several hours after removal.

C

An important nursing consideration when suctioning a young child who has had heart surgery is to: a. Perform suctioning at least every hour. b. Suction for no longer than 30 seconds at a time. c. Administer supplemental oxygen before and after suctioning. d. Expect symptoms of respiratory distress when suctioning.

C

During the first few days after surgery for cleft lip, which intervention should the nurse do? a. Leave infant in crib at all times to prevent suture strain. b. Keep infant heavily sedated to prevent suture strain. c. Remove restraints periodically to cuddle infant. d. Alternate position from prone to side-lying to supine.

C

It is generally recommended that a child with acute streptococcal pharyngitis can return to school: a. When the sore throat is better. c. After taking antibiotics for 24 hours. b. If no complications develop. d. After taking antibiotics for 3 days.

C

It is now recommended that children with asthma who are taking long-term inhaled steroids should be assessed frequently because they may develop: a. Cough. c. Slowed growth. b. Osteoporosis. d. Cushing's syndrome.

C

The diet of a child with chronic renal failure is usually characterized as: a. High in protein. b. Low in vitamin D. c. Low in phosphorus. Test Bank - Maternal Child Nursing Care by Perry (6th Edition, 2017) 765 d. Supplemented with vitamins A, E, and K.

C

The most appropriate nursing diagnosis for the child with acute glomerulonephritis is: a. Risk for Injury related to malignant process and treatment. b. Deficient Fluid Volume related to excessive losses. c. Excess Fluid Volume related to decreased plasma filtration. d. Excess Fluid Volume related to fluid accumulation in tissues and third spaces

C

The nurse caring for the woman in labor should understand that increased variability of the fetal heart rate may be caused by: a. narcotics. b. barbiturates. c. methamphetamines. d. tranquilizers.

C

The nurse caring for the woman in labor should understand that maternal hypotension can result in: a. early decelerations. b. fetal dysrhythmias. c. uteroplacental insufficiency. d. spontaneous rupture of membranes.

C

The nurse is admitting a school-age child in acute renal failure with reduced glomerular filtration rate. Which urine test is the most useful clinical indication of glomerular filtration rate? a. pH c. Creatinine clearance b. Osmolality d. Protein level

C

The nurse is admitting a young child to the hospital because bacterial meningitis is suspected. The PRIORITY of nursing care is to: A. initiate isolation precautions as soon as the diagnosis is confirmed. B. initiate isolation precautions as soon as the causative agent is identified. C. administer antibiotic therapy as soon as it is ordered. D. administer sedatives/analgesics on a preventive schedule to manage pain.

C

The nurse is caring for a child after heart surgery. What should she or he do if evidence is found of cardiac tamponade? a. Increase analgesia. b. Apply warming blankets. c. Immediately report this to the physician. d. Encourage the child to cough, turn, and breathe deeply.

C

The nurse is teaching parents of a child with chronic renal failure (CRF) about the use of recombinant human erythropoietin (rHuEPO) subcutaneous injections. Which statement indicates the parents have understood the teaching? a. These injections will help with the hypertension. b. Were glad the injections only need to be given once a month. c. The red blood cell count should begin to improve with these injections. d. Urine output should begin to improve with these injections

C

The nurse is teaching the parent about the diet of a child experiencing severe edema associated with acute glomerulonephritis. Which information should the nurse include in the teaching? a. You will need to decrease the number of calories in your childs diet. b. Your childs diet will need an increased amount of protein. c. You will need to avoid adding salt to your childs food. d. Your childs diet will consist of low-fat, low-carbohydrate foods

C

The nurse providing care for the laboring woman should understand that late fetal heart rate (FHR) decelerations are the result of: a. altered cerebral blood flow. b. umbilical cord compression. c. uteroplacental insufficiency. d. meconium fluid.

C

What is an advantage of external electronic fetal monitoring? a. The ultrasound transducer can accurately measure short-term variability and beat-to-beat changes in the fetal heart rate. b. The tocotransducer can measure and record the frequency, regularity, intensity, and approximate duration of uterine contractions (UCs). c. The tocotransducer is especially valuable for measuring uterine activity during the first stage of labor. d. Once correctly applied by the nurse, the transducer need not be repositioned even when the woman changes positions.

C

What is best described as the inability of the heart to pump an adequate amount of blood to the systemic circulation at normal filling pressures? a. Pulmonary congestion c. Congestive heart failure b. Congenital heart defect d. Systemic venous congestion

C

When assessing a child for possible congenital heart defects (CHDs), where should the nurse measure blood pressure? a. The right arm c. All four extremities b. The left arm d. Both arms while the child is crying

C

Which action by the school nurse is important in the prevention of rheumatic fever? a. Encourage routine cholesterol screenings. b. Conduct routine blood pressure screenings. c. Refer children with sore throats for throat cultures. d. Recommend salicylates instead of acetaminophen for minor discomforts.

C

Which defect results in increased pulmonary blood flow? a. Pulmonic stenosis c. Atrial septal defect b. Tricuspid atresia d. Transposition of the great arteries

C

Which intervention for treating croup at home should be taught to parents? a. Have a decongestant available to give the child when an attack occurs. b. Have the child sleep in a dry room. c. Take the child outside. d. Give the child an antibiotic at bedtime.

C

A woman presents to the emergency department with complaints of bleeding and cramping. The initial nursing history is significant for a last menstrual period 6 weeks ago. On sterile speculum examination, the primary care provider finds that the cervix is closed. The anticipated plan of care for this woman would be based on a probable diagnosis of which type of spontaneous abortion? a. Incomplete c. Threatened b. Inevitable d. Septic

C A woman with a threatened abortion presents with spotting, mild cramps, and no cervical dilation. A woman with an incomplete abortion would present with heavy bleeding, mild to severe cramping, and cervical dilation. An inevitable abortion manifests with the same symptoms as an incomplete abortion: heavy bleeding, mild to severe cramping, and cervical dilation. A woman with a septic abortion presents with malodorous bleeding and typically a dilated cervix.

What condition indicates concealed hemorrhage when the patient experiences an abruptio placentae? a. Decrease in abdominal pain c. Hard, boardlike abdomen b. Bradycardia d. Decrease in fundal height

C Concealed hemorrhage occurs when the edges of the placenta do not separate. The formation of a hematoma behind the placenta and subsequent infiltration of the blood into the uterine muscle results in a very firm, boardlike abdomen. Abdominal pain may increase. The patient will have shock symptoms that include tachycardia. As bleeding occurs, the fundal height will increase.

A woman arrives for evaluation of her symptoms, which include a missed period, adnexal fullness, tenderness, and dark red vaginal bleeding. On examination the nurse notices an ecchymotic blueness around the woman's umbilicus and recognizes this assessment finding as: a. Normal integumentary changes associated with pregnancy. b. Turner's sign associated with appendicitis. c. Cullen's sign associated with a ruptured ectopic pregnancy. d. Chadwick's sign associated with early pregnancy.

C Cullen's sign, the blue ecchymosis seen in the umbilical area, indicates hematoperitoneum associated with an undiagnosed ruptured intraabdominal ectopic pregnancy. Linea nigra on the abdomen is the normal integumentary change associated with pregnancy. It manifests as a brown, pigmented, vertical line on the lower abdomen. Turner's sign is ecchymosis in the flank area, often associated with pancreatitis. Chadwick's sign is the blue-purple color of the cervix that may be seen during or around the eighth week of pregnancy.

The labor of a pregnant woman with preeclampsia is going to be induced. Before initiating the Pitocin infusion, the nurse reviews the woman's latest laboratory test findings, which reveal a platelet count of 90,000, an elevated aspartate transaminase (AST) level, and a falling hematocrit. The nurse notifies the physician because the laboratory results are indicative of: a. Eclampsia. b. Disseminated intravascular coagulation (DIC). c. HELLP syndrome. d. Idiopathic thrombocytopenia.

C HELLP syndrome is a laboratory diagnosis for a variant of severe preeclampsia that involves hepatic dysfunction characterized by hemolysis (H), elevated liver enzymes (EL), and low platelets (LP). Eclampsia is determined by the presence of seizures. DIC is a potential complication associated with HELLP syndrome. Idiopathic thrombocytopenia is the presence of low platelets of unknown cause and is not associated with preeclampsia.

In providing nutritional counseling for the pregnant woman experiencing cholecystitis, the nurse would: a. Assess the woman's dietary history for adequate calories and proteins. b. Instruct the woman that the bulk of calories should come from proteins. c. Instruct the woman to eat a low-fat diet and avoid fried foods. d. Instruct the woman to eat a low-cholesterol, low-salt diet.

C Instructing the woman to eat a low-fat diet and avoid fried foods is appropriate nutritional counseling for this client. Caloric and protein intake do not predispose a woman to the development of cholecystitis. The woman should be instructed to limit protein intake and choose foods that are high in carbohydrates. A low-cholesterol diet may be the result of limiting fats. However, a low-salt diet is not indicated.

Magnesium sulfate is given to women with preeclampsia and eclampsia to: a. Improve patellar reflexes and increase respiratory efficiency. b. Shorten the duration of labor. c. Prevent and treat convulsions. d. Prevent a boggy uterus and lessen lochial flow.

C Magnesium sulfate is the drug of choice to prevent convulsions, although it can generate other problems. Loss of patellar reflexes and respiratory depression are signs of magnesium toxicity. Magnesium sulfate can increase the duration of labor. Women are at risk for a boggy uterus and heavy lochial flow as a result of magnesium sulfate therapy.

Methotrexate is recommended as part of the treatment plan for which obstetric complication? a. Complete hydatidiform mole c. Unruptured ectopic pregnancy b. Missed abortion d. Abruptio placentae

C Methotrexate is an effective, nonsurgical treatment option for a hemodynamically stable woman whose ectopic pregnancy is unruptured and less than 4 cm in diameter. Methotrexate is not indicated or recommended as a treatment option for complete hydatidiform mole, missed abortion, and abruptio placentae.

A primigravida is being monitored in her prenatal clinic for preeclampsia. What finding should concern her nurse? a. Blood pressure (BP) increase to 138/86 mm Hg b. Weight gain of 0.5 kg during the past 2 weeks c. A dipstick value of 3+ for protein in her urine d. Pitting pedal edema at the end of the day

C Proteinuria is defined as a concentration of 1+ or greater via dipstick measurement. A dipstick value of 3+ should alert the nurse that additional testing or assessment should be made. Generally, hypertension is defined as a BP of 140/90 or an increase in systolic pressure of 30 mm Hg or in diastolic pressure of 15 mm Hg. Preeclampsia may be manifested as a rapid weight gain of more than 2 kg in 1 week. Edema occurs in many normal pregnancies and in women with preeclampsia. Therefore, the presence of edema is no longer considered diagnostic of preeclampsia.

Which condition would not be classified as a bleeding disorder in late pregnancy? a. Placenta previa. c. Spontaneous abortion. b. Abruptio placentae. d. Cord insertion.

C Spontaneous abortion is another name for miscarriage; by definition it occurs early in pregnancy. Placenta previa is a cause of bleeding disorders in later pregnancy. Abruptio placentae is a cause of bleeding disorders in later pregnancy. Cord insertion is a cause of bleeding disorders in later pregnancy.

Nurses should be aware that HELLP syndrome: a. Is a mild form of preeclampsia. b. Can be diagnosed by a nurse alert to its symptoms. c. Is characterized by hemolysis, elevated liver enzymes, and low platelets. d. Is associated with preterm labor but not perinatal mortality.

C The acronym HELLP stands for hemolysis (H), elevated liver enzymes (EL), and low platelets (LP). HELLP syndrome is a variant of severe preeclampsia. HELLP syndrome is difficult to identify because the symptoms often are not obvious. It must be diagnosed in the laboratory. Preterm labor is greatly increased, and so is perinatal mortality.

A pregnant woman has been receiving a magnesium sulfate infusion for treatment of severe preeclampsia for 24 hours. On assessment the nurse finds the following vital signs: temperature of 37.3° C, pulse rate of 88 beats/min, respiratory rate of 10 breaths/min, blood pressure (BP) of 148/90 mm Hg, absent deep tendon reflexes, and no ankle clonus. The client complains, "I'm so thirsty and warm." The nurse: a. Calls for a stat magnesium sulfate level. b. Administers oxygen. c. Discontinues the magnesium sulfate infusion. d. Prepares to administer hydralazine.

C The client is displaying clinical signs and symptoms of magnesium toxicity. Magnesium should be discontinued immediately. In addition, calcium gluconate, the antidote for magnesium, may be administered. Hydralazine is an antihypertensive commonly used to treat hypertension in severe preeclampsia. Typically it is administered for a systolic BP greater than 160 mm Hg or a diastolic BP greater than 110 mm Hg.

What finding on a prenatal visit at 10 weeks could suggest a hydatidiform mole? a. Complaint of frequent mild nausea b. Blood pressure of 120/80 mm Hg c. Fundal height measurement of 18 cm d. History of bright red spotting for 1 day, weeks ago

C The uterus in a hydatidiform molar pregnancy is often larger than would be expected on the basis of the duration of the pregnancy. Nausea increases in a molar pregnancy because of the increased production of hCG. A woman with a molar pregnancy may have early-onset pregnancy-induced hypertension. In the patient's history, bleeding is normally described as brownish.

Preeclampsia is a unique disease process related only to human pregnancy. The exact cause of this condition continues to elude researchers. The American College of Obstetricians and Gynecologists has developed a comprehensive list of risk factors associated with the development of preeclampsia. Which client exhibits the greatest number of these risk factors? a. A 30-year-old obese Caucasian with her third pregnancy b. A 41-year-old Caucasian primigravida c. An African-American client who is 19 years old and pregnant with twins d. A 25-year-old Asian-American whose pregnancy is the result of donor insemination

C Three risk factors are present for this woman. She is of African-American ethnicity, is at the young end of the age distribution, and has a multiple pregnancy. In planning care for this client the nurse must monitor blood pressure frequently and teach the woman regarding early warning signs. The 30-year-old client only has one known risk factor, obesity. Age distribution appears to be U-shaped, with women less than 20 years and more than 40 years being at greatest risk. Preeclampsia continues to be seen more frequently in primigravidas; this client is a multigravida woman. Two risk factors are present for the 41-year-old client. Her age and status as a primigravida put her at increased risk for preeclampsia. Caucasian women are at a lower risk than African-American women. The Asian-American client exhibits only one risk factor. Pregnancies that result from donor insemination, oocyte donation, and embryo donation are at an increased risk of developing preeclampsia.

*A nurse is conducting dietary teaching on high-fiber foods for parents of a child with constipation. Which foods should the nurse include as being high in fiber (Select all that apply)? a. White rice b. Avocados c. Whole grain breads d. Bran pancakes e. Raw carrots

C D E

A nurse is conducting discharge teaching to parents about the care of their infant after cardiac surgery. The nurse instructs the parents to notify the physician if what conditions occur (Select all that apply)? a. Respiratory rate of 36 at rest b. Appetite slowly increasing c. Temperature above 37.7° C (100° F) d. New, frequent coughing e. Turning blue or bluer than normal

C D E

The nurse is caring for a 10-month-old infant with respiratory syncytial virus (RSV) bronchiolitis. Which interventions should be included in the child's care (Select all that apply)? a. Administer antibiotics. b. Administer cough syrup. c. Encourage infant to drink 8 ounces of formula every 4 hours. d. Institute cluster care to encourage adequate rest. e. Place on noninvasive oxygen monitoring.

C D E

The nurse is conducting discharge teaching about signs and symptoms of heart failure to parents of an infant with a repaired tetralogy of Fallot. Which signs and symptoms should the nurse include (Select all that apply)? a. Warm flushed extremities b. Weight loss c. Decreased urinary output d. Sweating (inappropriate) e. Fatigue

C D E

Which clinical manifestations would the nurse expect to see as shock progresses in a child and becomes decompensated shock (Select all that apply)? a. Thirst and diminished urinary output b. Irritability and apprehension c. Cool extremities and decreased skin turgor d. Confusion and somnolence e. Normal blood pressure and narrowing pulse pressure f. Tachypnea and poor capillary refill time

C D F

Nursing interventions for the child after a cardiac catheterization include which of the following (Select all that apply)? a. Allow ambulation as tolerated. b. Monitor vital signs every 2 hours. c. Assess the affected extremity for temperature and color. d. Check pulses above the catheterization site for equality and symmetry. e. Remove pressure dressing after 4 hours. f. Maintain a patent peripheral intravenous catheter until discharge.

C F

The baseline fetal heart rate (FHR) is the average rate during a 10-minute segment. Changes in FHR are categorized as periodic or episodic. These patterns include both accelerations and decelerations. The labor nurse is evaluating the patient's most recent 10-minute segment on the monitor strip and notes a late deceleration. This is likely to be caused by which physiologic alteration? (Select all that apply.) a. Spontaneous fetal movement b. Compression of the fetal head c. Placental abruption d. Cord around the baby's neck e. Maternal supine hypotension CE

CE

Step 3

Cleanse the perineum and vulva if necessary.

C D E

Clinical manifestations of increased intracranial pressure (ICP) in infants are (Select all that apply): a. Low-pitched cry. b. Sunken fontanel. c. Diplopia and blurred vision. d. Irritability. e. Distended scalp veins. f. Increased blood pressure.

*A stool specimen from a child with diarrhea shows the presence of neutrophils and red blood cells. This is most suggestive of which condition? a. Protein intolerance b. Fat malabsorption c. Parasitic infection d. Bacterial gastroenteritis

D

*An infant with pyloric stenosis experiences excessive vomiting that can result in: a. Hyperchloremia. b. Hypernatremia. c. Metabolic acidosis. d. Metabolic alkalosis.

D

*Careful hand washing before and after contact can prevent the spread of which condition in day care and school settings? a. Irritable bowel syndrome b. Hepatic cirrhosis c. Ulcerative colitis d. Hepatitis A

D

*Therapeutic management of most children with Hirschsprung's disease is primarily: a. Daily enemas. b. Low-fiber diet. c. Permanent colostomy. d. Surgical removal of affected section of bowel.

D

*What should the nurse stress in a teaching plan for the mother of an 11-year-old boy with ulcerative colitis? a. Preventing the spread of illness to others b. Nutritional guidance and preventing constipation c. Teaching daily use of enemas d. Coping with stress and avoiding triggers

D

*Which clinical manifestation would most suggest acute appendicitis? a. Rebound tenderness b. Bright red or dark red rectal bleeding c. Abdominal pain that is relieved by eating d. Abdominal pain that is most intense at McBurney's point

D

*Which type of dehydration results from water loss in excess of electrolyte loss? a. Isotonic dehydration b. Hypotonic dehydration c. Isosmotic dehydration d. Hypertonic dehydration

D

*Which type of hernia has an impaired blood supply to the herniated organ? a. Hiatal hernia b. Omphalocele c. Incarcerated hernia d. Strangulated hernia

D

. The nurse is assisting the pediatric provider with a newborn examination. The provider notes that the infant has hypospadias. The nurse understands that hypospadias refers to: a. Absence of a urethral opening. b. Penis shorter than usual for age. c. Urethral opening along dorsal surface of penis. d. Urethral opening along ventral surface of penis.

D

12. What is the most appropriate nursing response to the father of a newborn infant with myelomeningocele who asks about the cause of this condition? a. "One of the parents carries a defective gene that causes myelomeningocele." b. "A deficiency in folic acid in the father is the most likely cause." c. "Offspring of parents who have a spinal abnormality are at greater risk for myelomeningocele." d. "There may be a variety of different causes."

D

13. Which statement best describes a myelomeningocele? a. Fissure in the spinal column that leaves the meninges and the spinal cord exposed. b. Herniation of the brain and meninges through a defect in the skull. c. Hernial protrusion of a sac-like cyst of meninges with spinal fluid but no neural elements. d. Visible defect with an external sac-like protrusion containing meninges, spinal fluid, and nerves.

D

17. The nurse is caring for an intubated infant with botulism. Which health care provider prescriptions should the nurse clarify with the health care provider before implementing? a. Administer 250 mg botulism immune globulin intravenously (BIG-IV) one time. b. Provide total parenteral nutrition (TPN) at 25 mL/hr intravenously. c. Titrate oxygen to keep pulse oximetry saturations greater than 92. d. Administer gentamicin sulfate 10 mg per intravenous piggyback every 12 hours.

D

2. The parents of a child diagnosed with cerebral palsy ask the nurse if any drugs can decrease their child's spasticity. The nurse's response should be based on what knowledge? a. Anticonvulsant medications are sometimes useful for controlling spasticity. b. Medications that would be useful in reducing spasticity are too toxic for use with children. c. Many different medications can be highly effective in controlling spasticity. d. Implantation of a pump to deliver medication into the intrathecal space to decrease spasticity has recently become available.

D

8. The nurse is talking to a parent with a child who has a latex allergy. Which statement by the parent would indicate a correct understanding of the teaching? a. "My child will have an allergic reaction if he comes in contact with yeast products." b. "My child may have an upset stomach if he eats a food made with wheat or barley." c. "My child will probably develop an allergy to peanuts." d. "My child should not eat bananas or kiwis."

D

A child has had cold symptoms for more than 2 weeks, a headache, nasal congestion with purulent nasal drainage, facial tenderness, and a cough that increases during sleep. The nurse recognizes that these symptoms are characteristic of which respiratory condition? a. Allergic rhinitis c. Asthma b. Bronchitis d. Sinusitis

D

A common side effect of corticosteroid therapy is: a. Fever. c. Weight loss. b. Hypertension. d. Increased appetite

D

A common, serious complication of rheumatic fever is: a. Seizures. c. Pulmonary hypertension. b. Cardiac arrhythmias. d. Cardiac valve damage.

D

A mother asks the nurse what would be the first indication that acute glomerulonephritis is improving. The nurses best response should be that the: a. Blood pressure will stabilize. c. Urine will be free of protein. b. Child will have more energy. d. Urinary output will increase

D

A nurse may be called on to stimulate the fetal scalp: a. as part of fetal scalp blood sampling. b. in response to tocolysis. c. in preparation for fetal oxygen saturation monitoring. d. to elicit an acceleration in the fetal heart rate (FHR).

D

A preschool child is being admitted to the hospital with dehydration and a urinary tract infection (UTI). Which urinalysis result should the nurse expect with these conditions? a. WBC <1; specific gravity 1.008 c. WBC >2; specific gravity 1.016 b. WBC <2; specific gravity 1.025 d. WBC >2; specific gravity 1.030

D

An advantage of peritoneal dialysis is that: a. Treatments are done in hospitals. b. Protein loss is less extensive. c. Dietary limitations are not necessary. d. Parents and older children can perform treatments.

D

As a perinatal nurse you realize that a fetal heart rate that is tachycardic, is bradycardic, or has late decelerations or loss of variability is nonreassuring and is associated with: a. hypotension. b. cord compression. c. maternal drug use. d. hypoxemia.

D

Caring for the newborn with a cleft lip and palate before surgical repair includes: a. Gastrostomy feedings. b. Keeping the infant in near-horizontal position during feedings. c. Allowing little or no sucking. d. Providing satisfaction of sucking needs.

D

Chronic otitis media with effusion (OME) is differentiated from acute otitis media (AOM) because it is usually characterized by: a. Fever as high as 40° C (104° F). c. Nausea and vomiting. b. Severe pain in the ear. d. A feeling of fullness in the ear.

D

Cystic fibrosis (CF) is suspected in a toddler. Which test is essential in establishing this diagnosis? a. Bronchoscopy c. Urine creatinine b. Serum calcium d. Sweat chloride test

D

For what reason might a newborn infant with a cardiac defect, such as coarctation of the aorta, that results in a right-to-left shunt receive prostaglandin E1? a. To decrease inflammation c. To decrease respirations b. To control pain d. To improve oxygenation

D

José is a 4-year-old child scheduled for a cardiac catheterization. Preoperative teaching should be: a. Directed at his parents because he is too young to understand. b. Detailed in regard to the actual procedures so he will know what to expect. c. Done several days before the procedure so that he will be prepared. d. Adapted to his level of development so that he can understand.

D

Pancreatic enzymes are administered to the child with cystic fibrosis. Nursing considerations should include: a. Do not administer pancreatic enzymes if the child is receiving antibiotics. b. Decrease dose of pancreatic enzymes if the child is having frequent, bulky stools. c. Administer pancreatic enzymes between meals if at all possible. d. Pancreatic enzymes can be swallowed whole or sprinkled on a small amount of food taken at the beginning of a meal.

D

Parents have understood teaching about prevention of childhood otitis media if they make which statement? a. "We will only prop the bottle during the daytime feedings." b. "Breastfeeding will be discontinued after 4 months of age." c. "We will place the child flat right after feedings." d. "We will be sure to keep immunizations up to date."

D

Seventy-two hours after cardiac surgery, a young child has a temperature of 37.7° C (101° F). The nurse should: a. Keep the child warm with blankets. b. Apply a hypothermia blanket. c. Record the temperature on nurses' notes. d. Report findings to physician.

D

Skin testing for tuberculosis (the Mantoux test) is recommended: a. Every year for all children older than 2 years. b. Every year for all children older than 10 years. c. Every 2 years for all children starting at age 1 year. d. Periodically for children who reside in high-prevalence regions.

D

Surgical closure of the ductus arteriosus would: a. Stop the loss of unoxygenated blood to the systemic circulation. b. Decrease the edema in legs and feet. c. Increase the oxygenation of blood. d. Prevent the return of oxygenated blood to the lungs.

D

The most common causative agent of bacterial endocarditis is: a. Staphylococcus albus. c. Staphylococcus albicans. b. Streptococcus hemolyticus. d. Streptococcus viridans.

D

The most common cause of acute renal failure in children is: a. Pyelonephritis. c. Urinary tract obstruction. b. Tubular destruction. d. Severe dehydration

D

The most common cause of decreased variability in the fetal heart rate (FHR) that lasts 30 minutes or less is: a. altered cerebral blood flow. b. fetal hypoxemia. c. umbilical cord compression. d. fetal sleep cycles.

D

The mother of a toddler yells to the nurse, "Help! He is choking to death on his food." The nurse determines that lifesaving measures are necessary based on: a. Gagging. c. Pulse over 100 beats/min. b. Coughing. d. Inability to speak.

D

The nurse is caring for a child with acute renal failure. What clinical manifestation should he or she recognize as a sign of hyperkalemia? a. Dyspnea c. Oliguria b. Seizure d. Cardiac arrhythmia

D

The nurse is caring for a school-age girl who has had a cardiac catheterization. The child tells the nurse that her bandage is "too wet." The nurse finds the bandage and bed soaked with blood. The most appropriate initial nursing action is to: a. Notify the physician. b. Apply a new bandage with more pressure. c. Place the child in the Trendelenburg position. d. Apply direct pressure above the catheterization site.

D

The nurse is caring for an adolescent who has just started dialysis. The child seems always angry, hostile, or depressed. The nurse should recognize that this is most likely related to: a. Neurologic manifestations that occur with dialysis. b. Physiologic manifestations of renal disease. c. Adolescents having few coping mechanisms. d. Adolescents often resenting the control and enforced dependence imposed by dialysis

D

The nurse should recommend medical attention if a child with a slight head injury experiences: a. Sleepiness. b. Headache, even if slight. c. Vomiting, even once. d. Confusion or abnormal behavior.

D

The parent of an infant with nasopharyngitis should be instructed to notify the health care professional if the infant: a. Becomes fussy. c. Has a fever over 99° F. b. Has a cough. d. Shows signs of an earache.

D

The parents of a young child with congestive heart failure tell the nurse that they are "nervous" about giving digoxin. The nurse's response should be based on knowing that: a. It is a safe, frequently used drug. b. It is difficult to either overmedicate or undermedicate with digoxin. c. Parents lack the expertise necessary to administer digoxin. d. Parents must learn specific, important guidelines for administration of digoxin.

D

What is an expected assessment finding in a child with coarctation of the aorta? a. Orthostatic hypotension b. Systolic hypertension in the lower extremities c. Blood pressure higher on the left side of the body d. Disparity in blood pressure between the upper and lower extremities

D

What type of shock is characterized by a hypersensitivity reaction causing massive vasodilation and capillary leaks, which may occur with drug or latex allergy? a. Neurogenic shock c. Hypovolemic shock b. Cardiogenic shock d. Anaphylactic shock

D

When assessing the relative advantages and disadvantages of internal and external electronic fetal monitoring, nurses comprehend that both: a. can be used when membranes are intact. b. measure the frequency, duration, and intensity of uterine contractions. c. may need to rely on the woman to indicate when uterine activity (UA) is occurring. d. can be used during the antepartum and intrapartum periods.

D

When caring for an infant with an upper respiratory tract infection and elevated temperature, an appropriate nursing intervention is to: a. Give tepid water baths to reduce fever. b. Encourage food intake to maintain caloric needs. c. Have child wear heavy clothing to prevent chilling. d. Give small amounts of favorite fluids frequently to prevent dehydration.

D

When caring for the child with Kawasaki disease, the nurse should understand that: a. The child's fever is usually responsive to antibiotics within 48 hours. b. The principal area of involvement is the joints. c. Aspirin is contraindicated. d. Therapeutic management includes administration of gamma globulin and aspirin.

D

When discussing hyperlipidemia with a group of adolescents, the nurse should explain that high levels of what substance are thought to protect against cardiovascular disease? a. Cholesterol c. Low-density lipoproteins (LDLs) b. Triglycerides d. High-density lipoproteins (HDLs).

D

When using intermittent auscultation (IA) for fetal heart rate, nurses should be aware that: a. they can be expected to cover only two or three patients when IA is the primary method of fetal assessment. b. the best course is to use the descriptive terms associated with electronic fetal monitoring (EFM) when documenting results. c. if the heartbeat cannot be found immediately, a shift must be made to EFM. d. ultrasound can be used to find the fetal heartbeat and reassure the mother if initial difficulty was a factor.

D

Which clinical manifestation would be seen in a child with chronic renal failure? a. Hypotension c. Hypokalemia b. Massive hematuria d. Unpleasant uremic breath odor

D

Which diagnostic finding is present when a child has primary nephrotic syndrome? a. Hyperalbuminemia b. Positive ASO titer c. Leukocytosis d. Proteinuria

D

Which fetal heart rate (FHR) finding would concern the nurse during labor? a. Accelerations with fetal movement b. Early decelerations c. An average FHR of 126 beats/min d. Late decelerations

D

Which information should the nurse teach workers at a day care center about respiratory syncytial virus (RSV)? a. RSV is transmitted through particles in the air. b. RSV can live on skin or paper for up to a few seconds after contact. c. RSV can survive on nonporous surfaces for about 60 minutes. d. Frequent hand washing can decrease the spread of the virus.

D

Which intervention is appropriate when examining a male infant for cryptorchidism? a. Cooling the examiners hands c. Eliciting the cremasteric reflex b. Taking a rectal temperature d. Warming the room

D

Which postoperative intervention should be questioned for a child after a cardiac catheterization? a. Continue intravenous (IV) fluids until the infant is tolerating oral fluids. b. Check the dressing for bleeding. c. Assess peripheral circulation on the affected extremity. d. Keep the affected leg flexed and elevated.

D

Which statement is characteristic of acute otitis media (AOM)? a. The etiology is unknown. b. Permanent hearing loss often results. c. It can be treated by intramuscular antibiotics. d. It is treated with a broad range of antibiotics.

D

Which vitamin supplements are necessary for children with cystic fibrosis? a. Vitamin C and calcium c. Magnesium b. Vitamins B6 and B12 d. Vitamins A, D, E, and K

D

While evaluating an external monitor tracing of a woman in active labor whose labor is being induced, the nurse notes that the fetal heart rate (FHR) begins to decelerate at the onset of several contractions and returns to baseline before each contraction ends. The nurse should: a. change the woman's position. b. discontinue the oxytocin infusion. c. insert an internal monitor. d. document the finding in the patient's record.

D

Why do infants and young children quickly have respiratory distress in acute and chronic alterations of the respiratory system? a. They have a widened, shorter airway. b. There is a defect in their sucking ability. c. The gag reflex increases mucus production. d. Mucus and edema obstruct small airways.

D

You are evaluating the fetal monitor tracing of your patient, who is in active labor. Suddenly you see the fetal heart rate (FHR) drop from its baseline of 125 beats/min down to 80 beats/min. You reposition the mother, provide oxygen, increase intravenous (IV) fluid, and perform a vaginal examination. The cervix has not changed. Five minutes have passed, and the fetal heart rate remains in the 80s. What additional nursing measures should you take? a. Call for staff assistance. b. Insert a Foley catheter. c. Start Pitocin. d. Notify the care provider immediately.

D

A placenta previa in which the placental edge just reaches the internal os is more commonly known as: a. Total c. Complete b. Partial d. Marginal

D A placenta previa that does not cover any part of the cervix is termed marginal. With a total placenta previa, the placenta completely covers the os. When the patient experiences a partial placenta previa, the lower border of the placenta is within 3 cm of the internal cervical os but does not completely cover the os. A complete placenta previa is termed total. The placenta completely covers the internal cervical os.

Your patient has been receiving magnesium sulfate for 20 hours for treatment of preeclampsia. She just delivered a viable infant girl 30 minutes ago. What uterine findings would you expect to observe/assess in this client? a. Absence of uterine bleeding in the postpartum period b. A fundus firm below the level of the umbilicus c. Scant lochia flow d. A boggy uterus with heavy lochia flow

D Because of the tocolytic effects of magnesium sulfate, this patient most likely would have a boggy uterus with increased amounts of bleeding and a heavy lochia flow in the postpartum period.

As related to the care of the patient with miscarriage, nurses should be aware that: a. It is a natural pregnancy loss before labor begins. b. It occurs in fewer than 5% of all clinically recognized pregnancies. c. It often can be attributed to careless maternal behavior such as poor nutrition or excessive exercise. d. If it occurs before the twelfth week of pregnancy, it may manifest only as moderate discomfort and blood loss.

D Before the sixth week the only evidence may be a heavy menstrual flow. After the twelfth week more severe pain, similar to that of labor, is likely. Miscarriage is a natural pregnancy loss, but by definition it occurs before 20 weeks of gestation, before the fetus is viable. Miscarriages occur in approximately 10% to 15% of all clinically recognized pregnancies. Miscarriage can be caused by a number of disorders or illnesses outside of the mother's control or knowledge.

Nurses should be aware that chronic hypertension: a. Is defined as hypertension that begins during pregnancy and lasts for the duration of pregnancy. b. Is considered severe when the systolic blood pressure (BP) is greater than 140 mm Hg or the diastolic BP is greater than 90 mm Hg. c. Is general hypertension plus proteinuria. d. Can occur independently of or simultaneously with gestational hypertension.

D Hypertension is present before pregnancy or diagnosed before 20 weeks of gestation and persists longer than 6 weeks postpartum. The range for hypertension is systolic BP greater than 140 mm Hg or diastolic BP greater than 90 mm Hg. It becomes severe with a diastolic BP of 110 mm Hg or higher. Proteinuria is an excessive concentration of protein in the urine. It is a complication of hypertension, not a defining characteristic.

A woman with preeclampsia has a seizure. The nurse's primary duty during the seizure is to: a. Insert an oral airway. b. Suction the mouth to prevent aspiration. c. Administer oxygen by mask. d. Stay with the client and call for help.

D If a client becomes eclamptic, the nurse should stay her and call for help. Insertion of an oral airway during seizure activity is no longer the standard of care. The nurse should attempt to keep the airway patent by turning the client's head to the side to prevent aspiration. Once the seizure has ended, it may be necessary to suction the client's mouth. Oxygen would be administered after the convulsion has ended.

A 32-year-old primigravida is admitted with a diagnosis of ectopic pregnancy. Nursing care is based on the knowledge that: a. Bed rest and analgesics are the recommended treatment. b. She will be unable to conceive in the future. c. A D&C will be performed to remove the products of conception. d. Hemorrhage is the major concern.

D Severe bleeding occurs if the fallopian tube ruptures. The recommended treatment is to remove the pregnancy before rupture in order to prevent hemorrhaging. If the tube must be removed, the woman's fertility will decrease; however, she will not be infertile. D&C is performed on the inside of the uterine cavity. The ectopic pregnancy is located within the tubes.

A woman at 39 weeks of gestation with a history of preeclampsia is admitted to the labor and birth unit. She suddenly experiences increased contraction frequency of every 1 to 2 minutes; dark red vaginal bleeding; and a tense, painful abdomen. The nurse suspects the onset of: a. Eclamptic seizure. c. Placenta previa. b. Rupture of the uterus. d. Placental abruption.

D Uterine tenderness in the presence of increasing tone may be the earliest finding of premature separation of the placenta (abruptio placentae or placental abruption). Women with hypertension are at increased risk for an abruption. Eclamptic seizures are evidenced by the presence of generalized tonic-clonic convulsions. Uterine rupture manifests as hypotonic uterine activity, signs of hypovolemia, and in many cases the absence of pain. Placenta previa manifests with bright red, painless vaginal bleeding.

A child with secondary enuresis who complains of dysuria or urgency should be evaluated for what condition (Select all that apply)? a. Hypocalciuria b. Nephrotic syndrome c. Glomerulonephritis d. Urinary tract infection (UTI) e. Diabetes mellitus

D E

Which description of the phases of the second stage of labor is accurate?

Descent phase: Significant increase in contractions, Ferguson reflux activated, average duration varied

Step 5

Determine dilation, presenting part, status of membranes, and characteristics of amniotic fluid.

The nurse who performs vaginal examinations to assess a woman's progress in labor should:

Discuss the findings with the woman and her partner.

Step 7

Document findings and report to the provider.

When managing the care of a woman in the second stage of labor, the nurse uses various measures to enhance the progress of fetal descent. These measures include:

Encouraging the woman to try various upright positions, including squatting and standing.

Which action is correct when palpation is used to assess the characteristics and pattern of uterine contractions?

Evaluate the intensity by pressing the fingertips into the uterine fundus.

Step 6

Explain findings to the patient.

Group care activities as much as possible.

Fatigue related to energy expenditure during labor and birth

A nulliparous woman who has just begun the second stage of her labor would most likely:

Feel tired yet relieved that the worst is over.

A woman who is gravida 3 para 2 enters the intrapartum unit. The most important nursing assessments are:

Fetal heart rate, maternal vital signs, and the woman's nearness to birth.

For the labor nurse, care of the expectant mother begins with any or all of these situations, with the exception of:

Formulation of the woman's plan of care for labor.

Leopold maneuvers would be an inappropriate method of assessment to determine:

Gender of the fetus.

Because the risk for childbirth complications may be revealed, nurses should know that the point of maximal intensity (PMI) of the fetal heart tone (FHT) is:

Heard lower and closer to the midline of the mother's abdomen as the fetus descends and rotates internally.

What is an essential part of nursing care for the laboring woman?

Helping the woman manage the pain

D

How should the nurse explain positioning for a lumbar puncture to a 5-year-old child? a. "You will be on your knees with your head down on the table." b. "You will be able to sit up with your chin against your chest." c. "You will be on your side with the head of your bed slightly raised." d. "You will lie on your side and bend your knees so that they touch your chin."

A pregnant woman is in her third trimester. She asks the nurse to explain how she can tell true labor from false labor. The nurse would explain that "true" labor contractions:

Increase with activity such as ambulation.

A woman who is 39 weeks pregnant expresses fear about her impending labor and how she will manage. The nurse's best response is:

It's normal to be anxious about labor. Let's discuss what makes you afraid."

The most critical nursing action in caring for the newborn immediately after birth is:

Keeping the newborn's airway clear.The care given immediately after the birth focuses on assessing and stabilizing the newborn.

If a woman complains of back labor pain, the nurse could best suggest that she:

Lean over a birth ball with her knees on the floor.

implement a number of care measures to help the client view the childbirth experience in a positive manner. Which intervention would be key for the nurse to use while providing care?

Limiting the number of procedures that invade her body

When implementing care, the nurse would anticipate that a woman from which country would have the father of the baby in attendance?

Mexico

What is an expected characteristic of amniotic fluid?

Pale, straw color with small white particles

Step 2

Position the woman to prevent supine hypotension.

The nurse thoroughly dries the infant immediately after birth primarily to:

Reduce heat loss from evaporation.

Continue to provide comfort measures and minimize distractions.

Risk for impaired individual coping

Encourage frequent voiding and catheterize if necessary.

Risk for impaired urinary elimination

After an emergency birth, the nurse encourages the woman to breastfeed her newborn. The primary purpose of this activity is to:

Stimulate the uterus to contract Stimulation of the nipples through breastfeeding or manual stimulation causes the release of oxytocin and prevents maternal hemorrhage.

The nurse expects to administer an oxytocic , Pitocin, Methergine to a woman after expulsion of her placenta to:

Stimulate uterine contraction.

B

The Glasgow Coma Scale consists of an assessment of: a. Pupil reactivity and motor response. b. Eye opening and verbal and motor responses. c. Level of consciousness and verbal response. d. Intracranial pressure (ICP) and level of consciousness.

C D

The MOST appropriate nursing interventions when caring for a child experiencing a seizure include: (Select all that apply.) A. restraining the child when a seizure occurs to prevent bodily harm. B. placing a padded tongue between the teeth if they become clenched. C. avoid suctioning the child during the seizure. D. describing and documenting the seizure activity observed. E. applying supplemental oxygen after inserting an artificial oral airway.

A means of controlling the birth of the fetal head with a vertex presentation is:

The Ritgen maneuver.The Ritgen maneuver extends the head during the actual birth and protects the perineum. Gentle, steady pressure against the fundus of the uterus facilitates vaginal birth.

The nurse recognizes that a woman is in true labor when she states:

The contractions in my uterus are getting stronger and closer together."Regular, strong contractions with the presence of cervical change indicate that the woman is experiencing true labor.

Through vaginal examination the nurse determines that a woman is 4 cm dilated, and the external fetal monitor shows uterine contractions every 3.5 to 4 minutes. The nurse would report this as:First stage, active phase.

The first stage, active phase of maternal progress indicates that the woman is in the active phase of the first stage of labor.

C

The initial clinical manifestation of generalized seizures is: a. Being confused. b. Feeling frightened. c. Losing consciousness. d. Seeing flashing lights.

C

The most common clinical manifestation of brain tumors in children is: a. Irritability. b. Seizures. c. Headaches and vomiting. d. Fever and poor fine motor control.

D

The mother of a 1-month-old infant tells the nurse that she worries that her baby will get meningitis like her oldest son did when he was an infant. The nurse should base her response on knowing that: a. Meningitis rarely occurs during infancy. b. Often a genetic predisposition to meningitis is found. c. Vaccination to prevent all types of meningitis is now available. d. Vaccination to prevent Haemophilus influenzae type b meningitis has decreased the frequency of this disease in children.

B

The nurse has received report on four children. Which child should the nurse assess first? a. A school-age child in a coma with stable vital signs b. A preschool child with a head injury and decreasing level of consciousness c. An adolescent admitted after a motor vehicle accident who is oriented to person and place d. A toddler in a persistent vegetative state with a low-grade fever

D

The nurse is assessing a child who was just admitted to the hospital for observation after a head injury. The most essential part of the nursing assessment to detect early signs of a worsening condition is: a. Posturing. b. Focal neurologic signs. c. Vital signs. d. Level of consciousness.

C

The nurse is caring for a 2-year-old girl who is unconscious but stable following a car accident. Her parents are staying at the bedside most of the time. An appropriate nursing intervention is to: A. suggest that the parents go home until she is alert enough to know that they are present. B. use ointment on her lips but do not attempt to cleanse her teeth until swallowing returns. C. encourage the parents to hold, talk, and sing to her as they usually would. D. position her with proper body alignment and head of bed lowered 15 degrees.

C

The nurse is caring for a child with multiple injuries who is comatose. The nurse should recognize that pain: A. cannot occur if the child is comatose. B. may occur if the child regains consciousness. C. requires astute nursing assessment and management. D. is best assessed by family members who are familiar with the child.

D

The nurse is caring for a child with severe head trauma after a car accident. Which is an ominous sign that often precedes death? a. Papilledema b. Doll's head maneuver c. Delirium d. Periodic and irregular breathing

C D E

The nurse is caring for a neonate with suspected meningitis. Which clinical manifestations should the nurse prepare to assess if meningitis is confirmed (Select all that apply)? a. Headache b. Photophobia c. Bulging anterior fontanel d. Weak cry e. Poor muscle tone

A

The nurse is caring for a toddler who has had surgery for a brain tumor. During an assessment, the nurse notes that the child is becoming irritable and pupils are unequal and sluggish. The MOST appropriate nursing action is to: A. notify the health care provider immediately. B. document level of consciousness. C. observe closely for signs of increased intracranial pressure (ICP). D. administer pain medication and assess for response.

B

The nurse is closely monitoring a child who is unconscious after a fall and notices that the child suddenly has a fixed and dilated pupil. The nurse should interpret this as: a. Eye trauma. b. Neurosurgical emergency. c. Severe brainstem damage. d. Indication of brain death.

C

The nurse is discussing long-term care with the parents of a child who has a ventriculoperitoneal shunt to correct hydrocephalus. In the discussion the nurse should include that: A. parental protection is essential until the child reaches adulthood. B. cognitive impairment is to be expected with hydrocephalus. C. shunt malfunction or infection requires immediate treatment. D. most usual childhood activities must be restricted.

A

The nurse is doing a neurologic assessment on a child whose level of consciousness has been variable since sustaining a cervical neck injury 12 hours ago. The MOST appropriate nursing assessment in this case is: A. reactivity of pupils. B. doll's head maneuver. C. oculovestibular response. D. funduscopic examination to identify papilledema.

B D E

The nurse is monitoring an infant for signs of increased intracranial pressure (ICP). Which are late signs of increased ICP in an infant (Select all that apply)? a. Tachycardia b. Alteration in pupil size and reactivity c. Increased motor response d. Extension or flexion posturing e. Cheyne-Stokes respirations

D

The nurse is performing a Glasgow Coma Scale (GCS) on a school-age child with a head injury. The child opens eyes spontaneously, obeys commands, and is oriented to person, time, and place. Which is the score the nurse should record? a. 8 b. 13 c. 11 d. 15

A

The nurse is planning care for a school-age child with bacterial meningitis. The plan should include: A. keeping environmental stimuli at a minimum. B. avoiding giving pain medications that could dull sensorium. C. measuring head circumference to assess developing complications. D. having child move head side to side at least every 2 hours.

B

The nurse is preparing a school-age child for a computed tomography (CT) scan to assess cerebral function. When preparing the child for the scan, which statement should the nurse include? a. "Pain medication will be given." b. "The scan will not hurt." c. "You will be able to move once the equipment is in place." d. "Unfortunately no one can remain in the room with you during the test."

Nurses alert to signs of the onset of the second stage of labor can be certain that this stage has begun when:

The nurse is unable to feel the cervix during a vaginal examination.The only certain objective sign that the second stage has begun is the inability to feel the cervix because it is fully dilated and effaced.

A

The nurse who is concerned about increased intracranial pressure in an infant should assess for: A. irritability B. photophobia. C. pulsating anterior fontanel. D. vomiting and diarrhea.

It is paramount for the obstetric nurse to understand the regulatory procedures and criteria for admitting a woman to the hospital labor unit. Which guideline is an important legal requirement of maternity care?

The patient's weight gain is calculated to determine whether she is at greater risk for cephalopelvic disproportion (CPD) and cesarean birth.

When assessing a multiparous woman who has just given birth to an 8 lb boy, nurse notes that the woman's fundus is firm and has become globular in shape. A gush of dark red blood comes from her vagina. nurse concludes that:

The placenta has separated.

B

The postoperative care of a preschool child who has had a brain tumor removed should include which information? A. Colorless drainage is to be expected. B. Close supervision is needed while the child is regaining consciousness. C. Positioning is on the side in the Trendelenburg position. D. Analgesics are contraindicated because of altered consciousness.

A

The priority nursing intervention when a child is unconscious after a fall is to: a. Establish an adequate airway. b. Perform neurologic assessment. c. Monitor intercranial pressure. d. Determine whether a neck injury is present.

C

The temperature of an adolescent who is unconscious is 105° F. The PRIORITY nursing action is to: A. continue to monitor temperature. B. initiate a pain assessment. C. apply a hypothermia blanket. D. administer acetaminophen or ibuprofen.

With regard to a woman's intake and output during labor, nurses should be aware that:

The tradition of restricting the laboring woman to clear liquids and ice chips is being challenged because regional anesthesia is used more often than general anesthesia.

A C E

The treatment of brain tumors in children consists of which therapies (Select all that apply)? a. Surgery b. Bone marrow transplantation c. Chemotherapy d. Stem cell transplantation e. Radiation f. Myelography

When assessing a woman in the first stage of labor, the nurse recognizes that the most conclusive sign that uterine contractions are effective would be:Dilation of the cervix.

The vaginal examination reveals whether the woman is in true labor

B

The vector reservoir for agents causing viral encephalitis in the United States is: a. Tarantula spiders. b. Mosquitoes and ticks. c. Carnivorous wild animals. d. Domestic and wild animals.

Which nursing assessment indicates that a woman who is in second-stage labor is almost ready to give birth?

The vulva bulges and encircles the fetal head.

The nurse knows that the second stage of labor, the descent phase, has begun when:

The woman experiences a strong urge to bear down.

The primary difference between the labor of a nullipara and that of a multipara is the:

Total duration of labor.

Step 1

Use sterile gloves and soluble gel for lubrication.

D

What action may be beneficial in reducing the risk of Reye's syndrome? a. Immunization against the disease b. Medical attention for all head injuries c. Prompt treatment of bacterial meningitis d. Avoidance of aspirin and ibuprofen for children with varicella or those suspected of having influenza

At 1 minute after birth, the nurse assesses the newborn to assign an Apgar score. The apical heart rate is 110 bpm, and the infant is crying vigorously with the limbs flexed. The infant's trunk is pink, but the hands and feet are blue.

What is the correct Apgar score for this infant? The Apgar score is 9 because 1 point is deducted from the total score of 10 for the infant's blue hands and feet.

D

What nursing intervention is used to prevent increased intracranial pressure (ICP) in an unconscious child? A. Suctioning child frequently B. Providing environmental stimulation C. Turning head side to side every hour D. Avoiding activities that cause pain or crying

Under which circumstance would it be unnecessary for the nurse to perform a vaginal examination?

When accelerations of the fetal heart rate (FHR) are noted An accelerated FHR is a positive sign; however, variable decelerations merit a vaginal examination

A

When caring for the child with Reye's syndrome, the priority nursing intervention is to: a. Monitor intake and output. b. Observe for petechiae. c. Prevent skin breakdown. d. Do range-of-motion (ROM) exercises.

B

When taking the history of a child hospitalized with Reye's syndrome, the nurse should not be surprised that a week ago the child had recovered from: a. Measles. b. Varicella. c. Meningitis. d. Hepatitis.

When planning care for a laboring woman whose membranes have ruptured, the nurse recognizes that the woman's risk for Intrauterine infection has increased

When the membranes rupture, microorganisms from the vagina can ascend into the amniotic sac and cause chorioamnionitis and placentitis.

A

Which clinical manifestations would suggest hydrocephalus in a neonate? a. Bulging fontanel and dilated scalp veins b. Closed fontanel and high-pitched cry c. Constant low-pitched cry and restlessness d. Depressed fontanel and decreased blood pressure

A

Which information should the nurse give to a child who is to have magnetic resonance imaging (MRI) of the brain? a. "Your head will be restrained during the procedure." b. "You will have to drink a special fluid before the test." c. "You will have to lie flat after the test is finished." d. "You will have electrodes placed on your head with glue."

C

Which neurologic diagnostic test gives a visualized horizontal and vertical cross section of the brain at any axis? a. Nuclear brain scan b. Echoencephalography c. Computed tomography (CT) scan d. Magnetic resonance imaging (MRI)

A

Which statement best describes a neuroblastoma? a. Diagnosis is usually made after metastasis occurs. b. Early diagnosis is usually possible because of the obvious clinical manifestations. c. It is the most common brain tumor in young children. d. It is the most common benign tumor in young children.

B

Which statement best describes a subdural hematoma? a. Bleeding occurs between the dura and the skull. b. Bleeding occurs between the dura and the cerebrum. c. Bleeding is generally arterial, and brain compression occurs rapidly. d. The hematoma commonly occurs in the parietotemporal region.

C

Which term is used to describe a child's level of consciousness when the child can be aroused with stimulation? a. Stupor b. Confusion c. Obtundation d. Disorientation

C

Which type of fracture describes traumatic separation of cranial sutures? a. Basilar b. Compound c. Diastatic d. Depressed

C

Which type of seizure involves both hemispheres of the brain? a. Focal b. Partial c. Generalized d. Acquired

A

Which type of seizure may be difficult to detect? a. Absence b. Simple partial c. Generalized d. Complex partial

D

Why are infants particularly vulnerable to acceleration-deceleration head injuries? A. The anterior fontanel is not yet closed. B. The nervous tissue is not well developed. C. The scalp of the head has extensive vascularity. D. Musculoskeletal support of head is insufficient.

Which term is used when a patient remains in a deep sleep, responsive only to vigorous and repeated stimulation? a. Coma b. Stupor c. Obtundation d. Persistent vegetative state

b. Stupor

A 5-year-old girl sustained a concussion when she fell out of a tree. In preparation for discharge, the nurse is discussing home care with her mother. Which statement made by the mother indicates a correct understanding of the teaching? a. "I should expect my child to have a few episodes of vomiting." b. "If I notice sleep disturbances, I should contact the physician immediately." c. "I should expect my child to have some behavioral changes after the accident." d. "If I notice diplopia, I will have my child rest for 1 hour."

c. "I should expect my child to have some behavioral changes after the accident."

Which statement is most descriptive of a concussion? a. Petechial hemorrhages cause amnesia. b. Visible bruising and tearing of cerebral tissue occur. c. It is a transient, reversible neuronal dysfunction. d. A slight lesion develops remote from the site of trauma.

c. It is a transient, reversible neuronal dysfunction.

A primigravida at 39 weeks of gestation is observed for 2 hours in the intrapartum unit. The fetal heart rate has been normal. Contractions are 5 to 9 minutes apart, 20 to 30 seconds in duration, and of mild intensity. Cervical

dilation is 1 to 2 cm and uneffaced (unchanged from admission). Membranes are intact. The nurse should expect the woman to be:Discharged home to await the onset of true labor.

For women who have a history of sexual abuse, a number of traumatic memories may be triggered during labor. The woman may fight the labor process and react with pain or anger. Alternately, she may become a passive player and

emotionally absent herself from the process The nurse is in a unique position of being able to assist the client to associate the sensations of labor with the process of childbirth and not the past abuse. The nurse can

Women who have participated in childbirth education classes often bring a "birth bag" or "Lamaze bag" with them to the hospital. These items often assist in reducing stress and providing comfort measures. The nurse caring

for women in labor should be aware of common items that a client may bring, including Rolling pin.Tennis balls.Pillow.Stuffed animal or photo.

nurse teaches a pregnant woman about the characteristics of true labor contractions. nurse evaluates the woman's understanding of the instructions when she states, "True labor contractions will Continue and get stronger even if I

relax and take a shower."They may become intense during walking and continue despite comfort measures. Typically true labor contractions are felt in the lower back, radiating to the lower portion of the abdomen


Conjuntos de estudio relacionados

Chapter 17: The uterus and vagina

View Set

Chapter 20 Concept Overview Videos

View Set

OCR GCSE Computer Science - Unit 1 Revision

View Set

RNN - LSTM - Transfer Learning Let 6

View Set

Assessing for Apical-Radial Pulse Deficit

View Set

Classical Music (Listening Exam)

View Set